A Spoonful of Honey Helps a Coughing Child Sleep

Article Type
Changed
Tue, 12/13/2016 - 12:08
Display Headline
A Spoonful of Honey Helps a Coughing Child Sleep
Finally, we have a safe and effective alternative to OTC cough and cold remedies for young children with upper respiratory infections.

PRACTICE CHANGER
When a parent brings in a child (ages 1 to 5 years) with cough, runny nose, and other symptoms of a viral upper respiratory infection (URI), recommend that honey be given at bedtime.1

STRENGTH OF RECOMMENDATION
A: Based on a well-designed, randomized controlled trial.1

ILLUSTRATIVE CASE
A mother brings in her 18-month-old son because he’s had a runny nose and low-grade fever for the past four days—and a cough that kept them both up last night. You diagnose a viral URI, and she requests a strong cough medicine so he (and she) can get a good night’s sleep. What can you recommend that is both safe and effective for a child of this age?

Primary care office visits by coughing kids with URIs are common. In addition to the cost of such visits, Americans spend about $3.5 billion a year on OTC cough and cold remedies—often giving them to young children.

It’s not enough to tell parents what not to do
As clinicians (and parents), we understand the desire to give a coughing child something to ease the symptoms. We also know that OTC cough and cold medications can lead to serious complications and even death. Between 1983 and 2007, 118 pediatric deaths were attributed to the misuse of such preparations.2 And in a three-year span (2005 to 2008), the American Association of Poison Control Centers received 64,658 calls related to exposure to cough and cold remedies in children younger than 2; 28 of them resulted in a major adverse reaction or death.3

The FDA recommends against the use of OTC cough and cold medications in children younger than 2 years,4 and the American Academy of Pediatrics has issued strict warnings about the use of OTC cough and cold preparations in children younger than 6.5 But warning parents of the dangers of giving these remedies to young children without offering an alternative doesn’t satisfy anyone’s needs, and many parents continue to use them.

What about honey?
A study published in 2007 evaluated buckwheat honey and found it to be superior to no treatment and equal to honey-flavored dextromethorphan in reducing cough severity and improving sleep for children and their parents.6 Honey is known to have both antioxidant and antimicrobial properties—a possible scientific explanation for its effect. Before honey could be recommended for kids with URIs, however, more evidence of its efficacy was needed.

STUDY SUMMARY
Honey reduces cough frequency and severity

Cohen et al sought to determine whether honey, administered before bedtime, would decrease coughing in children between the ages of 1 and 5—and improve sleep for both the children and their caregivers.1 They enrolled 300 children with a nocturnal cough of < 7 days’ duration and a diagnosis of URI in a one-night study.

Children were excluded if they had signs or symptoms of asthma, pneumonia, sinusitis, allergic rhinitis, or laryngotracheobronchitis, or if they had been given any cough remedy (including honey) the night before. Parents completed a five-question survey, using a 7-point Likert scale, to assess the child’s cough and both the child’s and parents’ sleep the previous night. Only children whose parents rated their child’s cough severity ≥ 3 for two of the three related questions were included in the trial.

The study had a double-blind randomized design, with four treatment arms. Three groups received 10 g (about 1.5 tsp) of one of three types of honey: eucalyptus, citrus, or labiatae (derived from plants including sage, mint, and thyme); the fourth group received a placebo of silan date extract, which is similar to honey in color, texture, and taste.

Children in all four groups received the preparation 30 minutes before bedtime. Neither the parents, the physicians, nor the study coordinators knew which preparation the children received. The following day, research assistants telephoned the parent who had completed the initial survey and asked the same five questions. The primary outcome measure was the change in cough frequency from the night before to the night after treatment. Secondary measures included cough severity and the effect on sleep for both the child and the parent.

Of the 300 children initially enrolled, 270 (90%) completed the trial, with an even distribution among the groups. While there were improvements across all outcomes for both the treatment and placebo groups, the changes were statistically significant only in the treatment groups.

There were no significant differences in efficacy noted among the three types of honey. Adverse effects (stomachache, nausea, or vomiting) were noted by four parents in the treatment groups and one in the placebo group, a difference that was not statistically significant.

 

 

WHAT’S NEW?
We have more evidence of honey’s efficacy

For children older than 1 with a viral URI, we can now recommend 1.5 tsp honey to be given prior to bedtime as a cough remedy. This may reduce the use of potentially harmful and often ineffective OTC cough and cold remedies.

CAVEATS
Honey is unsafe for the youngest children
An obvious limitation of this study was its brevity. Although one night of improved cough and sleep is important, a study that showed honey’s sustained benefit as a cough suppressant would be more convincing. What’s more, there are safety concerns that are age-related.

Honey is considered unsafe for children younger than 1 because of the risk of botulism. And honey has the potential to increase dental caries if it is given nightly for a prolonged period of time.

We do not know whether all varieties of honey will have the same benefit, and the source of store-bought honey is not always identified. The authors of this study received funding from the Honey Board of Israel.

CHALLENGES TO IMPLEMENTATION
Parents may be reluctant to abandon OTCs

Changing the behavior of parents and other caregivers who are accustomed to treating children with OTC cough and cold remedies is likely to be an uphill battle.

Because honey is readily available, however—often as close as the pantry—and perceived to be safe and nutritious, a recommendation from a trusted clinician could go a long way toward aiding the implementation of its use as an alternative symptom-reliever for kids with cough.           

REFERENCES
1. Cohen HA, Rozen J, Kristal H, et al. Effect of honey on nocturnal cough and sleep quality: a double-blind, randomized, placebo-controlled study. Pediatrics. 2012;130:465-471.

2. Dart RC, Paul IM, Bond GR, et al. Pediatric fatalities associated with over the counter (nonprescription) cough and cold medications. Ann Emerg Med. 2009;53:411-417.

3. Srinivasan A, Budnitz D, Shehab N, et al. Infant deaths associated with cough and cold medications—two states, 2005. JAMA. 2007;297:800-801.

4. FDA. Public health advisory: FDA recommends that over-the-counter (OTC) cough and cold products not be used for Infants and children under 2 years of age. www.fda.gov/Drugs/DrugSafety/PostmarketDrugSafetyInformationforPatientsandProviders/DrugSafetyInformationforHeathcareProfessionals/PublicHealthAdvisories/ucm051137.htm. Accessed May 30, 2013.

5. American Academy of Pediatrics. Withdrawal of cold medicines: addressing parent concerns. www.aap.org/en-us/professional-resources/practice-support/Pages/Withdrawal-of-Cold-Medicines-Addressing-Parent-Concerns.aspx. Accessed May 30, 2013.

6. Paul IM, Beiler J, McMonagle A, et al. Effect of honey, dextromethorphan, and no treatment on nocturnal cough and sleep quality for coughing children and their parents. Arch Pediatr Adolesc Med. 2007;161:1140-1146.

Acknowledgement
The PURLs Surveillance System was developed with support from Grant Number UL1RR024999 from the National Center for Research Resources, a Clinical Translational Science Award to the University of Chicago. The content is solely the responsibility of the authors and does not necessarily represent the official views of the National Center for Research Resources or the National Institutes of Health.

Copyright © 2013. The Family Physicians Inquiries Network. All rights reserved.

Reprinted with permission from the Family Physicians Inquiries Network and The Journal of Family Practice. 2013;62(3):145-147.

Author and Disclosure Information

Evan Ashkin, MD, Anne Mounsey, MD

Issue
Clinician Reviews - 23(7)
Publications
Topics
Page Number
28
Legacy Keywords
cough, child, toddler, pediatric patient, OTC, over the counter, cough medicine, cold remedies, honey, viral upper respiratory infection, dextromethorphan
Sections
Author and Disclosure Information

Evan Ashkin, MD, Anne Mounsey, MD

Author and Disclosure Information

Evan Ashkin, MD, Anne Mounsey, MD

Finally, we have a safe and effective alternative to OTC cough and cold remedies for young children with upper respiratory infections.
Finally, we have a safe and effective alternative to OTC cough and cold remedies for young children with upper respiratory infections.

PRACTICE CHANGER
When a parent brings in a child (ages 1 to 5 years) with cough, runny nose, and other symptoms of a viral upper respiratory infection (URI), recommend that honey be given at bedtime.1

STRENGTH OF RECOMMENDATION
A: Based on a well-designed, randomized controlled trial.1

ILLUSTRATIVE CASE
A mother brings in her 18-month-old son because he’s had a runny nose and low-grade fever for the past four days—and a cough that kept them both up last night. You diagnose a viral URI, and she requests a strong cough medicine so he (and she) can get a good night’s sleep. What can you recommend that is both safe and effective for a child of this age?

Primary care office visits by coughing kids with URIs are common. In addition to the cost of such visits, Americans spend about $3.5 billion a year on OTC cough and cold remedies—often giving them to young children.

It’s not enough to tell parents what not to do
As clinicians (and parents), we understand the desire to give a coughing child something to ease the symptoms. We also know that OTC cough and cold medications can lead to serious complications and even death. Between 1983 and 2007, 118 pediatric deaths were attributed to the misuse of such preparations.2 And in a three-year span (2005 to 2008), the American Association of Poison Control Centers received 64,658 calls related to exposure to cough and cold remedies in children younger than 2; 28 of them resulted in a major adverse reaction or death.3

The FDA recommends against the use of OTC cough and cold medications in children younger than 2 years,4 and the American Academy of Pediatrics has issued strict warnings about the use of OTC cough and cold preparations in children younger than 6.5 But warning parents of the dangers of giving these remedies to young children without offering an alternative doesn’t satisfy anyone’s needs, and many parents continue to use them.

What about honey?
A study published in 2007 evaluated buckwheat honey and found it to be superior to no treatment and equal to honey-flavored dextromethorphan in reducing cough severity and improving sleep for children and their parents.6 Honey is known to have both antioxidant and antimicrobial properties—a possible scientific explanation for its effect. Before honey could be recommended for kids with URIs, however, more evidence of its efficacy was needed.

STUDY SUMMARY
Honey reduces cough frequency and severity

Cohen et al sought to determine whether honey, administered before bedtime, would decrease coughing in children between the ages of 1 and 5—and improve sleep for both the children and their caregivers.1 They enrolled 300 children with a nocturnal cough of < 7 days’ duration and a diagnosis of URI in a one-night study.

Children were excluded if they had signs or symptoms of asthma, pneumonia, sinusitis, allergic rhinitis, or laryngotracheobronchitis, or if they had been given any cough remedy (including honey) the night before. Parents completed a five-question survey, using a 7-point Likert scale, to assess the child’s cough and both the child’s and parents’ sleep the previous night. Only children whose parents rated their child’s cough severity ≥ 3 for two of the three related questions were included in the trial.

The study had a double-blind randomized design, with four treatment arms. Three groups received 10 g (about 1.5 tsp) of one of three types of honey: eucalyptus, citrus, or labiatae (derived from plants including sage, mint, and thyme); the fourth group received a placebo of silan date extract, which is similar to honey in color, texture, and taste.

Children in all four groups received the preparation 30 minutes before bedtime. Neither the parents, the physicians, nor the study coordinators knew which preparation the children received. The following day, research assistants telephoned the parent who had completed the initial survey and asked the same five questions. The primary outcome measure was the change in cough frequency from the night before to the night after treatment. Secondary measures included cough severity and the effect on sleep for both the child and the parent.

Of the 300 children initially enrolled, 270 (90%) completed the trial, with an even distribution among the groups. While there were improvements across all outcomes for both the treatment and placebo groups, the changes were statistically significant only in the treatment groups.

There were no significant differences in efficacy noted among the three types of honey. Adverse effects (stomachache, nausea, or vomiting) were noted by four parents in the treatment groups and one in the placebo group, a difference that was not statistically significant.

 

 

WHAT’S NEW?
We have more evidence of honey’s efficacy

For children older than 1 with a viral URI, we can now recommend 1.5 tsp honey to be given prior to bedtime as a cough remedy. This may reduce the use of potentially harmful and often ineffective OTC cough and cold remedies.

CAVEATS
Honey is unsafe for the youngest children
An obvious limitation of this study was its brevity. Although one night of improved cough and sleep is important, a study that showed honey’s sustained benefit as a cough suppressant would be more convincing. What’s more, there are safety concerns that are age-related.

Honey is considered unsafe for children younger than 1 because of the risk of botulism. And honey has the potential to increase dental caries if it is given nightly for a prolonged period of time.

We do not know whether all varieties of honey will have the same benefit, and the source of store-bought honey is not always identified. The authors of this study received funding from the Honey Board of Israel.

CHALLENGES TO IMPLEMENTATION
Parents may be reluctant to abandon OTCs

Changing the behavior of parents and other caregivers who are accustomed to treating children with OTC cough and cold remedies is likely to be an uphill battle.

Because honey is readily available, however—often as close as the pantry—and perceived to be safe and nutritious, a recommendation from a trusted clinician could go a long way toward aiding the implementation of its use as an alternative symptom-reliever for kids with cough.           

REFERENCES
1. Cohen HA, Rozen J, Kristal H, et al. Effect of honey on nocturnal cough and sleep quality: a double-blind, randomized, placebo-controlled study. Pediatrics. 2012;130:465-471.

2. Dart RC, Paul IM, Bond GR, et al. Pediatric fatalities associated with over the counter (nonprescription) cough and cold medications. Ann Emerg Med. 2009;53:411-417.

3. Srinivasan A, Budnitz D, Shehab N, et al. Infant deaths associated with cough and cold medications—two states, 2005. JAMA. 2007;297:800-801.

4. FDA. Public health advisory: FDA recommends that over-the-counter (OTC) cough and cold products not be used for Infants and children under 2 years of age. www.fda.gov/Drugs/DrugSafety/PostmarketDrugSafetyInformationforPatientsandProviders/DrugSafetyInformationforHeathcareProfessionals/PublicHealthAdvisories/ucm051137.htm. Accessed May 30, 2013.

5. American Academy of Pediatrics. Withdrawal of cold medicines: addressing parent concerns. www.aap.org/en-us/professional-resources/practice-support/Pages/Withdrawal-of-Cold-Medicines-Addressing-Parent-Concerns.aspx. Accessed May 30, 2013.

6. Paul IM, Beiler J, McMonagle A, et al. Effect of honey, dextromethorphan, and no treatment on nocturnal cough and sleep quality for coughing children and their parents. Arch Pediatr Adolesc Med. 2007;161:1140-1146.

Acknowledgement
The PURLs Surveillance System was developed with support from Grant Number UL1RR024999 from the National Center for Research Resources, a Clinical Translational Science Award to the University of Chicago. The content is solely the responsibility of the authors and does not necessarily represent the official views of the National Center for Research Resources or the National Institutes of Health.

Copyright © 2013. The Family Physicians Inquiries Network. All rights reserved.

Reprinted with permission from the Family Physicians Inquiries Network and The Journal of Family Practice. 2013;62(3):145-147.

PRACTICE CHANGER
When a parent brings in a child (ages 1 to 5 years) with cough, runny nose, and other symptoms of a viral upper respiratory infection (URI), recommend that honey be given at bedtime.1

STRENGTH OF RECOMMENDATION
A: Based on a well-designed, randomized controlled trial.1

ILLUSTRATIVE CASE
A mother brings in her 18-month-old son because he’s had a runny nose and low-grade fever for the past four days—and a cough that kept them both up last night. You diagnose a viral URI, and she requests a strong cough medicine so he (and she) can get a good night’s sleep. What can you recommend that is both safe and effective for a child of this age?

Primary care office visits by coughing kids with URIs are common. In addition to the cost of such visits, Americans spend about $3.5 billion a year on OTC cough and cold remedies—often giving them to young children.

It’s not enough to tell parents what not to do
As clinicians (and parents), we understand the desire to give a coughing child something to ease the symptoms. We also know that OTC cough and cold medications can lead to serious complications and even death. Between 1983 and 2007, 118 pediatric deaths were attributed to the misuse of such preparations.2 And in a three-year span (2005 to 2008), the American Association of Poison Control Centers received 64,658 calls related to exposure to cough and cold remedies in children younger than 2; 28 of them resulted in a major adverse reaction or death.3

The FDA recommends against the use of OTC cough and cold medications in children younger than 2 years,4 and the American Academy of Pediatrics has issued strict warnings about the use of OTC cough and cold preparations in children younger than 6.5 But warning parents of the dangers of giving these remedies to young children without offering an alternative doesn’t satisfy anyone’s needs, and many parents continue to use them.

What about honey?
A study published in 2007 evaluated buckwheat honey and found it to be superior to no treatment and equal to honey-flavored dextromethorphan in reducing cough severity and improving sleep for children and their parents.6 Honey is known to have both antioxidant and antimicrobial properties—a possible scientific explanation for its effect. Before honey could be recommended for kids with URIs, however, more evidence of its efficacy was needed.

STUDY SUMMARY
Honey reduces cough frequency and severity

Cohen et al sought to determine whether honey, administered before bedtime, would decrease coughing in children between the ages of 1 and 5—and improve sleep for both the children and their caregivers.1 They enrolled 300 children with a nocturnal cough of < 7 days’ duration and a diagnosis of URI in a one-night study.

Children were excluded if they had signs or symptoms of asthma, pneumonia, sinusitis, allergic rhinitis, or laryngotracheobronchitis, or if they had been given any cough remedy (including honey) the night before. Parents completed a five-question survey, using a 7-point Likert scale, to assess the child’s cough and both the child’s and parents’ sleep the previous night. Only children whose parents rated their child’s cough severity ≥ 3 for two of the three related questions were included in the trial.

The study had a double-blind randomized design, with four treatment arms. Three groups received 10 g (about 1.5 tsp) of one of three types of honey: eucalyptus, citrus, or labiatae (derived from plants including sage, mint, and thyme); the fourth group received a placebo of silan date extract, which is similar to honey in color, texture, and taste.

Children in all four groups received the preparation 30 minutes before bedtime. Neither the parents, the physicians, nor the study coordinators knew which preparation the children received. The following day, research assistants telephoned the parent who had completed the initial survey and asked the same five questions. The primary outcome measure was the change in cough frequency from the night before to the night after treatment. Secondary measures included cough severity and the effect on sleep for both the child and the parent.

Of the 300 children initially enrolled, 270 (90%) completed the trial, with an even distribution among the groups. While there were improvements across all outcomes for both the treatment and placebo groups, the changes were statistically significant only in the treatment groups.

There were no significant differences in efficacy noted among the three types of honey. Adverse effects (stomachache, nausea, or vomiting) were noted by four parents in the treatment groups and one in the placebo group, a difference that was not statistically significant.

 

 

WHAT’S NEW?
We have more evidence of honey’s efficacy

For children older than 1 with a viral URI, we can now recommend 1.5 tsp honey to be given prior to bedtime as a cough remedy. This may reduce the use of potentially harmful and often ineffective OTC cough and cold remedies.

CAVEATS
Honey is unsafe for the youngest children
An obvious limitation of this study was its brevity. Although one night of improved cough and sleep is important, a study that showed honey’s sustained benefit as a cough suppressant would be more convincing. What’s more, there are safety concerns that are age-related.

Honey is considered unsafe for children younger than 1 because of the risk of botulism. And honey has the potential to increase dental caries if it is given nightly for a prolonged period of time.

We do not know whether all varieties of honey will have the same benefit, and the source of store-bought honey is not always identified. The authors of this study received funding from the Honey Board of Israel.

CHALLENGES TO IMPLEMENTATION
Parents may be reluctant to abandon OTCs

Changing the behavior of parents and other caregivers who are accustomed to treating children with OTC cough and cold remedies is likely to be an uphill battle.

Because honey is readily available, however—often as close as the pantry—and perceived to be safe and nutritious, a recommendation from a trusted clinician could go a long way toward aiding the implementation of its use as an alternative symptom-reliever for kids with cough.           

REFERENCES
1. Cohen HA, Rozen J, Kristal H, et al. Effect of honey on nocturnal cough and sleep quality: a double-blind, randomized, placebo-controlled study. Pediatrics. 2012;130:465-471.

2. Dart RC, Paul IM, Bond GR, et al. Pediatric fatalities associated with over the counter (nonprescription) cough and cold medications. Ann Emerg Med. 2009;53:411-417.

3. Srinivasan A, Budnitz D, Shehab N, et al. Infant deaths associated with cough and cold medications—two states, 2005. JAMA. 2007;297:800-801.

4. FDA. Public health advisory: FDA recommends that over-the-counter (OTC) cough and cold products not be used for Infants and children under 2 years of age. www.fda.gov/Drugs/DrugSafety/PostmarketDrugSafetyInformationforPatientsandProviders/DrugSafetyInformationforHeathcareProfessionals/PublicHealthAdvisories/ucm051137.htm. Accessed May 30, 2013.

5. American Academy of Pediatrics. Withdrawal of cold medicines: addressing parent concerns. www.aap.org/en-us/professional-resources/practice-support/Pages/Withdrawal-of-Cold-Medicines-Addressing-Parent-Concerns.aspx. Accessed May 30, 2013.

6. Paul IM, Beiler J, McMonagle A, et al. Effect of honey, dextromethorphan, and no treatment on nocturnal cough and sleep quality for coughing children and their parents. Arch Pediatr Adolesc Med. 2007;161:1140-1146.

Acknowledgement
The PURLs Surveillance System was developed with support from Grant Number UL1RR024999 from the National Center for Research Resources, a Clinical Translational Science Award to the University of Chicago. The content is solely the responsibility of the authors and does not necessarily represent the official views of the National Center for Research Resources or the National Institutes of Health.

Copyright © 2013. The Family Physicians Inquiries Network. All rights reserved.

Reprinted with permission from the Family Physicians Inquiries Network and The Journal of Family Practice. 2013;62(3):145-147.

Issue
Clinician Reviews - 23(7)
Issue
Clinician Reviews - 23(7)
Page Number
28
Page Number
28
Publications
Publications
Topics
Article Type
Display Headline
A Spoonful of Honey Helps a Coughing Child Sleep
Display Headline
A Spoonful of Honey Helps a Coughing Child Sleep
Legacy Keywords
cough, child, toddler, pediatric patient, OTC, over the counter, cough medicine, cold remedies, honey, viral upper respiratory infection, dextromethorphan
Legacy Keywords
cough, child, toddler, pediatric patient, OTC, over the counter, cough medicine, cold remedies, honey, viral upper respiratory infection, dextromethorphan
Sections
Article Source

PURLs Copyright

Inside the Article

Prescribing an Antibiotic? Pair It With Probiotics

Article Type
Changed
Tue, 12/13/2016 - 12:08
Display Headline
Prescribing an Antibiotic? Pair It With Probiotics

PRACTICE CHANGER
Recommend that patients taking antibiotics also take probiotics, which have been found to be effective both for the prevention and treatment of antibiotic-associated diarrhea (AAD).1

STRENGTH OF

RECOMMENDATION
A: Based on a systematic review and meta-analysis of randomized controlled trials (RCTs).

ILLUSTRATIVE CASE
When you prescribe an antibiotic for a 45-year-old patient with Helicobacter pylori, he worries that the medication will cause diarrhea. Should you recommend that he take probiotics?

More than a third of patients taking antibiotics develop AAD,2 and in 17% of cases, AAD is fatal.3,4 Although the diarrhea may be the result of increased gastrointestinal (GI) motility in some cases, a disruption of the GI flora that normally acts as a barrier to infection and aids in the digestion of carbohydrates is a far more common cause.

Morbidity and mortality are high
AAD is associated with several pathogens, including Clostridium difficile, Clostridium perfringens, Klebsiella oxytoca, and Staphylococcus aureus,2 and varies widely in severity. Pseudomembranous colitis secondary to C difficile is the main cause of AAD-related mortality, which more than doubled from 2002 to 2009.3,4 C difficile infections cost the US health care system up to $1.3 billion annually.5 With such high rates of morbidity and mortality and high health care costs associated with AAD, even a small reduction in the number of cases would have a big impact.

Probiotics replenish the natural GI flora with nonpathogenic organisms. A 2006 meta-analysis of 31 RCTs assessing the efficacy of probiotics for both the prevention of AAD and treatment of C difficile found a pooled relative risk (RR) of 0.43 for AAD in the patients taking probiotics.6 However, many of the studies included in that meta-analysis were small. As a result, in 2010, the Society for Healthcare Epidemiology of America (SHEA) and the Infectious Diseases Society of America (IDSA) recommended against the use of probiotics for the prevention of primary C difficile infection, citing a lack of high-quality evidence.7

Nonetheless, that same year, 98% of gastroenterologists surveyed expressed a belief that probiotics had a role in the treatment of GI illness.8 And in 2011, the Third Yale Working Group on Probiotic Use published recommendations for probiotic use based on expert opinion.9 The meta-analysis detailed in this PURL, which included more than 30 trials published since the 2006 meta-analysis, addressed the efficacy of probiotics for prevention and treatment of AAD.

STUDY SUMMARY
Probiotics significantly reduce AAD
Hempel et al reviewed 82 studies and pooled data from 63 RCTs (N = 11,811) to identify the RR of AAD among patients who received probiotics during antibiotic treatment, compared with those who received no probiotics or were given a placebo.1 The studies encompassed a variety of antibiotics, taken alone or in combination, and several probiotics, including Lactobacillus, Bifidobacterium, Saccharomyces, and some combinations.

The outcome: The pooled RR for AAD in the probiotics groups was 0.58, with a number needed to treat of 13. Although the authors reported that the overall quality of the included trials was poor, a sensitivity analysis of the higher-quality studies yielded similar results.

Subgroup analyses by type of probiotic and duration of antibiotic treatment were also consistent with the overall pooled RR. In subgroup analysis by age, a similar decrease in AAD was found among the youngest patients (ages 0 to 17) and those between the ages of 17 and 65. Among patients older than 65—for whom there were just three studies—a nonsignificant decrease in risk was found. Twenty-three of the studies assessed adverse outcomes, of which none were reported.

WHAT'S NEW
A reason to pair antibiotics and probiotics

This meta-analysis reached a similar conclusion to the 2006 meta-analysis: Probiotics appear to be effective in preventing and treating AAD in children and adults receiving a wide variety of antibiotics for a number of conditions. The results were also consistent with those of a new meta-analysis that looked specifically at one pathogen—and found a reduction of 66% in C difficile–associated diarrhea in patients taking probiotics with their antibiotics.10

CAVEATS
Limited data on the safety

of probiotics exist
There was some heterogeneity among the studies in the meta-analysis by Hempel et al,1 and some of the studies were of poor quality. Because of this, the authors used subgroup and sensitivity analysis, which supported their initial conclusion.

Probiotics have generally been considered safe; however, there have been rare reports of sepsis and fungemia associated with probiotic use, especially in immunosuppressed patients.1 Fifty-nine of the included studies did not assess adverse events, which limited the ability of this meta-analysis to assess safety.1 Patients taking probiotics should be monitored for adverse effects.

CHALLENGES TO IMPLEMENTATION
Lack of guidance on dosing and duration
Since probiotics are considered food supplements, health insurance will not cover the cost (which will likely be more than $20 per month). No single probiotic strain has high-quality evidence; however, most of the RCTs included in the meta-analysis used combinations of Lactobacillus species, which are usually found in OTC antidiarrheal probiotic supplements. No standard dose exists, but dose ranges in RCTs are 107 to 1010 colony-forming units per capsule (taken one to three times daily);1 however, product labels have variable accuracy.11 The duration of treatment ranges from one to three weeks—or as long as the patient continues to take antibiotics. .

 

 

REFERENCES
1. Hempel S, Newberry S, Maher A, et al. Probiotics for the prevention and treatment of antibiotic-associated diarrhea. JAMA. 2012; 307:1959-1969.

2. McFarland LV. Antibiotic-associated diarrhea: epidemiology, trends and treatment. Future Microbiol. 2008;3:563-578.

3. Pepin J, Valiquette L, Cossette B. Mortality attributable to nosocomial Clostridium difficile–associated disease during an epidemic caused by a hypervirulent strain in Quebec. CMAJ. 2005;173:1037-1042.

4. Perry A, Dellon E, Lund J, et al. Burden of gastrointestinal disease in the United States: 2012 Update. Gastroenterology. 2012;143: 1179-1187.

5. Dubberke E, Wertheimer A. A review of current literature on the economic burden of Clostridium difficile infection. Infect Control Hosp Epidemiol. 2009;30:57-66.

6. McFarland L. Meta-analysis of probiotics for the prevention of antibiotic associated diarrhea and the treatment of Clostridium difficile disease. Am J Gastroenterol. 2006;101:

812-822.

7. Cohen S, Gerding D, Johnson S, et al. Clinical practice guidelines for Clostridium difficile infection in adults: 2010 update by the Society for Healthcare Epidemiology of America and the Infectious Diseases Society of America. Infect Control Hosp Epidemiol. 2010;31: 431-455.

8. Williams M, Ha C, Ciorba M. Probiotics as therapy in gastroenterology. J Clin Gastroenterol. 2010;44:631-636.

9. Floch M, Walker A, Madsne K, et al. Recommendations for probiotic use—2011 update. J Clin Gastroenterol. 2011;45(suppl):S168-S171.

10. Johnston BC, Ma SS, Goldenberg JZ, et al. Probiotics for the prevention of Clostridium difficile–associated diarrhea: a systematic review and meta-analysis. Ann Intern Med. 2012;157:878-888.

11. Hamilton-Miller J, Shah S. Deficiencies in microbiological quality and labeling of probiotic supplements. Int J Food Microbiol. 2002; 72:175-176.

Acknowledgement
The PURLs Surveillance System was developed with support from Grant Number UL1RR024999 from the National Center for Research Resources, a Clinical Translational Science Award to the University of Chicago. The content is solely the responsibility of the authors and does not necessarily represent the official views of the National Center for Research Resources or the National Institutes of Health.

Copyright © 2012. The Family Physicians Inquiries Network. All rights reserved.

Reprinted with permission from the Family Physicians Inquiries Network and The Journal of Family Practice. 2012;61:673-674.

Author and Disclosure Information

Blake Rodgers, MD, Kate Kirley, MD, Anne Mounsey, MD

Issue
Clinician Reviews - 23(4)
Publications
Page Number
49-50
Legacy Keywords
Blake Rodgers; MD; Kate Kirley; MD; Anne Mounsey; MD; probiotics, antibiotic-associated diarrhea, antibiotic course, Helicobacter pylori, Clostridium difficile, pseudomembranous colitis, Lactobacillus, Bifidobacterium, Saccharomycesprobiotics, antibiotic-associated diarrhea, antibiotic course, Helicobacter pylori, Clostridium difficile, pseudomembranous colitis, Lactobacillus, Bifidobacterium, Saccharomyces; PURLs
Sections
Author and Disclosure Information

Blake Rodgers, MD, Kate Kirley, MD, Anne Mounsey, MD

Author and Disclosure Information

Blake Rodgers, MD, Kate Kirley, MD, Anne Mounsey, MD

PRACTICE CHANGER
Recommend that patients taking antibiotics also take probiotics, which have been found to be effective both for the prevention and treatment of antibiotic-associated diarrhea (AAD).1

STRENGTH OF

RECOMMENDATION
A: Based on a systematic review and meta-analysis of randomized controlled trials (RCTs).

ILLUSTRATIVE CASE
When you prescribe an antibiotic for a 45-year-old patient with Helicobacter pylori, he worries that the medication will cause diarrhea. Should you recommend that he take probiotics?

More than a third of patients taking antibiotics develop AAD,2 and in 17% of cases, AAD is fatal.3,4 Although the diarrhea may be the result of increased gastrointestinal (GI) motility in some cases, a disruption of the GI flora that normally acts as a barrier to infection and aids in the digestion of carbohydrates is a far more common cause.

Morbidity and mortality are high
AAD is associated with several pathogens, including Clostridium difficile, Clostridium perfringens, Klebsiella oxytoca, and Staphylococcus aureus,2 and varies widely in severity. Pseudomembranous colitis secondary to C difficile is the main cause of AAD-related mortality, which more than doubled from 2002 to 2009.3,4 C difficile infections cost the US health care system up to $1.3 billion annually.5 With such high rates of morbidity and mortality and high health care costs associated with AAD, even a small reduction in the number of cases would have a big impact.

Probiotics replenish the natural GI flora with nonpathogenic organisms. A 2006 meta-analysis of 31 RCTs assessing the efficacy of probiotics for both the prevention of AAD and treatment of C difficile found a pooled relative risk (RR) of 0.43 for AAD in the patients taking probiotics.6 However, many of the studies included in that meta-analysis were small. As a result, in 2010, the Society for Healthcare Epidemiology of America (SHEA) and the Infectious Diseases Society of America (IDSA) recommended against the use of probiotics for the prevention of primary C difficile infection, citing a lack of high-quality evidence.7

Nonetheless, that same year, 98% of gastroenterologists surveyed expressed a belief that probiotics had a role in the treatment of GI illness.8 And in 2011, the Third Yale Working Group on Probiotic Use published recommendations for probiotic use based on expert opinion.9 The meta-analysis detailed in this PURL, which included more than 30 trials published since the 2006 meta-analysis, addressed the efficacy of probiotics for prevention and treatment of AAD.

STUDY SUMMARY
Probiotics significantly reduce AAD
Hempel et al reviewed 82 studies and pooled data from 63 RCTs (N = 11,811) to identify the RR of AAD among patients who received probiotics during antibiotic treatment, compared with those who received no probiotics or were given a placebo.1 The studies encompassed a variety of antibiotics, taken alone or in combination, and several probiotics, including Lactobacillus, Bifidobacterium, Saccharomyces, and some combinations.

The outcome: The pooled RR for AAD in the probiotics groups was 0.58, with a number needed to treat of 13. Although the authors reported that the overall quality of the included trials was poor, a sensitivity analysis of the higher-quality studies yielded similar results.

Subgroup analyses by type of probiotic and duration of antibiotic treatment were also consistent with the overall pooled RR. In subgroup analysis by age, a similar decrease in AAD was found among the youngest patients (ages 0 to 17) and those between the ages of 17 and 65. Among patients older than 65—for whom there were just three studies—a nonsignificant decrease in risk was found. Twenty-three of the studies assessed adverse outcomes, of which none were reported.

WHAT'S NEW
A reason to pair antibiotics and probiotics

This meta-analysis reached a similar conclusion to the 2006 meta-analysis: Probiotics appear to be effective in preventing and treating AAD in children and adults receiving a wide variety of antibiotics for a number of conditions. The results were also consistent with those of a new meta-analysis that looked specifically at one pathogen—and found a reduction of 66% in C difficile–associated diarrhea in patients taking probiotics with their antibiotics.10

CAVEATS
Limited data on the safety

of probiotics exist
There was some heterogeneity among the studies in the meta-analysis by Hempel et al,1 and some of the studies were of poor quality. Because of this, the authors used subgroup and sensitivity analysis, which supported their initial conclusion.

Probiotics have generally been considered safe; however, there have been rare reports of sepsis and fungemia associated with probiotic use, especially in immunosuppressed patients.1 Fifty-nine of the included studies did not assess adverse events, which limited the ability of this meta-analysis to assess safety.1 Patients taking probiotics should be monitored for adverse effects.

CHALLENGES TO IMPLEMENTATION
Lack of guidance on dosing and duration
Since probiotics are considered food supplements, health insurance will not cover the cost (which will likely be more than $20 per month). No single probiotic strain has high-quality evidence; however, most of the RCTs included in the meta-analysis used combinations of Lactobacillus species, which are usually found in OTC antidiarrheal probiotic supplements. No standard dose exists, but dose ranges in RCTs are 107 to 1010 colony-forming units per capsule (taken one to three times daily);1 however, product labels have variable accuracy.11 The duration of treatment ranges from one to three weeks—or as long as the patient continues to take antibiotics. .

 

 

REFERENCES
1. Hempel S, Newberry S, Maher A, et al. Probiotics for the prevention and treatment of antibiotic-associated diarrhea. JAMA. 2012; 307:1959-1969.

2. McFarland LV. Antibiotic-associated diarrhea: epidemiology, trends and treatment. Future Microbiol. 2008;3:563-578.

3. Pepin J, Valiquette L, Cossette B. Mortality attributable to nosocomial Clostridium difficile–associated disease during an epidemic caused by a hypervirulent strain in Quebec. CMAJ. 2005;173:1037-1042.

4. Perry A, Dellon E, Lund J, et al. Burden of gastrointestinal disease in the United States: 2012 Update. Gastroenterology. 2012;143: 1179-1187.

5. Dubberke E, Wertheimer A. A review of current literature on the economic burden of Clostridium difficile infection. Infect Control Hosp Epidemiol. 2009;30:57-66.

6. McFarland L. Meta-analysis of probiotics for the prevention of antibiotic associated diarrhea and the treatment of Clostridium difficile disease. Am J Gastroenterol. 2006;101:

812-822.

7. Cohen S, Gerding D, Johnson S, et al. Clinical practice guidelines for Clostridium difficile infection in adults: 2010 update by the Society for Healthcare Epidemiology of America and the Infectious Diseases Society of America. Infect Control Hosp Epidemiol. 2010;31: 431-455.

8. Williams M, Ha C, Ciorba M. Probiotics as therapy in gastroenterology. J Clin Gastroenterol. 2010;44:631-636.

9. Floch M, Walker A, Madsne K, et al. Recommendations for probiotic use—2011 update. J Clin Gastroenterol. 2011;45(suppl):S168-S171.

10. Johnston BC, Ma SS, Goldenberg JZ, et al. Probiotics for the prevention of Clostridium difficile–associated diarrhea: a systematic review and meta-analysis. Ann Intern Med. 2012;157:878-888.

11. Hamilton-Miller J, Shah S. Deficiencies in microbiological quality and labeling of probiotic supplements. Int J Food Microbiol. 2002; 72:175-176.

Acknowledgement
The PURLs Surveillance System was developed with support from Grant Number UL1RR024999 from the National Center for Research Resources, a Clinical Translational Science Award to the University of Chicago. The content is solely the responsibility of the authors and does not necessarily represent the official views of the National Center for Research Resources or the National Institutes of Health.

Copyright © 2012. The Family Physicians Inquiries Network. All rights reserved.

Reprinted with permission from the Family Physicians Inquiries Network and The Journal of Family Practice. 2012;61:673-674.

PRACTICE CHANGER
Recommend that patients taking antibiotics also take probiotics, which have been found to be effective both for the prevention and treatment of antibiotic-associated diarrhea (AAD).1

STRENGTH OF

RECOMMENDATION
A: Based on a systematic review and meta-analysis of randomized controlled trials (RCTs).

ILLUSTRATIVE CASE
When you prescribe an antibiotic for a 45-year-old patient with Helicobacter pylori, he worries that the medication will cause diarrhea. Should you recommend that he take probiotics?

More than a third of patients taking antibiotics develop AAD,2 and in 17% of cases, AAD is fatal.3,4 Although the diarrhea may be the result of increased gastrointestinal (GI) motility in some cases, a disruption of the GI flora that normally acts as a barrier to infection and aids in the digestion of carbohydrates is a far more common cause.

Morbidity and mortality are high
AAD is associated with several pathogens, including Clostridium difficile, Clostridium perfringens, Klebsiella oxytoca, and Staphylococcus aureus,2 and varies widely in severity. Pseudomembranous colitis secondary to C difficile is the main cause of AAD-related mortality, which more than doubled from 2002 to 2009.3,4 C difficile infections cost the US health care system up to $1.3 billion annually.5 With such high rates of morbidity and mortality and high health care costs associated with AAD, even a small reduction in the number of cases would have a big impact.

Probiotics replenish the natural GI flora with nonpathogenic organisms. A 2006 meta-analysis of 31 RCTs assessing the efficacy of probiotics for both the prevention of AAD and treatment of C difficile found a pooled relative risk (RR) of 0.43 for AAD in the patients taking probiotics.6 However, many of the studies included in that meta-analysis were small. As a result, in 2010, the Society for Healthcare Epidemiology of America (SHEA) and the Infectious Diseases Society of America (IDSA) recommended against the use of probiotics for the prevention of primary C difficile infection, citing a lack of high-quality evidence.7

Nonetheless, that same year, 98% of gastroenterologists surveyed expressed a belief that probiotics had a role in the treatment of GI illness.8 And in 2011, the Third Yale Working Group on Probiotic Use published recommendations for probiotic use based on expert opinion.9 The meta-analysis detailed in this PURL, which included more than 30 trials published since the 2006 meta-analysis, addressed the efficacy of probiotics for prevention and treatment of AAD.

STUDY SUMMARY
Probiotics significantly reduce AAD
Hempel et al reviewed 82 studies and pooled data from 63 RCTs (N = 11,811) to identify the RR of AAD among patients who received probiotics during antibiotic treatment, compared with those who received no probiotics or were given a placebo.1 The studies encompassed a variety of antibiotics, taken alone or in combination, and several probiotics, including Lactobacillus, Bifidobacterium, Saccharomyces, and some combinations.

The outcome: The pooled RR for AAD in the probiotics groups was 0.58, with a number needed to treat of 13. Although the authors reported that the overall quality of the included trials was poor, a sensitivity analysis of the higher-quality studies yielded similar results.

Subgroup analyses by type of probiotic and duration of antibiotic treatment were also consistent with the overall pooled RR. In subgroup analysis by age, a similar decrease in AAD was found among the youngest patients (ages 0 to 17) and those between the ages of 17 and 65. Among patients older than 65—for whom there were just three studies—a nonsignificant decrease in risk was found. Twenty-three of the studies assessed adverse outcomes, of which none were reported.

WHAT'S NEW
A reason to pair antibiotics and probiotics

This meta-analysis reached a similar conclusion to the 2006 meta-analysis: Probiotics appear to be effective in preventing and treating AAD in children and adults receiving a wide variety of antibiotics for a number of conditions. The results were also consistent with those of a new meta-analysis that looked specifically at one pathogen—and found a reduction of 66% in C difficile–associated diarrhea in patients taking probiotics with their antibiotics.10

CAVEATS
Limited data on the safety

of probiotics exist
There was some heterogeneity among the studies in the meta-analysis by Hempel et al,1 and some of the studies were of poor quality. Because of this, the authors used subgroup and sensitivity analysis, which supported their initial conclusion.

Probiotics have generally been considered safe; however, there have been rare reports of sepsis and fungemia associated with probiotic use, especially in immunosuppressed patients.1 Fifty-nine of the included studies did not assess adverse events, which limited the ability of this meta-analysis to assess safety.1 Patients taking probiotics should be monitored for adverse effects.

CHALLENGES TO IMPLEMENTATION
Lack of guidance on dosing and duration
Since probiotics are considered food supplements, health insurance will not cover the cost (which will likely be more than $20 per month). No single probiotic strain has high-quality evidence; however, most of the RCTs included in the meta-analysis used combinations of Lactobacillus species, which are usually found in OTC antidiarrheal probiotic supplements. No standard dose exists, but dose ranges in RCTs are 107 to 1010 colony-forming units per capsule (taken one to three times daily);1 however, product labels have variable accuracy.11 The duration of treatment ranges from one to three weeks—or as long as the patient continues to take antibiotics. .

 

 

REFERENCES
1. Hempel S, Newberry S, Maher A, et al. Probiotics for the prevention and treatment of antibiotic-associated diarrhea. JAMA. 2012; 307:1959-1969.

2. McFarland LV. Antibiotic-associated diarrhea: epidemiology, trends and treatment. Future Microbiol. 2008;3:563-578.

3. Pepin J, Valiquette L, Cossette B. Mortality attributable to nosocomial Clostridium difficile–associated disease during an epidemic caused by a hypervirulent strain in Quebec. CMAJ. 2005;173:1037-1042.

4. Perry A, Dellon E, Lund J, et al. Burden of gastrointestinal disease in the United States: 2012 Update. Gastroenterology. 2012;143: 1179-1187.

5. Dubberke E, Wertheimer A. A review of current literature on the economic burden of Clostridium difficile infection. Infect Control Hosp Epidemiol. 2009;30:57-66.

6. McFarland L. Meta-analysis of probiotics for the prevention of antibiotic associated diarrhea and the treatment of Clostridium difficile disease. Am J Gastroenterol. 2006;101:

812-822.

7. Cohen S, Gerding D, Johnson S, et al. Clinical practice guidelines for Clostridium difficile infection in adults: 2010 update by the Society for Healthcare Epidemiology of America and the Infectious Diseases Society of America. Infect Control Hosp Epidemiol. 2010;31: 431-455.

8. Williams M, Ha C, Ciorba M. Probiotics as therapy in gastroenterology. J Clin Gastroenterol. 2010;44:631-636.

9. Floch M, Walker A, Madsne K, et al. Recommendations for probiotic use—2011 update. J Clin Gastroenterol. 2011;45(suppl):S168-S171.

10. Johnston BC, Ma SS, Goldenberg JZ, et al. Probiotics for the prevention of Clostridium difficile–associated diarrhea: a systematic review and meta-analysis. Ann Intern Med. 2012;157:878-888.

11. Hamilton-Miller J, Shah S. Deficiencies in microbiological quality and labeling of probiotic supplements. Int J Food Microbiol. 2002; 72:175-176.

Acknowledgement
The PURLs Surveillance System was developed with support from Grant Number UL1RR024999 from the National Center for Research Resources, a Clinical Translational Science Award to the University of Chicago. The content is solely the responsibility of the authors and does not necessarily represent the official views of the National Center for Research Resources or the National Institutes of Health.

Copyright © 2012. The Family Physicians Inquiries Network. All rights reserved.

Reprinted with permission from the Family Physicians Inquiries Network and The Journal of Family Practice. 2012;61:673-674.

Issue
Clinician Reviews - 23(4)
Issue
Clinician Reviews - 23(4)
Page Number
49-50
Page Number
49-50
Publications
Publications
Article Type
Display Headline
Prescribing an Antibiotic? Pair It With Probiotics
Display Headline
Prescribing an Antibiotic? Pair It With Probiotics
Legacy Keywords
Blake Rodgers; MD; Kate Kirley; MD; Anne Mounsey; MD; probiotics, antibiotic-associated diarrhea, antibiotic course, Helicobacter pylori, Clostridium difficile, pseudomembranous colitis, Lactobacillus, Bifidobacterium, Saccharomycesprobiotics, antibiotic-associated diarrhea, antibiotic course, Helicobacter pylori, Clostridium difficile, pseudomembranous colitis, Lactobacillus, Bifidobacterium, Saccharomyces; PURLs
Legacy Keywords
Blake Rodgers; MD; Kate Kirley; MD; Anne Mounsey; MD; probiotics, antibiotic-associated diarrhea, antibiotic course, Helicobacter pylori, Clostridium difficile, pseudomembranous colitis, Lactobacillus, Bifidobacterium, Saccharomycesprobiotics, antibiotic-associated diarrhea, antibiotic course, Helicobacter pylori, Clostridium difficile, pseudomembranous colitis, Lactobacillus, Bifidobacterium, Saccharomyces; PURLs
Sections
Article Source

PURLs Copyright

Inside the Article

Prescribing an antibiotic? Pair it with probiotics

Article Type
Changed
Thu, 07/16/2020 - 14:31
Display Headline
Prescribing an antibiotic? Pair it with probiotics
PRACTICE CHANGER

Recommend that patients taking antibiotics also take probiotics, which have been found to be effective both for the prevention and treatment of antibiotic-associated diarrhea (AAD).1

STRENGTH OF RECOMMENDATION

A: Based on a systematic review and meta-analysis of randomized controlled trials.

Hempel S, Newberry S, Maher A, et al. Probiotics for the prevention and treatment of antibiotic-associated diarrhea. JAMA. 2012;307: 1959-1969.

 

ILLUSTRATIVE CASE

When you prescribe an antibiotic for a 45-year-old patient with Helicobacter pylori, he worries that the medication will cause diarrhea. Should you recommend that he take probiotics?

More than a third of patients taking antibiotics develop AAD,2 and in 17% of cases, AAD is fatal.3,4 Although the diarrhea may be the result of increased gastrointestinal (GI) motility in some cases, a disruption of the GI flora that normally acts as a barrier to infection and aids in the digestion of carbohydrates is a far more common cause.

Morbidity and mortality are high
AAD is associated with several pathogens, including Clostridium difficile, Clostridium perfringens, Klebsiella oxytoca, and Staphylococcus aureus,2 and varies widely in severity. Pseudomembranous colitis secondary to C difficile is the main cause of AAD-related mortality, which more than doubled from 2002 to 2009.3,4 C difficile infections cost the US health care system up to $1.3 billion annually.5 With such high rates of morbidity and mortality and high health care costs associated with AAD, even a small reduction in the number of cases would have a big impact.

Probiotics replenish the natural GI flora with nonpathogenic organisms. A 2006 meta-analysis of 31 randomized controlled trials (RCTs) assessing the efficacy of probiotics for both the prevention of AAD and treatment of C difficile found a pooled relative risk of 0.43 for AAD in the patients taking probiotics.6 However, many of the studies included in that meta-analysis were small. As a result, in 2010, the Society for Healthcare Epidemiology of America (SHEA) and the Infectious Diseases Society of America (IDSA) recommended against the use of probiotics for the prevention of primary C difficile infection, citing a lack of high-quality evidence.7

Nonetheless, that same year, 98% of gastroenterologists surveyed expressed a belief that probiotics had a role in the treatment of GI illness.8 And in 2011, the 3rd Yale Working Group on Probiotic Use published recommendations for probiotic use based on expert opinion.9 The meta-analysis detailed in this PURL, which included more than 30 trials published since the 2006 meta-analysis, addressed the efficacy of probiotics for prevention and treatment of AAD.

STUDY SUMMARY: Probiotics significantly reduce AAD

Hempel et al reviewed 82 studies and pooled data from 63 RCTs (N=11,811) to identify the relative risk (RR) of AAD among patients who received probiotics during antibiotic treatment compared with those who received no probiotics or were given a placebo.1 The studies encompassed a variety of antibiotics, taken alone or in combination, and several probiotics, including Lactobacillus, Bifidobacterium, Saccharomyces, and some combinations.

The outcome: The pooled RR for AAD in the probiotics groups was 0.58 (95% confidence interval, 0.50-0.68; P<.001), with a number needed to treat of 13. Although the authors reported that the overall quality of the included trials was poor, a sensitivity analysis of the higher quality studies yielded similar results.

Subgroup analyses by type of probiotic and duration of antibiotic treatment were also consistent with the overall pooled RR. In subgroup analysis by age, a similar decrease in AAD was found among the youngest patients (0-17 years) and those between the ages of 17 and 65 years. Among patients older than 65 years—for whom there were just 3 studies—a non-significant decrease in risk was found. Twenty-three of the studies assessed adverse outcomes, and none was found.

WHAT’S NEW: A reason to pair antibiotics and probiotics

This meta-analysis reached a similar conclusion as the 2006 meta-analysis: Probiotics appear to be effective in preventing and treating AAD in children and adults receiving a wide variety of antibiotics for a number of conditions. The results were also consistent with those of a new meta-analysis that looked specifically at one pathogen—and found a reduction of 66% in C difficile-associated diarrhea in patients taking probiotics with their antibiotics.10

 

 

 

CAVEATS: Limited data on the safety of probiotics exist

There was some heterogeneity among the studies in the meta-analysis by Hempel et al, and some of the studies were of poor quality. Because of this, the authors used subgroup and sensitivity analysis, which supported their initial conclusion.

Probiotics have generally been considered safe; however, there have been rare reports of sepsis and fungemia associated with probiotic use, especially in immunosuppressed patients.1 Fifty-nine of the included studies did not assess adverse events, which limited the ability of this meta-analysis to assess safety.1 Patients taking probiotics should be monitored for adverse effects.

CHALLENGES TO IMPLEMENTATION: Lack of guidance on dosing and duration

Since probiotics are considered food supplements, health insurance will not cover the cost (which will likely be more than $20 per month; www.walgreens.com). No single probiotic strain has high-quality evidence; however, most of the RCTs included in the meta-analysis used combinations of Lactobacillus species, which are usually found in over-the-counter antidiarrheal probiotic supplements. No standard dose exists, but dose ranges in RCTs are 107 to 1010 colony-forming units per capsule (taken one to 3 times daily);1 however, product labels have variable accuracy.11 The duration of treatment ranges from one to 3 weeks—or as long as the patient continues to take antibiotics.

Acknowledgement

The PURLs Surveillance System was developed with support from Grant Number UL1RR024999 from the National Center for Research Resources, a Clinical Translational Science Award to the University of Chicago. The content is solely the responsibility of the authors and does not necessarily represent the official views of the National Center for Research Resources or the National Institutes of Health.

Files
References

1. Hempel S, Newberry S, Maher A, et al. probiotics for the prevention and treatment of antibiotic-associated diarrhea. JAMA. 2012;307:1959-1969.

2. McFarland LV. Antibiotic-associated diarrhea: epidemiology, trends and treatment. Future Microbiol. 2008;3:563-578.

3. Pepin J, Valiquette L, Cossette B. Mortality attributable to nosocomial Clostridium difficile-associated disease during an epidemic caused by a hypervirulent strain in Quebec. CMAJ. 2005;173:1037-1042.

4. Perry A, Dellon E, Lund J, et al. Burden of gastrointestinal disease in the United States: 2012 Update. Gastroenterology. 2012;143:1179-1187.

5. Dubberke E, Wertheimer A. review of current literature on the economic burden of Clostridium difficile Infection. Infect Control Hosp Epidemiol. 2009;30:57-66.

6. McFarland L. Meta-analysis of probiotics for the prevention of antibiotic associated diarrhea and the treatment of Clostridium difficile disease. Am J Gastroenterol. 2006;101:812-822.

7. Cohen S, Gerding D, Johnson S, et al. Clinical practice guidelines for Clostridium difficile infection in adults: 2010 update by the Society for Healthcare Epidemiology of America and the Infectious Diseases Society of America. Infect Control Hosp Epidemiol. 2010;31:431-455.

8. Williams M, Ha C, Ciorba M. Probiotics as therapy in gastroenterology. J Clin Gastroenterol. 2010;44:631-636.

9. Floch M, Walker A, Madsne K, et al. Recommendations for probiotic use—2011 update. J Clin Gastroenterol. 2011;45(suppl):S168-S171.

10. Johnston BC, Ma SS, Goldenberg JZ, et al. Probiotics for the prevention of Clostridium difficile-associated diarrhea: a systematic review and meta-analysis. Ann Intern Med. 2012;157:878-888.

11. Hamilton-Miller J, Shah S. Deficiencies in microbiological quality and labeling of probiotic supplements. Int J Food Microbiol. 2002;72:175-176.

Article PDF
Author and Disclosure Information

Blake Rodgers, MD
University of North Carolina, Chapel Hill

Kate Kirley, MD
The University of Chicago

Anne Mounsey, MD
University of North Carolina, Chapel Hill

PURLs EDITOR
Bernard Ewigman, MD, MSPH
The University of Chicago

Issue
The Journal of Family Practice - 62(3)
Publications
Topics
Page Number
148-150
Legacy Keywords
Blake Rodgers; MD; Kate Kirley; MD; Anne Mounsey; MD; antibiotics; diarrhea; probiotics; antibiotic-associated diarrhea; AAD; Clostridium difficile; C difficile; PURLs; gastrointestinal; GI
Sections
Files
Files
Author and Disclosure Information

Blake Rodgers, MD
University of North Carolina, Chapel Hill

Kate Kirley, MD
The University of Chicago

Anne Mounsey, MD
University of North Carolina, Chapel Hill

PURLs EDITOR
Bernard Ewigman, MD, MSPH
The University of Chicago

Author and Disclosure Information

Blake Rodgers, MD
University of North Carolina, Chapel Hill

Kate Kirley, MD
The University of Chicago

Anne Mounsey, MD
University of North Carolina, Chapel Hill

PURLs EDITOR
Bernard Ewigman, MD, MSPH
The University of Chicago

Article PDF
Article PDF
PRACTICE CHANGER

Recommend that patients taking antibiotics also take probiotics, which have been found to be effective both for the prevention and treatment of antibiotic-associated diarrhea (AAD).1

STRENGTH OF RECOMMENDATION

A: Based on a systematic review and meta-analysis of randomized controlled trials.

Hempel S, Newberry S, Maher A, et al. Probiotics for the prevention and treatment of antibiotic-associated diarrhea. JAMA. 2012;307: 1959-1969.

 

ILLUSTRATIVE CASE

When you prescribe an antibiotic for a 45-year-old patient with Helicobacter pylori, he worries that the medication will cause diarrhea. Should you recommend that he take probiotics?

More than a third of patients taking antibiotics develop AAD,2 and in 17% of cases, AAD is fatal.3,4 Although the diarrhea may be the result of increased gastrointestinal (GI) motility in some cases, a disruption of the GI flora that normally acts as a barrier to infection and aids in the digestion of carbohydrates is a far more common cause.

Morbidity and mortality are high
AAD is associated with several pathogens, including Clostridium difficile, Clostridium perfringens, Klebsiella oxytoca, and Staphylococcus aureus,2 and varies widely in severity. Pseudomembranous colitis secondary to C difficile is the main cause of AAD-related mortality, which more than doubled from 2002 to 2009.3,4 C difficile infections cost the US health care system up to $1.3 billion annually.5 With such high rates of morbidity and mortality and high health care costs associated with AAD, even a small reduction in the number of cases would have a big impact.

Probiotics replenish the natural GI flora with nonpathogenic organisms. A 2006 meta-analysis of 31 randomized controlled trials (RCTs) assessing the efficacy of probiotics for both the prevention of AAD and treatment of C difficile found a pooled relative risk of 0.43 for AAD in the patients taking probiotics.6 However, many of the studies included in that meta-analysis were small. As a result, in 2010, the Society for Healthcare Epidemiology of America (SHEA) and the Infectious Diseases Society of America (IDSA) recommended against the use of probiotics for the prevention of primary C difficile infection, citing a lack of high-quality evidence.7

Nonetheless, that same year, 98% of gastroenterologists surveyed expressed a belief that probiotics had a role in the treatment of GI illness.8 And in 2011, the 3rd Yale Working Group on Probiotic Use published recommendations for probiotic use based on expert opinion.9 The meta-analysis detailed in this PURL, which included more than 30 trials published since the 2006 meta-analysis, addressed the efficacy of probiotics for prevention and treatment of AAD.

STUDY SUMMARY: Probiotics significantly reduce AAD

Hempel et al reviewed 82 studies and pooled data from 63 RCTs (N=11,811) to identify the relative risk (RR) of AAD among patients who received probiotics during antibiotic treatment compared with those who received no probiotics or were given a placebo.1 The studies encompassed a variety of antibiotics, taken alone or in combination, and several probiotics, including Lactobacillus, Bifidobacterium, Saccharomyces, and some combinations.

The outcome: The pooled RR for AAD in the probiotics groups was 0.58 (95% confidence interval, 0.50-0.68; P<.001), with a number needed to treat of 13. Although the authors reported that the overall quality of the included trials was poor, a sensitivity analysis of the higher quality studies yielded similar results.

Subgroup analyses by type of probiotic and duration of antibiotic treatment were also consistent with the overall pooled RR. In subgroup analysis by age, a similar decrease in AAD was found among the youngest patients (0-17 years) and those between the ages of 17 and 65 years. Among patients older than 65 years—for whom there were just 3 studies—a non-significant decrease in risk was found. Twenty-three of the studies assessed adverse outcomes, and none was found.

WHAT’S NEW: A reason to pair antibiotics and probiotics

This meta-analysis reached a similar conclusion as the 2006 meta-analysis: Probiotics appear to be effective in preventing and treating AAD in children and adults receiving a wide variety of antibiotics for a number of conditions. The results were also consistent with those of a new meta-analysis that looked specifically at one pathogen—and found a reduction of 66% in C difficile-associated diarrhea in patients taking probiotics with their antibiotics.10

 

 

 

CAVEATS: Limited data on the safety of probiotics exist

There was some heterogeneity among the studies in the meta-analysis by Hempel et al, and some of the studies were of poor quality. Because of this, the authors used subgroup and sensitivity analysis, which supported their initial conclusion.

Probiotics have generally been considered safe; however, there have been rare reports of sepsis and fungemia associated with probiotic use, especially in immunosuppressed patients.1 Fifty-nine of the included studies did not assess adverse events, which limited the ability of this meta-analysis to assess safety.1 Patients taking probiotics should be monitored for adverse effects.

CHALLENGES TO IMPLEMENTATION: Lack of guidance on dosing and duration

Since probiotics are considered food supplements, health insurance will not cover the cost (which will likely be more than $20 per month; www.walgreens.com). No single probiotic strain has high-quality evidence; however, most of the RCTs included in the meta-analysis used combinations of Lactobacillus species, which are usually found in over-the-counter antidiarrheal probiotic supplements. No standard dose exists, but dose ranges in RCTs are 107 to 1010 colony-forming units per capsule (taken one to 3 times daily);1 however, product labels have variable accuracy.11 The duration of treatment ranges from one to 3 weeks—or as long as the patient continues to take antibiotics.

Acknowledgement

The PURLs Surveillance System was developed with support from Grant Number UL1RR024999 from the National Center for Research Resources, a Clinical Translational Science Award to the University of Chicago. The content is solely the responsibility of the authors and does not necessarily represent the official views of the National Center for Research Resources or the National Institutes of Health.

PRACTICE CHANGER

Recommend that patients taking antibiotics also take probiotics, which have been found to be effective both for the prevention and treatment of antibiotic-associated diarrhea (AAD).1

STRENGTH OF RECOMMENDATION

A: Based on a systematic review and meta-analysis of randomized controlled trials.

Hempel S, Newberry S, Maher A, et al. Probiotics for the prevention and treatment of antibiotic-associated diarrhea. JAMA. 2012;307: 1959-1969.

 

ILLUSTRATIVE CASE

When you prescribe an antibiotic for a 45-year-old patient with Helicobacter pylori, he worries that the medication will cause diarrhea. Should you recommend that he take probiotics?

More than a third of patients taking antibiotics develop AAD,2 and in 17% of cases, AAD is fatal.3,4 Although the diarrhea may be the result of increased gastrointestinal (GI) motility in some cases, a disruption of the GI flora that normally acts as a barrier to infection and aids in the digestion of carbohydrates is a far more common cause.

Morbidity and mortality are high
AAD is associated with several pathogens, including Clostridium difficile, Clostridium perfringens, Klebsiella oxytoca, and Staphylococcus aureus,2 and varies widely in severity. Pseudomembranous colitis secondary to C difficile is the main cause of AAD-related mortality, which more than doubled from 2002 to 2009.3,4 C difficile infections cost the US health care system up to $1.3 billion annually.5 With such high rates of morbidity and mortality and high health care costs associated with AAD, even a small reduction in the number of cases would have a big impact.

Probiotics replenish the natural GI flora with nonpathogenic organisms. A 2006 meta-analysis of 31 randomized controlled trials (RCTs) assessing the efficacy of probiotics for both the prevention of AAD and treatment of C difficile found a pooled relative risk of 0.43 for AAD in the patients taking probiotics.6 However, many of the studies included in that meta-analysis were small. As a result, in 2010, the Society for Healthcare Epidemiology of America (SHEA) and the Infectious Diseases Society of America (IDSA) recommended against the use of probiotics for the prevention of primary C difficile infection, citing a lack of high-quality evidence.7

Nonetheless, that same year, 98% of gastroenterologists surveyed expressed a belief that probiotics had a role in the treatment of GI illness.8 And in 2011, the 3rd Yale Working Group on Probiotic Use published recommendations for probiotic use based on expert opinion.9 The meta-analysis detailed in this PURL, which included more than 30 trials published since the 2006 meta-analysis, addressed the efficacy of probiotics for prevention and treatment of AAD.

STUDY SUMMARY: Probiotics significantly reduce AAD

Hempel et al reviewed 82 studies and pooled data from 63 RCTs (N=11,811) to identify the relative risk (RR) of AAD among patients who received probiotics during antibiotic treatment compared with those who received no probiotics or were given a placebo.1 The studies encompassed a variety of antibiotics, taken alone or in combination, and several probiotics, including Lactobacillus, Bifidobacterium, Saccharomyces, and some combinations.

The outcome: The pooled RR for AAD in the probiotics groups was 0.58 (95% confidence interval, 0.50-0.68; P<.001), with a number needed to treat of 13. Although the authors reported that the overall quality of the included trials was poor, a sensitivity analysis of the higher quality studies yielded similar results.

Subgroup analyses by type of probiotic and duration of antibiotic treatment were also consistent with the overall pooled RR. In subgroup analysis by age, a similar decrease in AAD was found among the youngest patients (0-17 years) and those between the ages of 17 and 65 years. Among patients older than 65 years—for whom there were just 3 studies—a non-significant decrease in risk was found. Twenty-three of the studies assessed adverse outcomes, and none was found.

WHAT’S NEW: A reason to pair antibiotics and probiotics

This meta-analysis reached a similar conclusion as the 2006 meta-analysis: Probiotics appear to be effective in preventing and treating AAD in children and adults receiving a wide variety of antibiotics for a number of conditions. The results were also consistent with those of a new meta-analysis that looked specifically at one pathogen—and found a reduction of 66% in C difficile-associated diarrhea in patients taking probiotics with their antibiotics.10

 

 

 

CAVEATS: Limited data on the safety of probiotics exist

There was some heterogeneity among the studies in the meta-analysis by Hempel et al, and some of the studies were of poor quality. Because of this, the authors used subgroup and sensitivity analysis, which supported their initial conclusion.

Probiotics have generally been considered safe; however, there have been rare reports of sepsis and fungemia associated with probiotic use, especially in immunosuppressed patients.1 Fifty-nine of the included studies did not assess adverse events, which limited the ability of this meta-analysis to assess safety.1 Patients taking probiotics should be monitored for adverse effects.

CHALLENGES TO IMPLEMENTATION: Lack of guidance on dosing and duration

Since probiotics are considered food supplements, health insurance will not cover the cost (which will likely be more than $20 per month; www.walgreens.com). No single probiotic strain has high-quality evidence; however, most of the RCTs included in the meta-analysis used combinations of Lactobacillus species, which are usually found in over-the-counter antidiarrheal probiotic supplements. No standard dose exists, but dose ranges in RCTs are 107 to 1010 colony-forming units per capsule (taken one to 3 times daily);1 however, product labels have variable accuracy.11 The duration of treatment ranges from one to 3 weeks—or as long as the patient continues to take antibiotics.

Acknowledgement

The PURLs Surveillance System was developed with support from Grant Number UL1RR024999 from the National Center for Research Resources, a Clinical Translational Science Award to the University of Chicago. The content is solely the responsibility of the authors and does not necessarily represent the official views of the National Center for Research Resources or the National Institutes of Health.

References

1. Hempel S, Newberry S, Maher A, et al. probiotics for the prevention and treatment of antibiotic-associated diarrhea. JAMA. 2012;307:1959-1969.

2. McFarland LV. Antibiotic-associated diarrhea: epidemiology, trends and treatment. Future Microbiol. 2008;3:563-578.

3. Pepin J, Valiquette L, Cossette B. Mortality attributable to nosocomial Clostridium difficile-associated disease during an epidemic caused by a hypervirulent strain in Quebec. CMAJ. 2005;173:1037-1042.

4. Perry A, Dellon E, Lund J, et al. Burden of gastrointestinal disease in the United States: 2012 Update. Gastroenterology. 2012;143:1179-1187.

5. Dubberke E, Wertheimer A. review of current literature on the economic burden of Clostridium difficile Infection. Infect Control Hosp Epidemiol. 2009;30:57-66.

6. McFarland L. Meta-analysis of probiotics for the prevention of antibiotic associated diarrhea and the treatment of Clostridium difficile disease. Am J Gastroenterol. 2006;101:812-822.

7. Cohen S, Gerding D, Johnson S, et al. Clinical practice guidelines for Clostridium difficile infection in adults: 2010 update by the Society for Healthcare Epidemiology of America and the Infectious Diseases Society of America. Infect Control Hosp Epidemiol. 2010;31:431-455.

8. Williams M, Ha C, Ciorba M. Probiotics as therapy in gastroenterology. J Clin Gastroenterol. 2010;44:631-636.

9. Floch M, Walker A, Madsne K, et al. Recommendations for probiotic use—2011 update. J Clin Gastroenterol. 2011;45(suppl):S168-S171.

10. Johnston BC, Ma SS, Goldenberg JZ, et al. Probiotics for the prevention of Clostridium difficile-associated diarrhea: a systematic review and meta-analysis. Ann Intern Med. 2012;157:878-888.

11. Hamilton-Miller J, Shah S. Deficiencies in microbiological quality and labeling of probiotic supplements. Int J Food Microbiol. 2002;72:175-176.

References

1. Hempel S, Newberry S, Maher A, et al. probiotics for the prevention and treatment of antibiotic-associated diarrhea. JAMA. 2012;307:1959-1969.

2. McFarland LV. Antibiotic-associated diarrhea: epidemiology, trends and treatment. Future Microbiol. 2008;3:563-578.

3. Pepin J, Valiquette L, Cossette B. Mortality attributable to nosocomial Clostridium difficile-associated disease during an epidemic caused by a hypervirulent strain in Quebec. CMAJ. 2005;173:1037-1042.

4. Perry A, Dellon E, Lund J, et al. Burden of gastrointestinal disease in the United States: 2012 Update. Gastroenterology. 2012;143:1179-1187.

5. Dubberke E, Wertheimer A. review of current literature on the economic burden of Clostridium difficile Infection. Infect Control Hosp Epidemiol. 2009;30:57-66.

6. McFarland L. Meta-analysis of probiotics for the prevention of antibiotic associated diarrhea and the treatment of Clostridium difficile disease. Am J Gastroenterol. 2006;101:812-822.

7. Cohen S, Gerding D, Johnson S, et al. Clinical practice guidelines for Clostridium difficile infection in adults: 2010 update by the Society for Healthcare Epidemiology of America and the Infectious Diseases Society of America. Infect Control Hosp Epidemiol. 2010;31:431-455.

8. Williams M, Ha C, Ciorba M. Probiotics as therapy in gastroenterology. J Clin Gastroenterol. 2010;44:631-636.

9. Floch M, Walker A, Madsne K, et al. Recommendations for probiotic use—2011 update. J Clin Gastroenterol. 2011;45(suppl):S168-S171.

10. Johnston BC, Ma SS, Goldenberg JZ, et al. Probiotics for the prevention of Clostridium difficile-associated diarrhea: a systematic review and meta-analysis. Ann Intern Med. 2012;157:878-888.

11. Hamilton-Miller J, Shah S. Deficiencies in microbiological quality and labeling of probiotic supplements. Int J Food Microbiol. 2002;72:175-176.

Issue
The Journal of Family Practice - 62(3)
Issue
The Journal of Family Practice - 62(3)
Page Number
148-150
Page Number
148-150
Publications
Publications
Topics
Article Type
Display Headline
Prescribing an antibiotic? Pair it with probiotics
Display Headline
Prescribing an antibiotic? Pair it with probiotics
Legacy Keywords
Blake Rodgers; MD; Kate Kirley; MD; Anne Mounsey; MD; antibiotics; diarrhea; probiotics; antibiotic-associated diarrhea; AAD; Clostridium difficile; C difficile; PURLs; gastrointestinal; GI
Legacy Keywords
Blake Rodgers; MD; Kate Kirley; MD; Anne Mounsey; MD; antibiotics; diarrhea; probiotics; antibiotic-associated diarrhea; AAD; Clostridium difficile; C difficile; PURLs; gastrointestinal; GI
Sections
PURLs Copyright

Copyright © 2013 The Family Physicians Inquiries Network. All rights reserved.

Disallow All Ads
Alternative CME
Use ProPublica
Hide sidebar & use full width
render the right sidebar.
Conference Recap Checkbox
Not Conference Recap
Clinical Edge
Display the Slideshow in this Article
Article PDF Media
Media Files

A spoonful of honey helps a coughing child sleep

Article Type
Changed
Thu, 07/16/2020 - 14:37
Display Headline
A spoonful of honey helps a coughing child sleep
PRACTICE CHANGER

When a parent brings in a child (ages 1-5 years) with cough, runny nose, and other symptoms of a viral upper respiratory infection (URI), recommend that honey be given at bedtime.1

STRENGTH OF RECOMMENDATION

A: Based on a well-designed, randomized controlled trial (RCT)

Cohen HA, Rozen J, Kristal H, et al. Effect of honey on nocturnal cough and sleep quality: a double-blind, randomized, placebo-controlled study. Pediatrics. 2012;130:465-471.

 

ILLUSTRATIVE CASE

A mother brings in her 18-month-old son because he’s had a runny nose and low-grade fever for the past 4 days—and a cough that kept them both up last night. You diagnose a viral URI, and she requests a strong cough medicine so he (and she) can get a good night’s sleep. What can you recommend that is both safe and effective for a child of this age?

For primary care physicians, office visits for coughing kids with URIs are commonplace. In addition to the cost of such visits, Americans spend some $3.5 billion a year on over-the-counter (OTC) cough and cold remedies, and often give them to young children.

It’s not enough to tell parents what not to do
As physicians (and parents), we understand the desire to give a coughing child something to ease the symptoms. We also know that OTC cough and cold medications can lead to serious complications, and even death. Between 1983 and 2007, 118 pediatric deaths were attributed to the misuse of such preparations.2 And, in a 3-year span (2005-2008), the American Association of Poison Control Centers received 64,658 calls for exposures to cough and cold remedies in children younger than 2 years of age, 28 of which resulted in a major adverse reaction or death.3

The US Food and Drug Administration recommends against the use of OTC cough and cold medications in children younger than 2 years,4 and the American Academy of Pediatrics has issued strict warnings about the use of OTC cough and cold preparations in children younger than 6 years.5 But warning parents of the dangers of giving them to young children without offering an alternative doesn’t satisfy anyone’s needs, and many parents continue to use these medications.

What about honey?
A study published in 2007 evaluated buckwheat honey and found it to be superior to no treatment and equal to honey-flavored dextromethorphan in reducing cough severity and improving sleep for children and their parents.6 Honey is known to have both antioxidant and antimicrobial properties—a possible scientific explanation for its effect. Before recommending honey for kids with URIs, however, more evidence of its efficacy was needed.

STUDY SUMMARY: Honey reduces cough frequency and severity

Cohen et al sought to determine whether honey, administered before bedtime, would decrease coughing in children between the ages of one and 5 years—and improve sleep for both the children and their caregivers.1 They enrolled 300 children with a nocturnal cough of <7 days’ duration and a diagnosis of URI in a one-night study.

Children were excluded if they had any signs or symptoms of asthma, pneumonia, sinusitis, allergic rhinitis, or laryngotracheobronchitis, or if they had been given any cough remedy, including honey, the night before. Parents completed a 5-question survey, using a 7-point Likert scale to assess the child’s cough and both the child’s and parents’ sleep the previous night. Only children whose parents rated their child’s cough severity ≥3 in 2 of the 3 questions related to cough were included in the trial.

The study had a double-blind randomized design, with 4 treatment arms. Three groups received 10 g (about 1.5 tsp) of one of 3 types of honey: eucalyptus, citrus, or labiatae (derived from plants including sage, mint, and thyme); the fourth group received a placebo of silan date extract, which is similar to honey in color, texture, and taste.

 

 

 

Children in all 4 groups received the preparation 30 minutes before bedtime. Neither the parents nor the physicians or study coordinators knew which preparation the children received. The following day, research assistants telephoned the parent who had completed the initial survey and asked the same 5 questions. The primary outcome measure was the change in cough frequency from the night before to the night after treatment. Secondary measures included cough severity and the effect on sleep for both the child and the parent.

Of the 300 children initially enrolled, 270 (90%) completed the trial, with an even distribution among the groups. While there were improvements across all outcomes for both the treatment and placebo groups, the changes were statistically significant only in the treatment groups. There were no significant differences in efficacy noted among the 3 types of honey. Adverse effects of stomachache, nausea, or vomiting were noted by 4 parents in the treatment groups and one in the placebo group, a difference that was not statistically significant.

WHAT’S NEW?: We have more evidence of honey’s efficacy

For children older than one year with a viral URI, we can now recommend 1.5 tsp honey to be given prior to bedtime as a cough remedy. This may reduce the use of potentially harmful and often ineffective OTC cough and cold remedies.

CAVEATS: Honey is unsafe for the youngest children

An obvious limitation of this study was its brevity. Although one night of improved cough and sleep is important, a study that showed honey’s sustained benefit as a cough suppressant would be more convincing. What’s more, there are safety concerns that are age-related.

Honey is considered unsafe for children younger than one year because of the risk of botulism. And honey has the potential to increase dental caries if it is given nightly for a prolonged period of time.

We do not know whether all varieties of honey will have the same benefit, and the source of store-bought honey is not always identified. The authors of this study received funding from the Honey Board of Israel.

CHALLENGES TO IMPLEMENTATION: Parents may be reluctant to abandon OTCs

Changing the behavior of parents and other caregivers who are accustomed to treating children with OTC cough and cold remedies is likely to be an uphill battle. Because honey is readily available, however—often as close as the pantry—and perceived to be safe and nutritious, a recommendation from a trusted physician could go a long way toward its implementation.

Acknowledgement

The PURLs Surveillance System was supported in part by Grant Number UL1RR024999 from the National Center for Research Resources, a Clinical Translational Science Award to the University of Chicago. The content is solely the responsibility of the authors and does not necessarily represent the official views of the National Center for Research Resources or the National Institutes of Health.

Files
References

1. Cohen HA, Rozen J, Kristal H, et al. Effect of honey on nocturnal cough and sleep quality: a double-blind, randomized, placebo-controlled study. Pediatrics. 2012;130:465-471.

2. Dart RC, Paul IM, Bond GR, et al. Pediatric fatalities associated with over the counter (nonprescription) cough and cold medications. Ann Emerg Med. 2009;53:411-417.

3. Srinivasan A, Budnitz D, Shehab N, et al. Infant deaths associated with cough and cold medications—two states, 2005. JAMA. 2007;297:800-801.

4. US Food and Drug Administration. Public Health Advisory: FDA recommends that over-the-counter (OTC) cough and cold products not be used for Infants and children under 2 years of age. Available at: http://www.fda.gov/Drugs/DrugSafety/PostmarketDrugSafetyInformationforPatientsandProviders/DrugSafetyInformationforHeathcareProfessionals/PublicHealthAdvisories/ucm051137.htm. Accessed February 14, 2013.

5. American Academy of Pediatrics. Withdrawal of cold medicines: addressing parent concerns. Available at: http://www.aap.org/en-us/professional-resources/practice-support/Pages/Withdrawal-of-Cold-Medicines-Addressing-Parent-Concerns.aspx. Accessed February 14, 2013.

6. Paul IM, Beiler J, McMonagle A, et al. Effect of honey, dextromethorphan, and no treatment on nocturnal cough and sleep quality for coughing children and their parents. Arch Pediatr Adolesc Med. 2007;161:1140-1146.

Article PDF
Author and Disclosure Information

Evan Ashkin, MD
Department of Family Medicine, University of North Carolina at Chapel Hill

Anne Mounsey, MD
Department of Family Medicine, University of North Carolina at Chapel Hill

PURLs EDITOR
James J. Stevermer, MD, MSPH
Department of Family Medicine, University of Missouri-Columbia

Issue
The Journal of Family Practice - 62(3)
Publications
Topics
Page Number
145-147
Legacy Keywords
Evan Ashkin; MD; Anne Mounsey; MD; honey; cough and cold; viral URI; upper respiratory infection; cough medicine; OTC; improving sleep; 5-question survey; asthma; cough severity; PURLs; pediatrics
Sections
Files
Files
Author and Disclosure Information

Evan Ashkin, MD
Department of Family Medicine, University of North Carolina at Chapel Hill

Anne Mounsey, MD
Department of Family Medicine, University of North Carolina at Chapel Hill

PURLs EDITOR
James J. Stevermer, MD, MSPH
Department of Family Medicine, University of Missouri-Columbia

Author and Disclosure Information

Evan Ashkin, MD
Department of Family Medicine, University of North Carolina at Chapel Hill

Anne Mounsey, MD
Department of Family Medicine, University of North Carolina at Chapel Hill

PURLs EDITOR
James J. Stevermer, MD, MSPH
Department of Family Medicine, University of Missouri-Columbia

Article PDF
Article PDF
PRACTICE CHANGER

When a parent brings in a child (ages 1-5 years) with cough, runny nose, and other symptoms of a viral upper respiratory infection (URI), recommend that honey be given at bedtime.1

STRENGTH OF RECOMMENDATION

A: Based on a well-designed, randomized controlled trial (RCT)

Cohen HA, Rozen J, Kristal H, et al. Effect of honey on nocturnal cough and sleep quality: a double-blind, randomized, placebo-controlled study. Pediatrics. 2012;130:465-471.

 

ILLUSTRATIVE CASE

A mother brings in her 18-month-old son because he’s had a runny nose and low-grade fever for the past 4 days—and a cough that kept them both up last night. You diagnose a viral URI, and she requests a strong cough medicine so he (and she) can get a good night’s sleep. What can you recommend that is both safe and effective for a child of this age?

For primary care physicians, office visits for coughing kids with URIs are commonplace. In addition to the cost of such visits, Americans spend some $3.5 billion a year on over-the-counter (OTC) cough and cold remedies, and often give them to young children.

It’s not enough to tell parents what not to do
As physicians (and parents), we understand the desire to give a coughing child something to ease the symptoms. We also know that OTC cough and cold medications can lead to serious complications, and even death. Between 1983 and 2007, 118 pediatric deaths were attributed to the misuse of such preparations.2 And, in a 3-year span (2005-2008), the American Association of Poison Control Centers received 64,658 calls for exposures to cough and cold remedies in children younger than 2 years of age, 28 of which resulted in a major adverse reaction or death.3

The US Food and Drug Administration recommends against the use of OTC cough and cold medications in children younger than 2 years,4 and the American Academy of Pediatrics has issued strict warnings about the use of OTC cough and cold preparations in children younger than 6 years.5 But warning parents of the dangers of giving them to young children without offering an alternative doesn’t satisfy anyone’s needs, and many parents continue to use these medications.

What about honey?
A study published in 2007 evaluated buckwheat honey and found it to be superior to no treatment and equal to honey-flavored dextromethorphan in reducing cough severity and improving sleep for children and their parents.6 Honey is known to have both antioxidant and antimicrobial properties—a possible scientific explanation for its effect. Before recommending honey for kids with URIs, however, more evidence of its efficacy was needed.

STUDY SUMMARY: Honey reduces cough frequency and severity

Cohen et al sought to determine whether honey, administered before bedtime, would decrease coughing in children between the ages of one and 5 years—and improve sleep for both the children and their caregivers.1 They enrolled 300 children with a nocturnal cough of <7 days’ duration and a diagnosis of URI in a one-night study.

Children were excluded if they had any signs or symptoms of asthma, pneumonia, sinusitis, allergic rhinitis, or laryngotracheobronchitis, or if they had been given any cough remedy, including honey, the night before. Parents completed a 5-question survey, using a 7-point Likert scale to assess the child’s cough and both the child’s and parents’ sleep the previous night. Only children whose parents rated their child’s cough severity ≥3 in 2 of the 3 questions related to cough were included in the trial.

The study had a double-blind randomized design, with 4 treatment arms. Three groups received 10 g (about 1.5 tsp) of one of 3 types of honey: eucalyptus, citrus, or labiatae (derived from plants including sage, mint, and thyme); the fourth group received a placebo of silan date extract, which is similar to honey in color, texture, and taste.

 

 

 

Children in all 4 groups received the preparation 30 minutes before bedtime. Neither the parents nor the physicians or study coordinators knew which preparation the children received. The following day, research assistants telephoned the parent who had completed the initial survey and asked the same 5 questions. The primary outcome measure was the change in cough frequency from the night before to the night after treatment. Secondary measures included cough severity and the effect on sleep for both the child and the parent.

Of the 300 children initially enrolled, 270 (90%) completed the trial, with an even distribution among the groups. While there were improvements across all outcomes for both the treatment and placebo groups, the changes were statistically significant only in the treatment groups. There were no significant differences in efficacy noted among the 3 types of honey. Adverse effects of stomachache, nausea, or vomiting were noted by 4 parents in the treatment groups and one in the placebo group, a difference that was not statistically significant.

WHAT’S NEW?: We have more evidence of honey’s efficacy

For children older than one year with a viral URI, we can now recommend 1.5 tsp honey to be given prior to bedtime as a cough remedy. This may reduce the use of potentially harmful and often ineffective OTC cough and cold remedies.

CAVEATS: Honey is unsafe for the youngest children

An obvious limitation of this study was its brevity. Although one night of improved cough and sleep is important, a study that showed honey’s sustained benefit as a cough suppressant would be more convincing. What’s more, there are safety concerns that are age-related.

Honey is considered unsafe for children younger than one year because of the risk of botulism. And honey has the potential to increase dental caries if it is given nightly for a prolonged period of time.

We do not know whether all varieties of honey will have the same benefit, and the source of store-bought honey is not always identified. The authors of this study received funding from the Honey Board of Israel.

CHALLENGES TO IMPLEMENTATION: Parents may be reluctant to abandon OTCs

Changing the behavior of parents and other caregivers who are accustomed to treating children with OTC cough and cold remedies is likely to be an uphill battle. Because honey is readily available, however—often as close as the pantry—and perceived to be safe and nutritious, a recommendation from a trusted physician could go a long way toward its implementation.

Acknowledgement

The PURLs Surveillance System was supported in part by Grant Number UL1RR024999 from the National Center for Research Resources, a Clinical Translational Science Award to the University of Chicago. The content is solely the responsibility of the authors and does not necessarily represent the official views of the National Center for Research Resources or the National Institutes of Health.

PRACTICE CHANGER

When a parent brings in a child (ages 1-5 years) with cough, runny nose, and other symptoms of a viral upper respiratory infection (URI), recommend that honey be given at bedtime.1

STRENGTH OF RECOMMENDATION

A: Based on a well-designed, randomized controlled trial (RCT)

Cohen HA, Rozen J, Kristal H, et al. Effect of honey on nocturnal cough and sleep quality: a double-blind, randomized, placebo-controlled study. Pediatrics. 2012;130:465-471.

 

ILLUSTRATIVE CASE

A mother brings in her 18-month-old son because he’s had a runny nose and low-grade fever for the past 4 days—and a cough that kept them both up last night. You diagnose a viral URI, and she requests a strong cough medicine so he (and she) can get a good night’s sleep. What can you recommend that is both safe and effective for a child of this age?

For primary care physicians, office visits for coughing kids with URIs are commonplace. In addition to the cost of such visits, Americans spend some $3.5 billion a year on over-the-counter (OTC) cough and cold remedies, and often give them to young children.

It’s not enough to tell parents what not to do
As physicians (and parents), we understand the desire to give a coughing child something to ease the symptoms. We also know that OTC cough and cold medications can lead to serious complications, and even death. Between 1983 and 2007, 118 pediatric deaths were attributed to the misuse of such preparations.2 And, in a 3-year span (2005-2008), the American Association of Poison Control Centers received 64,658 calls for exposures to cough and cold remedies in children younger than 2 years of age, 28 of which resulted in a major adverse reaction or death.3

The US Food and Drug Administration recommends against the use of OTC cough and cold medications in children younger than 2 years,4 and the American Academy of Pediatrics has issued strict warnings about the use of OTC cough and cold preparations in children younger than 6 years.5 But warning parents of the dangers of giving them to young children without offering an alternative doesn’t satisfy anyone’s needs, and many parents continue to use these medications.

What about honey?
A study published in 2007 evaluated buckwheat honey and found it to be superior to no treatment and equal to honey-flavored dextromethorphan in reducing cough severity and improving sleep for children and their parents.6 Honey is known to have both antioxidant and antimicrobial properties—a possible scientific explanation for its effect. Before recommending honey for kids with URIs, however, more evidence of its efficacy was needed.

STUDY SUMMARY: Honey reduces cough frequency and severity

Cohen et al sought to determine whether honey, administered before bedtime, would decrease coughing in children between the ages of one and 5 years—and improve sleep for both the children and their caregivers.1 They enrolled 300 children with a nocturnal cough of <7 days’ duration and a diagnosis of URI in a one-night study.

Children were excluded if they had any signs or symptoms of asthma, pneumonia, sinusitis, allergic rhinitis, or laryngotracheobronchitis, or if they had been given any cough remedy, including honey, the night before. Parents completed a 5-question survey, using a 7-point Likert scale to assess the child’s cough and both the child’s and parents’ sleep the previous night. Only children whose parents rated their child’s cough severity ≥3 in 2 of the 3 questions related to cough were included in the trial.

The study had a double-blind randomized design, with 4 treatment arms. Three groups received 10 g (about 1.5 tsp) of one of 3 types of honey: eucalyptus, citrus, or labiatae (derived from plants including sage, mint, and thyme); the fourth group received a placebo of silan date extract, which is similar to honey in color, texture, and taste.

 

 

 

Children in all 4 groups received the preparation 30 minutes before bedtime. Neither the parents nor the physicians or study coordinators knew which preparation the children received. The following day, research assistants telephoned the parent who had completed the initial survey and asked the same 5 questions. The primary outcome measure was the change in cough frequency from the night before to the night after treatment. Secondary measures included cough severity and the effect on sleep for both the child and the parent.

Of the 300 children initially enrolled, 270 (90%) completed the trial, with an even distribution among the groups. While there were improvements across all outcomes for both the treatment and placebo groups, the changes were statistically significant only in the treatment groups. There were no significant differences in efficacy noted among the 3 types of honey. Adverse effects of stomachache, nausea, or vomiting were noted by 4 parents in the treatment groups and one in the placebo group, a difference that was not statistically significant.

WHAT’S NEW?: We have more evidence of honey’s efficacy

For children older than one year with a viral URI, we can now recommend 1.5 tsp honey to be given prior to bedtime as a cough remedy. This may reduce the use of potentially harmful and often ineffective OTC cough and cold remedies.

CAVEATS: Honey is unsafe for the youngest children

An obvious limitation of this study was its brevity. Although one night of improved cough and sleep is important, a study that showed honey’s sustained benefit as a cough suppressant would be more convincing. What’s more, there are safety concerns that are age-related.

Honey is considered unsafe for children younger than one year because of the risk of botulism. And honey has the potential to increase dental caries if it is given nightly for a prolonged period of time.

We do not know whether all varieties of honey will have the same benefit, and the source of store-bought honey is not always identified. The authors of this study received funding from the Honey Board of Israel.

CHALLENGES TO IMPLEMENTATION: Parents may be reluctant to abandon OTCs

Changing the behavior of parents and other caregivers who are accustomed to treating children with OTC cough and cold remedies is likely to be an uphill battle. Because honey is readily available, however—often as close as the pantry—and perceived to be safe and nutritious, a recommendation from a trusted physician could go a long way toward its implementation.

Acknowledgement

The PURLs Surveillance System was supported in part by Grant Number UL1RR024999 from the National Center for Research Resources, a Clinical Translational Science Award to the University of Chicago. The content is solely the responsibility of the authors and does not necessarily represent the official views of the National Center for Research Resources or the National Institutes of Health.

References

1. Cohen HA, Rozen J, Kristal H, et al. Effect of honey on nocturnal cough and sleep quality: a double-blind, randomized, placebo-controlled study. Pediatrics. 2012;130:465-471.

2. Dart RC, Paul IM, Bond GR, et al. Pediatric fatalities associated with over the counter (nonprescription) cough and cold medications. Ann Emerg Med. 2009;53:411-417.

3. Srinivasan A, Budnitz D, Shehab N, et al. Infant deaths associated with cough and cold medications—two states, 2005. JAMA. 2007;297:800-801.

4. US Food and Drug Administration. Public Health Advisory: FDA recommends that over-the-counter (OTC) cough and cold products not be used for Infants and children under 2 years of age. Available at: http://www.fda.gov/Drugs/DrugSafety/PostmarketDrugSafetyInformationforPatientsandProviders/DrugSafetyInformationforHeathcareProfessionals/PublicHealthAdvisories/ucm051137.htm. Accessed February 14, 2013.

5. American Academy of Pediatrics. Withdrawal of cold medicines: addressing parent concerns. Available at: http://www.aap.org/en-us/professional-resources/practice-support/Pages/Withdrawal-of-Cold-Medicines-Addressing-Parent-Concerns.aspx. Accessed February 14, 2013.

6. Paul IM, Beiler J, McMonagle A, et al. Effect of honey, dextromethorphan, and no treatment on nocturnal cough and sleep quality for coughing children and their parents. Arch Pediatr Adolesc Med. 2007;161:1140-1146.

References

1. Cohen HA, Rozen J, Kristal H, et al. Effect of honey on nocturnal cough and sleep quality: a double-blind, randomized, placebo-controlled study. Pediatrics. 2012;130:465-471.

2. Dart RC, Paul IM, Bond GR, et al. Pediatric fatalities associated with over the counter (nonprescription) cough and cold medications. Ann Emerg Med. 2009;53:411-417.

3. Srinivasan A, Budnitz D, Shehab N, et al. Infant deaths associated with cough and cold medications—two states, 2005. JAMA. 2007;297:800-801.

4. US Food and Drug Administration. Public Health Advisory: FDA recommends that over-the-counter (OTC) cough and cold products not be used for Infants and children under 2 years of age. Available at: http://www.fda.gov/Drugs/DrugSafety/PostmarketDrugSafetyInformationforPatientsandProviders/DrugSafetyInformationforHeathcareProfessionals/PublicHealthAdvisories/ucm051137.htm. Accessed February 14, 2013.

5. American Academy of Pediatrics. Withdrawal of cold medicines: addressing parent concerns. Available at: http://www.aap.org/en-us/professional-resources/practice-support/Pages/Withdrawal-of-Cold-Medicines-Addressing-Parent-Concerns.aspx. Accessed February 14, 2013.

6. Paul IM, Beiler J, McMonagle A, et al. Effect of honey, dextromethorphan, and no treatment on nocturnal cough and sleep quality for coughing children and their parents. Arch Pediatr Adolesc Med. 2007;161:1140-1146.

Issue
The Journal of Family Practice - 62(3)
Issue
The Journal of Family Practice - 62(3)
Page Number
145-147
Page Number
145-147
Publications
Publications
Topics
Article Type
Display Headline
A spoonful of honey helps a coughing child sleep
Display Headline
A spoonful of honey helps a coughing child sleep
Legacy Keywords
Evan Ashkin; MD; Anne Mounsey; MD; honey; cough and cold; viral URI; upper respiratory infection; cough medicine; OTC; improving sleep; 5-question survey; asthma; cough severity; PURLs; pediatrics
Legacy Keywords
Evan Ashkin; MD; Anne Mounsey; MD; honey; cough and cold; viral URI; upper respiratory infection; cough medicine; OTC; improving sleep; 5-question survey; asthma; cough severity; PURLs; pediatrics
Sections
PURLs Copyright

Copyright © 2013 The Family Physicians Inquiries Network. All rights reserved.

Disallow All Ads
Alternative CME
Use ProPublica
Hide sidebar & use full width
render the right sidebar.
Conference Recap Checkbox
Not Conference Recap
Clinical Edge
Display the Slideshow in this Article
Article PDF Media
Media Files

An obesity remedy for diabetes

Article Type
Changed
Tue, 05/03/2022 - 15:55
Display Headline
An obesity remedy for diabetes

PRACTICE CHANGER

Consider bariatric surgery for patients with diabetes who are obese; surgery is associated with higher remission rates than medical therapy, regardless of the amount of weight lost.1

STRENGTH OF RECOMMENDATION

B: Based on a single nonblinded randomized controlled trial (RCT).

Mingrone G, Panunzi S, De Gaetano A, et al. Bariatric surgery versus conventional medical therapy for type 2 diabetes. N Engl J Med. 2012;366:1577-1585.

ILLUSTRATIVE CASE

A 43-year-old woman with a body mass index (BMI) of 38 kg/m2 and a 5-year history of diabetes has a glycated hemoglobin (HbA1c) of 8.5% despite the use of oral hypoglycemic agents. Should you talk to her about gastric bypass surgery to treat her diabetes?

Diet and exercise are the first steps in addressing diabetes, but these interventions are often unsuccessful. The International Diabetes Federation (IDF) recommends consideration of bariatric surgery for patients who have a BMI >35 kg/m2 and diabetes that lifestyle modification and pharmacotherapy have failed to control.2

Surgery for diabetes: Is there ample evidence?
Until recently, the IDF’s recommendation was based on observational data and a single RCT that found increased resolution of diabetes following various bariatric procedures.3-5 In the study detailed in this PURL, Mingrone et al took another look.

STUDY SUMMARY: Surgery led to higher remission rates

This single-center, nonblinded RCT compared 2 malabsorptive procedures— Roux-en-Y gastric bypass and biliopancreatic diversion, a more complicated procedure not commonly performed—with medical therapy.1 The primary outcome was the rate of diabetes remission at 2 years, defined as a fasting glucose level <100 mg/dL and an HbA1c <6.5%. Changes in BMI and cholesterol levels were among the secondary endpoints.

To be eligible, patients had to be between the ages of 30 and 60 years and have a BMI ≥35 kg/m2, a history of type 2 diabetes ≥5 years, and an HbA1c ≥7.0%. Exclusion criteria included a history of type 1 diabetes, diabetes caused by an underlying disease or steroid treatment, previous bariatric surgery, pregnancy, diabetic complications, other severe medical conditions, and acute hospitalization. Both the gastric bypass and biliopancreatic diversion procedures were performed by independent surgical teams.

Participants (N=60) were evaluated at baseline and at 1, 3, 6, 9, 12, and 24 months after the intervention by a team that included a dietician, a nurse, and a physician. All received a diet plan with daily exercise designed by their team. Those in the medical therapy group had their medications titrated to reach a goal HbA1c <7%. Pharmacotherapy was stopped based on normalization of blood sugars or HbA1c <6.5%.

Within 15 days postsurgery, patients in both surgical arms had their diabetes medications stopped based on their blood glucose levels.

At 2 years, 75% of the patients in the gastric bypass arm and 95% of the patients in the biliopancreatic diversion arm (number needed to treat=1.3 and 1, respectively) were considered to be in diabetes remission, defined as a fasting blood sugar of <100 mg/dL and an HbA1c <6.5% after one year without pharmacotherapy. (Notably, this differs from that of the American Diabetes Association, which requires an HbA1c <6.0% for classification as complete remission.) None of the patients in the medical therapy arm was in remission at the 2-year mark.

On average, blood sugars normalized for gastric bypass patients by 10±2 months, vs 4±1 months for biliopancreatic diversion patients (P=.01). The average HbA1c at the end of 2 years was significantly different among all 3 groups (6.35%±1.42 for those undergoing gastric bypass, 4.95%±0.49 for the biliopancreatic diversion group, and 7.69%±0.57 for the medical therapy group), as was the change in HbA1c from baseline (TABLE). Changes in BMI and the number of patients who achieved normalization of total cholesterol were similar for both surgical groups. Interestingly, neither baseline BMI nor amount of weight lost or pre-enrollment duration of diabetes were predictors of diabetes remission or normalization of fasting glucose levels.

TABLE
Surgery vs medical therapy for diabetes: Gastric bypass and biliopancreatic diversion are more effective

 Gastric bypass (n=20)Biliopancreatic diversion (n=20)Medical therapy (n=20)
HbA1c at 2 years (%)6.35±1.42*
(n=19)
4.95±0.49
(n=19)
7.69±0.57
(n=18)
HbA1c change from baseline* (%)–25.18±20.89–43.01±9.64–8.39±9.93
BMI change from baseline* (%)–33.31±7.88–33.82±10.17*–4.73±6.37
Total cholesterol normalization (%)100*100*27.3
BMI, body mass index.
*P<0.01 for post hoc analysis comparing surgical arm to medical therapy.
Normalization of cholesterol was defined as a total cholesterol <201 mg/dL and HDL >40 mg/dL in men and >50 mg/dL in women (personal communication from author).

There were no deaths associated with this study. There were 2 adverse events requiring reoperation: an incisional hernia in a patient in the biliopancreatic diversion group and an intestinal obstruction in a patient in the gastric bypass group. Six patients in the biliopancreatic diversion arm developed metabolic abnormalities, including iron deficiency anemia, hypoalbuminemia, osteopenia, and osteoporosis. In the gastric bypass arm, 2 patients developed iron deficiency anemia.

 

 

WHAT’S NEW?: Evidence of efficacy has grown

This is the first RCT to evaluate biliopancreatic diversion and only the second to evaluate gastric bypass as strategies for controlling diabetes. Similar findings were demonstrated at 12 months in an RCT of 150 obese patients with diabetes in which intensive medical therapy was compared with either gastric bypass or sleeve gastrectomy,6 published concurrently with the Mingrone study. Like the Mingrone study, this study found that for select patients with diabetes, surgery may lead to better outcomes than medical management alone.

CAVEATS: Long-term effect is still uncertain

The long-term efficacy of surgery as a way to manage diabetes remains uncertain. Patients in this study were followed for just 2 years and the outcomes were metabolic measures rather than morbidity and mortality. A recent prospective observational study following patients for 6 years after gastric bypass found that the rate of remission for diabetes was 75% (95% confidence interval (CI), 63%-87%) at 2 years but dropped to 62% (95% CI, 49%-75%) at 6 years7

A larger study (N=4047) of longer duration—the Swedish Obese Subjects (SOS) cohort study —found a considerably larger drop: The diabetes remission rate for those who had surgery went from 72% at 2 years to 36% at 10-year follow-up, but that was still higher than the 10-year remission rate (13%) for the matched controls.4 It is still not clear exactly how long diabetic remission lasts after bariatric surgery or what effect a 10-year respite from the disease will have on the long-term morbidity and mortality of patients with diabetes.

Surgical risks. In small studies such as the one by Mingrone et al,1 it can be difficult to see the full extent of surgical complications. The much larger SOS study found low mortality rates (0.25%). But 13% of those who underwent bariatric surgery had postoperative complications (number needed to harm = 8), with 2.2% of patients requiring reoperation.4 Additionally, women who become pregnant after bariatric surgery are at increased risk for internal hernias or bowel obstruction during pregnancy.8

Furthermore, malabsorptive-type surgeries are known to cause nutritional deficiencies, leading to disorders including anemia and osteoporosis.6 Importantly, while women of childbearing-age who undergo bariatric surgery decrease their risk of developing gestational hypertension and gestational diabetes, they are more likely to have nutritional deficiencies during pregnancy and to have children with these deficiencies.8

CHALLENGES TO IMPLEMENTATION: The ideal candidate remains unclear

It is still not clear from this study which patients should be referred for bariatric surgery. Historically, BMI has been used as the main indication for bariatric surgery, but this and other, studies have found that remission of diabetes is independent of BMI and the amount of weight lost.9 A predictive 10-point Diabetes Surgery Score has recently been developed: It uses age, BMI, duration of diabetes, and C-peptide levels to predict the likelihood of diabetes remission after surgery.10 This scoring system has yet to be validated in non-Asian patients, and a threshold for recommending surgery has been not established. However, this tool indicates that younger patients with a shorter duration of diabetes (which was not a factor in the outcome of the Mingrone study) and no baseline use of insulin are most likely to benefit from surgery. Thus, these patients may be the ones we need to consider referring first.

Cost of surgery. Several studies have shown that bariatric surgery is cost-effective for the treatment of diabetes, and saves money after approximately 5 years.11,12 However, patients with diabetes and obesity may be uninsured or underinsured, and have high out-of-pocket costs. One challenge will be to ensure that surgery is a viable option for patients with financial constraints.

Acknowledgement

The PURLs Surveillance System was supported in part by Grant Number UL1RR024999 from the National Center for Research Resources, a Clinical Translational Science Award to the University of Chicago. The content is solely the responsibility of the authors and does not necessarily represent the official views of the National Center for Research Resources or the National Institutes of Health.

References

1. Mingrone G, Panunzi S, De Gaetano A, et al. Bariatric surgery versus conventional medical therapy for type 2 diabetes. N Engl J Med. 2012;366:1577-1585.

2. Dixon JB, Zimmet P, Alberti KG, et al. Bariatric surgery: an IDF statement for obese type 2 diabetes. Diabet Med. 2011;28:628-642.

3. Buchwald H, Estok R, Fahrbach K, et al. Weight and type 2 diabetes after bariatric surgery: systematic review and meta-analysis. Am J Med. 2009;122:248-256.

4. Sjöström L, Lindroos AK, Peltonen M, et al. Lifestyle, diabetes, and cardiovascular risk factors 10 years after bariatric surgery. N Engl J Med. 2004;351:2683-2693.

5. Dixon JB, O’Brien PE, Playfair J, et al. Adjustable gastric banding and conventional therapy for type 2 diabetes: a randomized controlled trial. JAMA. 2008;299:316-323.

6. Schauer PR, Kashyap SR, Wolski K, et al. Bariatric surgery versus intensive medical therapy in obese patients with diabetes. N Engl J Med. 2012;366:1568-1576.

7. Adams TD, Davidson LE, Litwin SE, et al. Health benefits of gastric bypass surgery after 6 years. JAMA. 2012;308:1122-1131.

8. Dalfrà MG, Busetto L, Chilelli NC, et al. Pregnancy and foetal outcome after bariatric surgery: a review of recent studies. J Matern Fetal Neonatal Med. 2012;25:1537-1543.

9. Livingston EH Pitfalls in using BMI as a selection criterion for bariatric surgery. Curr Opin Endocrinol Diabetes Obes. 2012;19:347-351.

10. Lee W-J, Hur K, Lakadawala M, et al. Predicting success of metabolic surgery: age, body mass index, C-peptide, and duration score. Surg Obes Relat Dis. 2012; [Epub ahead of print].

11. Terranova L, Busetto L, Vestri A, et al. Bariatric surgery: cost-effectiveness and budget impact. Obes Surg. 2012;22:646-653.

12. Hoerger TJ, Zhang P, Segel JE, et al. Cost-effectiveness of bariatric surgery for severely obese adults with diabetes. Diabetes Care. 2010;33:1933-1939.

Article PDF
Author and Disclosure Information

Dana Neutze, MD, PhD
Department of Family Medicine, University of North Carolina at Chapel Hill

Mari Egan, MD
Department of Family Medicine, University of Chicago

Anne Mounsey, MD
Department of Family Medicine, University of North Carolina at Chapel Hill

PURLs EDITOR James J. Stevermer, MD, MSPH
Department of Family Medicine, University of Missouri-Columbia

Issue
The Journal of Family Practice - 62(1)
Publications
Topics
Page Number
30-32
Legacy Keywords
Dana Neutze; MD; PhD; Mari Egan; MD; Anne Mounsey; MD; bariatric surgery; malabsorptive processes; gastric bypass; Roux-en-Y; biliopancreatic diversion; remission; PURLs; obesity; diabetes
Sections
Author and Disclosure Information

Dana Neutze, MD, PhD
Department of Family Medicine, University of North Carolina at Chapel Hill

Mari Egan, MD
Department of Family Medicine, University of Chicago

Anne Mounsey, MD
Department of Family Medicine, University of North Carolina at Chapel Hill

PURLs EDITOR James J. Stevermer, MD, MSPH
Department of Family Medicine, University of Missouri-Columbia

Author and Disclosure Information

Dana Neutze, MD, PhD
Department of Family Medicine, University of North Carolina at Chapel Hill

Mari Egan, MD
Department of Family Medicine, University of Chicago

Anne Mounsey, MD
Department of Family Medicine, University of North Carolina at Chapel Hill

PURLs EDITOR James J. Stevermer, MD, MSPH
Department of Family Medicine, University of Missouri-Columbia

Article PDF
Article PDF

PRACTICE CHANGER

Consider bariatric surgery for patients with diabetes who are obese; surgery is associated with higher remission rates than medical therapy, regardless of the amount of weight lost.1

STRENGTH OF RECOMMENDATION

B: Based on a single nonblinded randomized controlled trial (RCT).

Mingrone G, Panunzi S, De Gaetano A, et al. Bariatric surgery versus conventional medical therapy for type 2 diabetes. N Engl J Med. 2012;366:1577-1585.

ILLUSTRATIVE CASE

A 43-year-old woman with a body mass index (BMI) of 38 kg/m2 and a 5-year history of diabetes has a glycated hemoglobin (HbA1c) of 8.5% despite the use of oral hypoglycemic agents. Should you talk to her about gastric bypass surgery to treat her diabetes?

Diet and exercise are the first steps in addressing diabetes, but these interventions are often unsuccessful. The International Diabetes Federation (IDF) recommends consideration of bariatric surgery for patients who have a BMI >35 kg/m2 and diabetes that lifestyle modification and pharmacotherapy have failed to control.2

Surgery for diabetes: Is there ample evidence?
Until recently, the IDF’s recommendation was based on observational data and a single RCT that found increased resolution of diabetes following various bariatric procedures.3-5 In the study detailed in this PURL, Mingrone et al took another look.

STUDY SUMMARY: Surgery led to higher remission rates

This single-center, nonblinded RCT compared 2 malabsorptive procedures— Roux-en-Y gastric bypass and biliopancreatic diversion, a more complicated procedure not commonly performed—with medical therapy.1 The primary outcome was the rate of diabetes remission at 2 years, defined as a fasting glucose level <100 mg/dL and an HbA1c <6.5%. Changes in BMI and cholesterol levels were among the secondary endpoints.

To be eligible, patients had to be between the ages of 30 and 60 years and have a BMI ≥35 kg/m2, a history of type 2 diabetes ≥5 years, and an HbA1c ≥7.0%. Exclusion criteria included a history of type 1 diabetes, diabetes caused by an underlying disease or steroid treatment, previous bariatric surgery, pregnancy, diabetic complications, other severe medical conditions, and acute hospitalization. Both the gastric bypass and biliopancreatic diversion procedures were performed by independent surgical teams.

Participants (N=60) were evaluated at baseline and at 1, 3, 6, 9, 12, and 24 months after the intervention by a team that included a dietician, a nurse, and a physician. All received a diet plan with daily exercise designed by their team. Those in the medical therapy group had their medications titrated to reach a goal HbA1c <7%. Pharmacotherapy was stopped based on normalization of blood sugars or HbA1c <6.5%.

Within 15 days postsurgery, patients in both surgical arms had their diabetes medications stopped based on their blood glucose levels.

At 2 years, 75% of the patients in the gastric bypass arm and 95% of the patients in the biliopancreatic diversion arm (number needed to treat=1.3 and 1, respectively) were considered to be in diabetes remission, defined as a fasting blood sugar of <100 mg/dL and an HbA1c <6.5% after one year without pharmacotherapy. (Notably, this differs from that of the American Diabetes Association, which requires an HbA1c <6.0% for classification as complete remission.) None of the patients in the medical therapy arm was in remission at the 2-year mark.

On average, blood sugars normalized for gastric bypass patients by 10±2 months, vs 4±1 months for biliopancreatic diversion patients (P=.01). The average HbA1c at the end of 2 years was significantly different among all 3 groups (6.35%±1.42 for those undergoing gastric bypass, 4.95%±0.49 for the biliopancreatic diversion group, and 7.69%±0.57 for the medical therapy group), as was the change in HbA1c from baseline (TABLE). Changes in BMI and the number of patients who achieved normalization of total cholesterol were similar for both surgical groups. Interestingly, neither baseline BMI nor amount of weight lost or pre-enrollment duration of diabetes were predictors of diabetes remission or normalization of fasting glucose levels.

TABLE
Surgery vs medical therapy for diabetes: Gastric bypass and biliopancreatic diversion are more effective

 Gastric bypass (n=20)Biliopancreatic diversion (n=20)Medical therapy (n=20)
HbA1c at 2 years (%)6.35±1.42*
(n=19)
4.95±0.49
(n=19)
7.69±0.57
(n=18)
HbA1c change from baseline* (%)–25.18±20.89–43.01±9.64–8.39±9.93
BMI change from baseline* (%)–33.31±7.88–33.82±10.17*–4.73±6.37
Total cholesterol normalization (%)100*100*27.3
BMI, body mass index.
*P<0.01 for post hoc analysis comparing surgical arm to medical therapy.
Normalization of cholesterol was defined as a total cholesterol <201 mg/dL and HDL >40 mg/dL in men and >50 mg/dL in women (personal communication from author).

There were no deaths associated with this study. There were 2 adverse events requiring reoperation: an incisional hernia in a patient in the biliopancreatic diversion group and an intestinal obstruction in a patient in the gastric bypass group. Six patients in the biliopancreatic diversion arm developed metabolic abnormalities, including iron deficiency anemia, hypoalbuminemia, osteopenia, and osteoporosis. In the gastric bypass arm, 2 patients developed iron deficiency anemia.

 

 

WHAT’S NEW?: Evidence of efficacy has grown

This is the first RCT to evaluate biliopancreatic diversion and only the second to evaluate gastric bypass as strategies for controlling diabetes. Similar findings were demonstrated at 12 months in an RCT of 150 obese patients with diabetes in which intensive medical therapy was compared with either gastric bypass or sleeve gastrectomy,6 published concurrently with the Mingrone study. Like the Mingrone study, this study found that for select patients with diabetes, surgery may lead to better outcomes than medical management alone.

CAVEATS: Long-term effect is still uncertain

The long-term efficacy of surgery as a way to manage diabetes remains uncertain. Patients in this study were followed for just 2 years and the outcomes were metabolic measures rather than morbidity and mortality. A recent prospective observational study following patients for 6 years after gastric bypass found that the rate of remission for diabetes was 75% (95% confidence interval (CI), 63%-87%) at 2 years but dropped to 62% (95% CI, 49%-75%) at 6 years7

A larger study (N=4047) of longer duration—the Swedish Obese Subjects (SOS) cohort study —found a considerably larger drop: The diabetes remission rate for those who had surgery went from 72% at 2 years to 36% at 10-year follow-up, but that was still higher than the 10-year remission rate (13%) for the matched controls.4 It is still not clear exactly how long diabetic remission lasts after bariatric surgery or what effect a 10-year respite from the disease will have on the long-term morbidity and mortality of patients with diabetes.

Surgical risks. In small studies such as the one by Mingrone et al,1 it can be difficult to see the full extent of surgical complications. The much larger SOS study found low mortality rates (0.25%). But 13% of those who underwent bariatric surgery had postoperative complications (number needed to harm = 8), with 2.2% of patients requiring reoperation.4 Additionally, women who become pregnant after bariatric surgery are at increased risk for internal hernias or bowel obstruction during pregnancy.8

Furthermore, malabsorptive-type surgeries are known to cause nutritional deficiencies, leading to disorders including anemia and osteoporosis.6 Importantly, while women of childbearing-age who undergo bariatric surgery decrease their risk of developing gestational hypertension and gestational diabetes, they are more likely to have nutritional deficiencies during pregnancy and to have children with these deficiencies.8

CHALLENGES TO IMPLEMENTATION: The ideal candidate remains unclear

It is still not clear from this study which patients should be referred for bariatric surgery. Historically, BMI has been used as the main indication for bariatric surgery, but this and other, studies have found that remission of diabetes is independent of BMI and the amount of weight lost.9 A predictive 10-point Diabetes Surgery Score has recently been developed: It uses age, BMI, duration of diabetes, and C-peptide levels to predict the likelihood of diabetes remission after surgery.10 This scoring system has yet to be validated in non-Asian patients, and a threshold for recommending surgery has been not established. However, this tool indicates that younger patients with a shorter duration of diabetes (which was not a factor in the outcome of the Mingrone study) and no baseline use of insulin are most likely to benefit from surgery. Thus, these patients may be the ones we need to consider referring first.

Cost of surgery. Several studies have shown that bariatric surgery is cost-effective for the treatment of diabetes, and saves money after approximately 5 years.11,12 However, patients with diabetes and obesity may be uninsured or underinsured, and have high out-of-pocket costs. One challenge will be to ensure that surgery is a viable option for patients with financial constraints.

Acknowledgement

The PURLs Surveillance System was supported in part by Grant Number UL1RR024999 from the National Center for Research Resources, a Clinical Translational Science Award to the University of Chicago. The content is solely the responsibility of the authors and does not necessarily represent the official views of the National Center for Research Resources or the National Institutes of Health.

PRACTICE CHANGER

Consider bariatric surgery for patients with diabetes who are obese; surgery is associated with higher remission rates than medical therapy, regardless of the amount of weight lost.1

STRENGTH OF RECOMMENDATION

B: Based on a single nonblinded randomized controlled trial (RCT).

Mingrone G, Panunzi S, De Gaetano A, et al. Bariatric surgery versus conventional medical therapy for type 2 diabetes. N Engl J Med. 2012;366:1577-1585.

ILLUSTRATIVE CASE

A 43-year-old woman with a body mass index (BMI) of 38 kg/m2 and a 5-year history of diabetes has a glycated hemoglobin (HbA1c) of 8.5% despite the use of oral hypoglycemic agents. Should you talk to her about gastric bypass surgery to treat her diabetes?

Diet and exercise are the first steps in addressing diabetes, but these interventions are often unsuccessful. The International Diabetes Federation (IDF) recommends consideration of bariatric surgery for patients who have a BMI >35 kg/m2 and diabetes that lifestyle modification and pharmacotherapy have failed to control.2

Surgery for diabetes: Is there ample evidence?
Until recently, the IDF’s recommendation was based on observational data and a single RCT that found increased resolution of diabetes following various bariatric procedures.3-5 In the study detailed in this PURL, Mingrone et al took another look.

STUDY SUMMARY: Surgery led to higher remission rates

This single-center, nonblinded RCT compared 2 malabsorptive procedures— Roux-en-Y gastric bypass and biliopancreatic diversion, a more complicated procedure not commonly performed—with medical therapy.1 The primary outcome was the rate of diabetes remission at 2 years, defined as a fasting glucose level <100 mg/dL and an HbA1c <6.5%. Changes in BMI and cholesterol levels were among the secondary endpoints.

To be eligible, patients had to be between the ages of 30 and 60 years and have a BMI ≥35 kg/m2, a history of type 2 diabetes ≥5 years, and an HbA1c ≥7.0%. Exclusion criteria included a history of type 1 diabetes, diabetes caused by an underlying disease or steroid treatment, previous bariatric surgery, pregnancy, diabetic complications, other severe medical conditions, and acute hospitalization. Both the gastric bypass and biliopancreatic diversion procedures were performed by independent surgical teams.

Participants (N=60) were evaluated at baseline and at 1, 3, 6, 9, 12, and 24 months after the intervention by a team that included a dietician, a nurse, and a physician. All received a diet plan with daily exercise designed by their team. Those in the medical therapy group had their medications titrated to reach a goal HbA1c <7%. Pharmacotherapy was stopped based on normalization of blood sugars or HbA1c <6.5%.

Within 15 days postsurgery, patients in both surgical arms had their diabetes medications stopped based on their blood glucose levels.

At 2 years, 75% of the patients in the gastric bypass arm and 95% of the patients in the biliopancreatic diversion arm (number needed to treat=1.3 and 1, respectively) were considered to be in diabetes remission, defined as a fasting blood sugar of <100 mg/dL and an HbA1c <6.5% after one year without pharmacotherapy. (Notably, this differs from that of the American Diabetes Association, which requires an HbA1c <6.0% for classification as complete remission.) None of the patients in the medical therapy arm was in remission at the 2-year mark.

On average, blood sugars normalized for gastric bypass patients by 10±2 months, vs 4±1 months for biliopancreatic diversion patients (P=.01). The average HbA1c at the end of 2 years was significantly different among all 3 groups (6.35%±1.42 for those undergoing gastric bypass, 4.95%±0.49 for the biliopancreatic diversion group, and 7.69%±0.57 for the medical therapy group), as was the change in HbA1c from baseline (TABLE). Changes in BMI and the number of patients who achieved normalization of total cholesterol were similar for both surgical groups. Interestingly, neither baseline BMI nor amount of weight lost or pre-enrollment duration of diabetes were predictors of diabetes remission or normalization of fasting glucose levels.

TABLE
Surgery vs medical therapy for diabetes: Gastric bypass and biliopancreatic diversion are more effective

 Gastric bypass (n=20)Biliopancreatic diversion (n=20)Medical therapy (n=20)
HbA1c at 2 years (%)6.35±1.42*
(n=19)
4.95±0.49
(n=19)
7.69±0.57
(n=18)
HbA1c change from baseline* (%)–25.18±20.89–43.01±9.64–8.39±9.93
BMI change from baseline* (%)–33.31±7.88–33.82±10.17*–4.73±6.37
Total cholesterol normalization (%)100*100*27.3
BMI, body mass index.
*P<0.01 for post hoc analysis comparing surgical arm to medical therapy.
Normalization of cholesterol was defined as a total cholesterol <201 mg/dL and HDL >40 mg/dL in men and >50 mg/dL in women (personal communication from author).

There were no deaths associated with this study. There were 2 adverse events requiring reoperation: an incisional hernia in a patient in the biliopancreatic diversion group and an intestinal obstruction in a patient in the gastric bypass group. Six patients in the biliopancreatic diversion arm developed metabolic abnormalities, including iron deficiency anemia, hypoalbuminemia, osteopenia, and osteoporosis. In the gastric bypass arm, 2 patients developed iron deficiency anemia.

 

 

WHAT’S NEW?: Evidence of efficacy has grown

This is the first RCT to evaluate biliopancreatic diversion and only the second to evaluate gastric bypass as strategies for controlling diabetes. Similar findings were demonstrated at 12 months in an RCT of 150 obese patients with diabetes in which intensive medical therapy was compared with either gastric bypass or sleeve gastrectomy,6 published concurrently with the Mingrone study. Like the Mingrone study, this study found that for select patients with diabetes, surgery may lead to better outcomes than medical management alone.

CAVEATS: Long-term effect is still uncertain

The long-term efficacy of surgery as a way to manage diabetes remains uncertain. Patients in this study were followed for just 2 years and the outcomes were metabolic measures rather than morbidity and mortality. A recent prospective observational study following patients for 6 years after gastric bypass found that the rate of remission for diabetes was 75% (95% confidence interval (CI), 63%-87%) at 2 years but dropped to 62% (95% CI, 49%-75%) at 6 years7

A larger study (N=4047) of longer duration—the Swedish Obese Subjects (SOS) cohort study —found a considerably larger drop: The diabetes remission rate for those who had surgery went from 72% at 2 years to 36% at 10-year follow-up, but that was still higher than the 10-year remission rate (13%) for the matched controls.4 It is still not clear exactly how long diabetic remission lasts after bariatric surgery or what effect a 10-year respite from the disease will have on the long-term morbidity and mortality of patients with diabetes.

Surgical risks. In small studies such as the one by Mingrone et al,1 it can be difficult to see the full extent of surgical complications. The much larger SOS study found low mortality rates (0.25%). But 13% of those who underwent bariatric surgery had postoperative complications (number needed to harm = 8), with 2.2% of patients requiring reoperation.4 Additionally, women who become pregnant after bariatric surgery are at increased risk for internal hernias or bowel obstruction during pregnancy.8

Furthermore, malabsorptive-type surgeries are known to cause nutritional deficiencies, leading to disorders including anemia and osteoporosis.6 Importantly, while women of childbearing-age who undergo bariatric surgery decrease their risk of developing gestational hypertension and gestational diabetes, they are more likely to have nutritional deficiencies during pregnancy and to have children with these deficiencies.8

CHALLENGES TO IMPLEMENTATION: The ideal candidate remains unclear

It is still not clear from this study which patients should be referred for bariatric surgery. Historically, BMI has been used as the main indication for bariatric surgery, but this and other, studies have found that remission of diabetes is independent of BMI and the amount of weight lost.9 A predictive 10-point Diabetes Surgery Score has recently been developed: It uses age, BMI, duration of diabetes, and C-peptide levels to predict the likelihood of diabetes remission after surgery.10 This scoring system has yet to be validated in non-Asian patients, and a threshold for recommending surgery has been not established. However, this tool indicates that younger patients with a shorter duration of diabetes (which was not a factor in the outcome of the Mingrone study) and no baseline use of insulin are most likely to benefit from surgery. Thus, these patients may be the ones we need to consider referring first.

Cost of surgery. Several studies have shown that bariatric surgery is cost-effective for the treatment of diabetes, and saves money after approximately 5 years.11,12 However, patients with diabetes and obesity may be uninsured or underinsured, and have high out-of-pocket costs. One challenge will be to ensure that surgery is a viable option for patients with financial constraints.

Acknowledgement

The PURLs Surveillance System was supported in part by Grant Number UL1RR024999 from the National Center for Research Resources, a Clinical Translational Science Award to the University of Chicago. The content is solely the responsibility of the authors and does not necessarily represent the official views of the National Center for Research Resources or the National Institutes of Health.

References

1. Mingrone G, Panunzi S, De Gaetano A, et al. Bariatric surgery versus conventional medical therapy for type 2 diabetes. N Engl J Med. 2012;366:1577-1585.

2. Dixon JB, Zimmet P, Alberti KG, et al. Bariatric surgery: an IDF statement for obese type 2 diabetes. Diabet Med. 2011;28:628-642.

3. Buchwald H, Estok R, Fahrbach K, et al. Weight and type 2 diabetes after bariatric surgery: systematic review and meta-analysis. Am J Med. 2009;122:248-256.

4. Sjöström L, Lindroos AK, Peltonen M, et al. Lifestyle, diabetes, and cardiovascular risk factors 10 years after bariatric surgery. N Engl J Med. 2004;351:2683-2693.

5. Dixon JB, O’Brien PE, Playfair J, et al. Adjustable gastric banding and conventional therapy for type 2 diabetes: a randomized controlled trial. JAMA. 2008;299:316-323.

6. Schauer PR, Kashyap SR, Wolski K, et al. Bariatric surgery versus intensive medical therapy in obese patients with diabetes. N Engl J Med. 2012;366:1568-1576.

7. Adams TD, Davidson LE, Litwin SE, et al. Health benefits of gastric bypass surgery after 6 years. JAMA. 2012;308:1122-1131.

8. Dalfrà MG, Busetto L, Chilelli NC, et al. Pregnancy and foetal outcome after bariatric surgery: a review of recent studies. J Matern Fetal Neonatal Med. 2012;25:1537-1543.

9. Livingston EH Pitfalls in using BMI as a selection criterion for bariatric surgery. Curr Opin Endocrinol Diabetes Obes. 2012;19:347-351.

10. Lee W-J, Hur K, Lakadawala M, et al. Predicting success of metabolic surgery: age, body mass index, C-peptide, and duration score. Surg Obes Relat Dis. 2012; [Epub ahead of print].

11. Terranova L, Busetto L, Vestri A, et al. Bariatric surgery: cost-effectiveness and budget impact. Obes Surg. 2012;22:646-653.

12. Hoerger TJ, Zhang P, Segel JE, et al. Cost-effectiveness of bariatric surgery for severely obese adults with diabetes. Diabetes Care. 2010;33:1933-1939.

References

1. Mingrone G, Panunzi S, De Gaetano A, et al. Bariatric surgery versus conventional medical therapy for type 2 diabetes. N Engl J Med. 2012;366:1577-1585.

2. Dixon JB, Zimmet P, Alberti KG, et al. Bariatric surgery: an IDF statement for obese type 2 diabetes. Diabet Med. 2011;28:628-642.

3. Buchwald H, Estok R, Fahrbach K, et al. Weight and type 2 diabetes after bariatric surgery: systematic review and meta-analysis. Am J Med. 2009;122:248-256.

4. Sjöström L, Lindroos AK, Peltonen M, et al. Lifestyle, diabetes, and cardiovascular risk factors 10 years after bariatric surgery. N Engl J Med. 2004;351:2683-2693.

5. Dixon JB, O’Brien PE, Playfair J, et al. Adjustable gastric banding and conventional therapy for type 2 diabetes: a randomized controlled trial. JAMA. 2008;299:316-323.

6. Schauer PR, Kashyap SR, Wolski K, et al. Bariatric surgery versus intensive medical therapy in obese patients with diabetes. N Engl J Med. 2012;366:1568-1576.

7. Adams TD, Davidson LE, Litwin SE, et al. Health benefits of gastric bypass surgery after 6 years. JAMA. 2012;308:1122-1131.

8. Dalfrà MG, Busetto L, Chilelli NC, et al. Pregnancy and foetal outcome after bariatric surgery: a review of recent studies. J Matern Fetal Neonatal Med. 2012;25:1537-1543.

9. Livingston EH Pitfalls in using BMI as a selection criterion for bariatric surgery. Curr Opin Endocrinol Diabetes Obes. 2012;19:347-351.

10. Lee W-J, Hur K, Lakadawala M, et al. Predicting success of metabolic surgery: age, body mass index, C-peptide, and duration score. Surg Obes Relat Dis. 2012; [Epub ahead of print].

11. Terranova L, Busetto L, Vestri A, et al. Bariatric surgery: cost-effectiveness and budget impact. Obes Surg. 2012;22:646-653.

12. Hoerger TJ, Zhang P, Segel JE, et al. Cost-effectiveness of bariatric surgery for severely obese adults with diabetes. Diabetes Care. 2010;33:1933-1939.

Issue
The Journal of Family Practice - 62(1)
Issue
The Journal of Family Practice - 62(1)
Page Number
30-32
Page Number
30-32
Publications
Publications
Topics
Article Type
Display Headline
An obesity remedy for diabetes
Display Headline
An obesity remedy for diabetes
Legacy Keywords
Dana Neutze; MD; PhD; Mari Egan; MD; Anne Mounsey; MD; bariatric surgery; malabsorptive processes; gastric bypass; Roux-en-Y; biliopancreatic diversion; remission; PURLs; obesity; diabetes
Legacy Keywords
Dana Neutze; MD; PhD; Mari Egan; MD; Anne Mounsey; MD; bariatric surgery; malabsorptive processes; gastric bypass; Roux-en-Y; biliopancreatic diversion; remission; PURLs; obesity; diabetes
Sections
Article Source

PURLs Copyright

Copyright © 2013 The Family Physicians Inquiries Network. All rights reserved.

Inside the Article

Article PDF Media

Injection may be the best bet for young athletes’ knee pain

Article Type
Changed
Mon, 01/14/2019 - 11:34
Display Headline
Injection may be the best bet for young athletes’ knee pain
PRACTICE CHANGER

Consider giving dextrose/lidocaine injections to adolescents with Osgood-Schlatter disease (OSD) that persists despite physical therapy.1

STRENGTH OF RECOMMENDATION

A: Based on one well-designed, randomized controlled trial (RCT).

Topol GA, Podesta LA, Reeves KD, et al. Hyperosmolar dextrose injection for recalcitrant Osgood-Schlatter disease. Pediatrics. 2011;128: e1121-e1128.

 

ILLUSTRATIVE CASE

A 13-year-old boy comes in to your office for follow-up of anterior knee pain from OSD that has not responded to 2 months of physical therapy. he is still unable to play on his recreational soccer team. What treatment can you offer to help him return to the sport he enjoys?

OSD is characterized by inflammation of the growth plate just below the knee, the result of repetitive strain on the secondary ossification center of the tibial tuberosity.2 Closure of the tibial growth plate is the definitive remedy for OSD, but the pain that some adolescents experience until that happens can be long-lasting and considerable. Nine years after diagnosis of OSD, one study found, up to 60% of patients who had received conservative treatment reported pain on kneeling and 18% had sports limitations.3

Inability to play may affect self-esteem
Adolescents whose recreational activities are limited due to OSD may experience a number of negative effects, including alienation from friends, altered peer group dynamics, and a decline in self-esteem. Surgery, which involves excision of the pain-producing ossicle with or without tuberculoplasty, relieves the pain and allows patients to return to their chosen sport in 90% to 95% of cases that have not responded to conservative treatment.4,5 For a self-limiting (although prolonged) condition like OSD, most physicians and patients would prefer to avoid surgery and opt for a more conservative approach.

Dextrose injections have been shown to be safe and effective when used for the treatment of tendon and ligamentous disorders such as Achilles tendonitis and lateral epicondylitis, although the mechanism of action is not clear.6,7 The study detailed in this PURL is the first prospective RCT of dextrose injections for the treatment of OSD.

STUDY SUMMARY: injections get adolescents back in the game

Topol et al1 sought to compare the efficacy of injections of dextrose and lidocaine with lidocaine-only injections or supervised usual care in treating OSD in young athletes. Sixty-six Argentinian boys and girls ages 9 to 17 years, all of whom had anterior knee pain and participated in kicking or jumping sports on organized teams, were considered for the study. The absence of either patellofemoral crepitus or proximal patellar tendon tenderness was a prerequisite for participation, as was reproduction of the anterior knee pain and localization of pain precisely to the tibial tuberosity during a single leg squat to confirm the OSD diagnosis.

After diagnosis, the patients completed ≥2 months of formal and gently progressive hamstring stretching, quads strengthening, and gradual reintroduction into their respective sports. Those who experienced pain during team play that persisted for ≥3 months—54 patients, all but 3 of whom were male, with a total of 65 knees requiring treatment—were included in the study. Participants were randomized to the usual care group or to one of the injection groups, which was blinded to patients, guardians, and physicians.

The injection groups received a solution of lidocaine 1%, alone or with 12.5% dextrose, at the start of the study and again at 1 and 2 months. Adequate injection was determined by complete pain relief during a single leg squat, which was also used to determine both proximal and distal points of tenderness. Both injection groups received 0.5-mL injections with a 27-gauge needle, repeated at approximately 1-cm intervals for a total of 3 to 4 midline injections. After 5 minutes, the leg squat was repeated to detect any remaining pain, and painful areas were injected until the patient could do a pain-free leg squat.

Because pain reduction may precede full healing, those in both the lidocaine-only and the dextrose-lidocaine groups received injections on all 3 occasions even if they were pain free. They were instructed to avoid running for a week after the initial treatment and then to run as tolerated. Subsequent treatments required a 3-day rest from running. Participants were able to return to their sport after the second injection and rest period.

Patients in all 3 groups received handouts explaining hamstring stretches and quadriceps strengthening exercises. The usual care group received individual instruction from a physical therapist. They were also given a video and returned at least once, both to ensure that they were performing the exercises correctly and to encourage compliance.

The primary outcome involved the Nirschl Pain Phase Scale (NPPS), a 7-point measure of sports-related symptoms and level of participation. Scores of 4 to 7 represent sports limitation resulting from pain. Scores <4 (which may involve soreness or pain but participation in the sport is unlimited) and 0 (asymptomatic participation) were the threshold goals for the study.

 

 

 

The groups were similar at baseline, and follow-up was 100%. At 3 months, NPPS scores improved more in dextrose-treated knees than in either the lidocaine-treated knees (3.9 vs 2.4; P=.004) or those who received usual care (3.9 vs 1.2; P=.001), and lidocaine alone was significantly better than usual care (2.4 vs 1.2; P=.024). More than 90% of participants in both injection groups achieved unlimited sports participation by 3 months. However, knees treated with dextrose were significantly more likely than lidocaine-treated knees to allow asymptomatic participation (NPPS=0), with 14 of 21 knees and 5 of 22 knees, respectively, being pain-free. After one year, more dextrose-treated knees than lidocaine-treated knees were asymptomatic with sports participation (32 of 38 vs 6 of 13; P=.024).

There were no reported adverse effects during this study and fewer than 10% of subjects required acetaminophen for postinjection pain control.

WHAT’S NEW: OSD can be safely and effectively treated

This study found dextrose injections to be safe, well tolerated, and effective in treating patients with intractable OSD symptoms. The results suggest that the duration of both the sports limitation and sports-related symptoms may be reduced with dextrose injections in adolescent athletes with recalcitrant OSD.

CAVEATS: Lack of validated measure, controls

NPPS is not an ideal measure of OSD symptoms because it has not been validated. The failure to use a validated measure of tendinopathy symptoms (eg, the Victorian Institute of Sport Assessment-Patella8) is a significant limitation of this trial. The athletes included in this study had already failed to respond to the usual treatment, which suggests that injections should be reserved for those who have tried hamstring stretching and quad-strengthening exercises.

CHALLENGES TO IMPLEMENTATION: Patient and provider comfort may be an issue

Although the injections in this study were well tolerated, there is a risk of infection, bleeding, and pain with any injection or invasive procedure. In addition, adolescents often have difficulty tolerating injections, especially repeated needlesticks like those called for in the proposed treatment. The nonviscous nature of dextrose allows 27- to 30-gauge needles to be used, which may make the injections easier for teens to tolerate. Some physicians may be hesitant to start these young patients on a new injectable therapy.

Click here to view PURL METHODOLOGY

References

1. Topol GA, Podesta LA, Reeves KD, et al. Hyperosmolar dextrose injection for recalcitrant Osgood-Schlatter disease. Pediatrics. 2011;128:e1121-e1128.

2. Gholve PA, Scher DM, Khakharia S, et al. Osgood-Schlatter syndrome. Curr Opin Pediatr. 2007;19:44-50.

3. Krause BL, Willimas JP, Caterall A. Natural history of OsgoodSchlatter disease. J Pediatr Orthop. 1990;10:65-68.

4. Nierenberg G, Falah M, Keren Y, et al. Surgical treatment of residual Osgood-Schlatter disease in young adults: role of the mobile osseous fragment. Orthopedics. 2011;34:176.-

5. El-Husseini TF, Abdelgawad AA. Results of surgical treatment of unresolved Osgood-Schlatter disease in adults. J Knee Surg. 2010;23:103-107.

6. Ryan M, Wong A, Taunton J. Favorable outcomes after sonographically guided intratendinous injection of hyperosmolar dextrose for chronic insertional and midportion achilles tendinosis. AJR Am J Roentgenol. 2010;194:1047-1053.

7. Scarpone M, Rabago D, Zgierska A, et al. The efficacy of prolotherapy for lateral epicondylitis: a pilot study. Clin J Sport Med. 2008;18:248-254.

8. Visentini PJ, Khan KM, Cook JL, et al. The VISA score: an index of severity of symptoms in patients with jumper’s knee (patellar tendinosis). Victorian Institute of Sport Tendon Study Group. J Sci Med Sport. 1998;1:22-28.

Article PDF
Author and Disclosure Information
Young athlete sidelined? Check to see how he’s coping
Samantha O’Connell, PhD

Mark Sakr, DO
Department of Family Medicine, University of North Carolina at Chapel Hill

Anne Mounsey, MD
Department of Family Medicine, University of North Carolina at Chapel Hill

PURLs EDITOR
John Hickner, MD, MSc
Cleveland Clinic

Issue
The Journal of Family Practice - 61(8)
Publications
Topics
Page Number
486-488
Sections
Author and Disclosure Information
Young athlete sidelined? Check to see how he’s coping
Samantha O’Connell, PhD

Mark Sakr, DO
Department of Family Medicine, University of North Carolina at Chapel Hill

Anne Mounsey, MD
Department of Family Medicine, University of North Carolina at Chapel Hill

PURLs EDITOR
John Hickner, MD, MSc
Cleveland Clinic

Author and Disclosure Information
Young athlete sidelined? Check to see how he’s coping
Samantha O’Connell, PhD

Mark Sakr, DO
Department of Family Medicine, University of North Carolina at Chapel Hill

Anne Mounsey, MD
Department of Family Medicine, University of North Carolina at Chapel Hill

PURLs EDITOR
John Hickner, MD, MSc
Cleveland Clinic

Article PDF
Article PDF
PRACTICE CHANGER

Consider giving dextrose/lidocaine injections to adolescents with Osgood-Schlatter disease (OSD) that persists despite physical therapy.1

STRENGTH OF RECOMMENDATION

A: Based on one well-designed, randomized controlled trial (RCT).

Topol GA, Podesta LA, Reeves KD, et al. Hyperosmolar dextrose injection for recalcitrant Osgood-Schlatter disease. Pediatrics. 2011;128: e1121-e1128.

 

ILLUSTRATIVE CASE

A 13-year-old boy comes in to your office for follow-up of anterior knee pain from OSD that has not responded to 2 months of physical therapy. he is still unable to play on his recreational soccer team. What treatment can you offer to help him return to the sport he enjoys?

OSD is characterized by inflammation of the growth plate just below the knee, the result of repetitive strain on the secondary ossification center of the tibial tuberosity.2 Closure of the tibial growth plate is the definitive remedy for OSD, but the pain that some adolescents experience until that happens can be long-lasting and considerable. Nine years after diagnosis of OSD, one study found, up to 60% of patients who had received conservative treatment reported pain on kneeling and 18% had sports limitations.3

Inability to play may affect self-esteem
Adolescents whose recreational activities are limited due to OSD may experience a number of negative effects, including alienation from friends, altered peer group dynamics, and a decline in self-esteem. Surgery, which involves excision of the pain-producing ossicle with or without tuberculoplasty, relieves the pain and allows patients to return to their chosen sport in 90% to 95% of cases that have not responded to conservative treatment.4,5 For a self-limiting (although prolonged) condition like OSD, most physicians and patients would prefer to avoid surgery and opt for a more conservative approach.

Dextrose injections have been shown to be safe and effective when used for the treatment of tendon and ligamentous disorders such as Achilles tendonitis and lateral epicondylitis, although the mechanism of action is not clear.6,7 The study detailed in this PURL is the first prospective RCT of dextrose injections for the treatment of OSD.

STUDY SUMMARY: injections get adolescents back in the game

Topol et al1 sought to compare the efficacy of injections of dextrose and lidocaine with lidocaine-only injections or supervised usual care in treating OSD in young athletes. Sixty-six Argentinian boys and girls ages 9 to 17 years, all of whom had anterior knee pain and participated in kicking or jumping sports on organized teams, were considered for the study. The absence of either patellofemoral crepitus or proximal patellar tendon tenderness was a prerequisite for participation, as was reproduction of the anterior knee pain and localization of pain precisely to the tibial tuberosity during a single leg squat to confirm the OSD diagnosis.

After diagnosis, the patients completed ≥2 months of formal and gently progressive hamstring stretching, quads strengthening, and gradual reintroduction into their respective sports. Those who experienced pain during team play that persisted for ≥3 months—54 patients, all but 3 of whom were male, with a total of 65 knees requiring treatment—were included in the study. Participants were randomized to the usual care group or to one of the injection groups, which was blinded to patients, guardians, and physicians.

The injection groups received a solution of lidocaine 1%, alone or with 12.5% dextrose, at the start of the study and again at 1 and 2 months. Adequate injection was determined by complete pain relief during a single leg squat, which was also used to determine both proximal and distal points of tenderness. Both injection groups received 0.5-mL injections with a 27-gauge needle, repeated at approximately 1-cm intervals for a total of 3 to 4 midline injections. After 5 minutes, the leg squat was repeated to detect any remaining pain, and painful areas were injected until the patient could do a pain-free leg squat.

Because pain reduction may precede full healing, those in both the lidocaine-only and the dextrose-lidocaine groups received injections on all 3 occasions even if they were pain free. They were instructed to avoid running for a week after the initial treatment and then to run as tolerated. Subsequent treatments required a 3-day rest from running. Participants were able to return to their sport after the second injection and rest period.

Patients in all 3 groups received handouts explaining hamstring stretches and quadriceps strengthening exercises. The usual care group received individual instruction from a physical therapist. They were also given a video and returned at least once, both to ensure that they were performing the exercises correctly and to encourage compliance.

The primary outcome involved the Nirschl Pain Phase Scale (NPPS), a 7-point measure of sports-related symptoms and level of participation. Scores of 4 to 7 represent sports limitation resulting from pain. Scores <4 (which may involve soreness or pain but participation in the sport is unlimited) and 0 (asymptomatic participation) were the threshold goals for the study.

 

 

 

The groups were similar at baseline, and follow-up was 100%. At 3 months, NPPS scores improved more in dextrose-treated knees than in either the lidocaine-treated knees (3.9 vs 2.4; P=.004) or those who received usual care (3.9 vs 1.2; P=.001), and lidocaine alone was significantly better than usual care (2.4 vs 1.2; P=.024). More than 90% of participants in both injection groups achieved unlimited sports participation by 3 months. However, knees treated with dextrose were significantly more likely than lidocaine-treated knees to allow asymptomatic participation (NPPS=0), with 14 of 21 knees and 5 of 22 knees, respectively, being pain-free. After one year, more dextrose-treated knees than lidocaine-treated knees were asymptomatic with sports participation (32 of 38 vs 6 of 13; P=.024).

There were no reported adverse effects during this study and fewer than 10% of subjects required acetaminophen for postinjection pain control.

WHAT’S NEW: OSD can be safely and effectively treated

This study found dextrose injections to be safe, well tolerated, and effective in treating patients with intractable OSD symptoms. The results suggest that the duration of both the sports limitation and sports-related symptoms may be reduced with dextrose injections in adolescent athletes with recalcitrant OSD.

CAVEATS: Lack of validated measure, controls

NPPS is not an ideal measure of OSD symptoms because it has not been validated. The failure to use a validated measure of tendinopathy symptoms (eg, the Victorian Institute of Sport Assessment-Patella8) is a significant limitation of this trial. The athletes included in this study had already failed to respond to the usual treatment, which suggests that injections should be reserved for those who have tried hamstring stretching and quad-strengthening exercises.

CHALLENGES TO IMPLEMENTATION: Patient and provider comfort may be an issue

Although the injections in this study were well tolerated, there is a risk of infection, bleeding, and pain with any injection or invasive procedure. In addition, adolescents often have difficulty tolerating injections, especially repeated needlesticks like those called for in the proposed treatment. The nonviscous nature of dextrose allows 27- to 30-gauge needles to be used, which may make the injections easier for teens to tolerate. Some physicians may be hesitant to start these young patients on a new injectable therapy.

Click here to view PURL METHODOLOGY

PRACTICE CHANGER

Consider giving dextrose/lidocaine injections to adolescents with Osgood-Schlatter disease (OSD) that persists despite physical therapy.1

STRENGTH OF RECOMMENDATION

A: Based on one well-designed, randomized controlled trial (RCT).

Topol GA, Podesta LA, Reeves KD, et al. Hyperosmolar dextrose injection for recalcitrant Osgood-Schlatter disease. Pediatrics. 2011;128: e1121-e1128.

 

ILLUSTRATIVE CASE

A 13-year-old boy comes in to your office for follow-up of anterior knee pain from OSD that has not responded to 2 months of physical therapy. he is still unable to play on his recreational soccer team. What treatment can you offer to help him return to the sport he enjoys?

OSD is characterized by inflammation of the growth plate just below the knee, the result of repetitive strain on the secondary ossification center of the tibial tuberosity.2 Closure of the tibial growth plate is the definitive remedy for OSD, but the pain that some adolescents experience until that happens can be long-lasting and considerable. Nine years after diagnosis of OSD, one study found, up to 60% of patients who had received conservative treatment reported pain on kneeling and 18% had sports limitations.3

Inability to play may affect self-esteem
Adolescents whose recreational activities are limited due to OSD may experience a number of negative effects, including alienation from friends, altered peer group dynamics, and a decline in self-esteem. Surgery, which involves excision of the pain-producing ossicle with or without tuberculoplasty, relieves the pain and allows patients to return to their chosen sport in 90% to 95% of cases that have not responded to conservative treatment.4,5 For a self-limiting (although prolonged) condition like OSD, most physicians and patients would prefer to avoid surgery and opt for a more conservative approach.

Dextrose injections have been shown to be safe and effective when used for the treatment of tendon and ligamentous disorders such as Achilles tendonitis and lateral epicondylitis, although the mechanism of action is not clear.6,7 The study detailed in this PURL is the first prospective RCT of dextrose injections for the treatment of OSD.

STUDY SUMMARY: injections get adolescents back in the game

Topol et al1 sought to compare the efficacy of injections of dextrose and lidocaine with lidocaine-only injections or supervised usual care in treating OSD in young athletes. Sixty-six Argentinian boys and girls ages 9 to 17 years, all of whom had anterior knee pain and participated in kicking or jumping sports on organized teams, were considered for the study. The absence of either patellofemoral crepitus or proximal patellar tendon tenderness was a prerequisite for participation, as was reproduction of the anterior knee pain and localization of pain precisely to the tibial tuberosity during a single leg squat to confirm the OSD diagnosis.

After diagnosis, the patients completed ≥2 months of formal and gently progressive hamstring stretching, quads strengthening, and gradual reintroduction into their respective sports. Those who experienced pain during team play that persisted for ≥3 months—54 patients, all but 3 of whom were male, with a total of 65 knees requiring treatment—were included in the study. Participants were randomized to the usual care group or to one of the injection groups, which was blinded to patients, guardians, and physicians.

The injection groups received a solution of lidocaine 1%, alone or with 12.5% dextrose, at the start of the study and again at 1 and 2 months. Adequate injection was determined by complete pain relief during a single leg squat, which was also used to determine both proximal and distal points of tenderness. Both injection groups received 0.5-mL injections with a 27-gauge needle, repeated at approximately 1-cm intervals for a total of 3 to 4 midline injections. After 5 minutes, the leg squat was repeated to detect any remaining pain, and painful areas were injected until the patient could do a pain-free leg squat.

Because pain reduction may precede full healing, those in both the lidocaine-only and the dextrose-lidocaine groups received injections on all 3 occasions even if they were pain free. They were instructed to avoid running for a week after the initial treatment and then to run as tolerated. Subsequent treatments required a 3-day rest from running. Participants were able to return to their sport after the second injection and rest period.

Patients in all 3 groups received handouts explaining hamstring stretches and quadriceps strengthening exercises. The usual care group received individual instruction from a physical therapist. They were also given a video and returned at least once, both to ensure that they were performing the exercises correctly and to encourage compliance.

The primary outcome involved the Nirschl Pain Phase Scale (NPPS), a 7-point measure of sports-related symptoms and level of participation. Scores of 4 to 7 represent sports limitation resulting from pain. Scores <4 (which may involve soreness or pain but participation in the sport is unlimited) and 0 (asymptomatic participation) were the threshold goals for the study.

 

 

 

The groups were similar at baseline, and follow-up was 100%. At 3 months, NPPS scores improved more in dextrose-treated knees than in either the lidocaine-treated knees (3.9 vs 2.4; P=.004) or those who received usual care (3.9 vs 1.2; P=.001), and lidocaine alone was significantly better than usual care (2.4 vs 1.2; P=.024). More than 90% of participants in both injection groups achieved unlimited sports participation by 3 months. However, knees treated with dextrose were significantly more likely than lidocaine-treated knees to allow asymptomatic participation (NPPS=0), with 14 of 21 knees and 5 of 22 knees, respectively, being pain-free. After one year, more dextrose-treated knees than lidocaine-treated knees were asymptomatic with sports participation (32 of 38 vs 6 of 13; P=.024).

There were no reported adverse effects during this study and fewer than 10% of subjects required acetaminophen for postinjection pain control.

WHAT’S NEW: OSD can be safely and effectively treated

This study found dextrose injections to be safe, well tolerated, and effective in treating patients with intractable OSD symptoms. The results suggest that the duration of both the sports limitation and sports-related symptoms may be reduced with dextrose injections in adolescent athletes with recalcitrant OSD.

CAVEATS: Lack of validated measure, controls

NPPS is not an ideal measure of OSD symptoms because it has not been validated. The failure to use a validated measure of tendinopathy symptoms (eg, the Victorian Institute of Sport Assessment-Patella8) is a significant limitation of this trial. The athletes included in this study had already failed to respond to the usual treatment, which suggests that injections should be reserved for those who have tried hamstring stretching and quad-strengthening exercises.

CHALLENGES TO IMPLEMENTATION: Patient and provider comfort may be an issue

Although the injections in this study were well tolerated, there is a risk of infection, bleeding, and pain with any injection or invasive procedure. In addition, adolescents often have difficulty tolerating injections, especially repeated needlesticks like those called for in the proposed treatment. The nonviscous nature of dextrose allows 27- to 30-gauge needles to be used, which may make the injections easier for teens to tolerate. Some physicians may be hesitant to start these young patients on a new injectable therapy.

Click here to view PURL METHODOLOGY

References

1. Topol GA, Podesta LA, Reeves KD, et al. Hyperosmolar dextrose injection for recalcitrant Osgood-Schlatter disease. Pediatrics. 2011;128:e1121-e1128.

2. Gholve PA, Scher DM, Khakharia S, et al. Osgood-Schlatter syndrome. Curr Opin Pediatr. 2007;19:44-50.

3. Krause BL, Willimas JP, Caterall A. Natural history of OsgoodSchlatter disease. J Pediatr Orthop. 1990;10:65-68.

4. Nierenberg G, Falah M, Keren Y, et al. Surgical treatment of residual Osgood-Schlatter disease in young adults: role of the mobile osseous fragment. Orthopedics. 2011;34:176.-

5. El-Husseini TF, Abdelgawad AA. Results of surgical treatment of unresolved Osgood-Schlatter disease in adults. J Knee Surg. 2010;23:103-107.

6. Ryan M, Wong A, Taunton J. Favorable outcomes after sonographically guided intratendinous injection of hyperosmolar dextrose for chronic insertional and midportion achilles tendinosis. AJR Am J Roentgenol. 2010;194:1047-1053.

7. Scarpone M, Rabago D, Zgierska A, et al. The efficacy of prolotherapy for lateral epicondylitis: a pilot study. Clin J Sport Med. 2008;18:248-254.

8. Visentini PJ, Khan KM, Cook JL, et al. The VISA score: an index of severity of symptoms in patients with jumper’s knee (patellar tendinosis). Victorian Institute of Sport Tendon Study Group. J Sci Med Sport. 1998;1:22-28.

References

1. Topol GA, Podesta LA, Reeves KD, et al. Hyperosmolar dextrose injection for recalcitrant Osgood-Schlatter disease. Pediatrics. 2011;128:e1121-e1128.

2. Gholve PA, Scher DM, Khakharia S, et al. Osgood-Schlatter syndrome. Curr Opin Pediatr. 2007;19:44-50.

3. Krause BL, Willimas JP, Caterall A. Natural history of OsgoodSchlatter disease. J Pediatr Orthop. 1990;10:65-68.

4. Nierenberg G, Falah M, Keren Y, et al. Surgical treatment of residual Osgood-Schlatter disease in young adults: role of the mobile osseous fragment. Orthopedics. 2011;34:176.-

5. El-Husseini TF, Abdelgawad AA. Results of surgical treatment of unresolved Osgood-Schlatter disease in adults. J Knee Surg. 2010;23:103-107.

6. Ryan M, Wong A, Taunton J. Favorable outcomes after sonographically guided intratendinous injection of hyperosmolar dextrose for chronic insertional and midportion achilles tendinosis. AJR Am J Roentgenol. 2010;194:1047-1053.

7. Scarpone M, Rabago D, Zgierska A, et al. The efficacy of prolotherapy for lateral epicondylitis: a pilot study. Clin J Sport Med. 2008;18:248-254.

8. Visentini PJ, Khan KM, Cook JL, et al. The VISA score: an index of severity of symptoms in patients with jumper’s knee (patellar tendinosis). Victorian Institute of Sport Tendon Study Group. J Sci Med Sport. 1998;1:22-28.

Issue
The Journal of Family Practice - 61(8)
Issue
The Journal of Family Practice - 61(8)
Page Number
486-488
Page Number
486-488
Publications
Publications
Topics
Article Type
Display Headline
Injection may be the best bet for young athletes’ knee pain
Display Headline
Injection may be the best bet for young athletes’ knee pain
Sections
PURLs Copyright

Copyright © 2012 The Family Physicians Inquiries Network. All rights reserved.

Disallow All Ads
Alternative CME
Article PDF Media

Would this long-acting bronchodilator be better for your patient?

Article Type
Changed
Mon, 01/14/2019 - 13:40
Display Headline
Would this long-acting bronchodilator be better for your patient?
PRACTICE CHANGER

Consider adding tiotropium to the medication regimen of patients with moderate to very severe chronic obstructive pulmonary disease (COPD), as a multinational study found it to be more effective than salmeterol in preventing exacerbations.1

STRENGTH OF RECOMMENDATION

A: Based on one well-designed randomized controlled trial.

Vogelmeier C, Hederer B, Glaab T, et al; POET-COPD investigators. Tiotropium versus salmeterol for the prevention of exacerbations of COPD. N Engl J Med. 2011;364:1093-1103.

 

ILLUSTRATIVE CASE

A 60-year-old patient with moderate COPD and a history of frequent exacerbations comes in for a follow-up visit. She has been using albuterol and ipratropium intermittently. you want to add a longer-acting bronchodilator and wonder if tiotropium or salmeterol is more effective for reducing exacerbations.

COPD is the fourth leading cause of death in the United States.2 More than 12 million Americans have been diagnosed with COPD, and it is estimated that another 12 million would have a COPD diagnosis if all smokers older than 45 years underwent spirometry.2 The disorder accounts for some 16 million physician visits each year and costs the US health care system approximately $19 billion annually, with acute exacerbations and hospitalizations representing 58% of the total.2,3

Despite guidelines, COPD is often undertreated
One of the main goals of COPD treatment is to reduce the frequency and intensity of acute exacerbations, both to improve patients’ quality of life and reduce health care costs. The Global Initiative for Chronic Obstructive Lung Disease (GOLD) has developed guidelines for effective management of COPD, which recommend long-acting bronchodilators as first-line maintenance therapy for patients whose disease is moderate to very severe.4

Evidence suggests that physicians frequently undertreat moderate to severe COPD, however, following national guidelines only about a quarter of the time.5 This is, in part, because many clinicians doubt the efficacy of COPD treatment for improving symptoms or decreasing exacerbations.5,6 Yet studies have shown that the long-acting broncho dilators tiotropium (an anticholinergic agent) and salmeterol (a beta2-adrenergic agonist), used with or without inhaled corticosteroids, are effective in reducing the frequency of COPD exacerbations, improving quality of life and lung function, and reducing the number of hospitalizations.7-10

Long-acting bronchodilators are therefore clearly indicated but, until recently, there was little evidence as to which one is better.

STUDY SUMMARY: Tiotropium group had fewer exacerbations…

The Prevention Of Exacerbations with Tiotropium in COPD (POET-COPD) trial compared tiotropium with salmeterol for their ability to prevent exacerbations.1 This was a randomized double-blind trial of 7376 patients with moderate to very severe COPD diagnosed by spirometry. Participants were recruited from 725 medical centers in 25 countries. To be eligible, they had to be ≥40 years, with at least a 10 pack-year history of smoking, a forced expiratory volume in 1 second (FEV1) <70% predicted, an FEV1/forced vital capacity (FVC ) <70%, and at least one exacerbation in the previous year.

Patients were randomly assigned to either the tiotropium or the salmeterol group. Those on tiotropium received a daily dose of 18 mcg through a HandiHaler device, plus a placebo with a metered-dose inhaler twice a day. Patients in the other group received 50 mcg salmeterol through a metered-dose inhaler twice daily, plus a placebo with a HandiHaler once a day. These medications were in addition to patients’ current medication regimens, including inhaled corticosteroids, with this exception: Use of anticholinergics and long-acting beta-agonists was discontinued for the course of the trial.

All participants were followed for one year, with clinic visits at 2, 4, 8, and 12 months to assess for medication adherence and symptoms of exacerbation. The primary endpoint was the time to first exacerbation. This was defined as an increase in, or a new onset of, more than one symptom of COPD (ie, cough, sputum production, wheezing, dyspnea, and chest tightness), with at least one symptom lasting ≥3 days and leading to treatment with glucocorticoids and/or antibiotics, or hospitalization. Secondary outcomes were times to first moderate and severe exacerbations and use of steroids and antibiotics.

There were significant differences in several outcomes. The time to first exacerbation was 187 days for tiotropium vs 145 days for salmeterol, a difference of 42 days (hazard ratio [HR]=0.83; 95% confidence interval [CI], 0.77-0.90; P<.001). In addition, tiotropium reduced the annual number of exacerbations compared with salmeterol (rate ratio=0.89; 95% CI, 0.83-0.96; P=.002), with a number needed to treat (NNT) of 24 patients to prevent one moderate to severe exacerbation per year.

 

 

 

…and used fewer drugs

Compared with salmeterol, there was a 14% reduction in risk of a moderate exacerbation associated with tiotropium (HR=0.86; 95% CI, 0.79-0.93; P<.001; NNT=32) and a 28% reduction in risk of a severe exacerbation (HR=0.72; 95% CI, 0.61-0.85; P<.001; NNT=48). In addition, the tiotropium group had a 23% risk reduction in the use of systemic glucocorticoids (HR=0.77; 95% CI, 0.69-0.85; P<.001; NNT=26) compared with the salmeterol group, and a 15% risk reduction in the use of antibiotics (HR=0.85; 95% CI, 0.78-0.92; P<0.001; NNT=31). The difference in reduction in death rates between the 2 groups was not statistically significant.

The observed differences were consistent across all major subgroups (age, sex, smoking status, and severity of COPD) of patients studied. Interestingly, patients with low BMI or very severe COPD appeared to benefit the most from tiotropium.

WHAT’S NEW: The difference between 2 agents is clear

Although national guidelines recommend long-acting bronchodilators for COPD that is moderate or worse, there have been few data to guide clinicians in determining which one to use. The findings of this study suggest that tiotropium should be our first choice. Tiotropium’s once-a-day dosing is an additional benefit, as patients using it will likely have better compliance than those using twice-daily salmeterol. The data may also prompt development of a once-daily inhaled corticosteroid/ long-acting anticholinergic combination.

CAVEATS: Cost, funding source

Cost may be an issue. Spiriva and Serevent, the brand names for tiotropium and salmeterol, respectively, are second-tier medications on several formularies, and tiotropium is about 45% more expensive (tiotropium=$262, salmeterol=$181 for one month’s supply; www.drugstore.com, accessed January 19, 2012). There are also several long-acting beta-agonists in development that will be dosed once daily; once they’re approved, tiotropium’s once-a-day dosing may no longer be seen as an advantage.

It is also worth noting that this trial was supported by Boehringer Ingelheim and Pfizer, which jointly market Spiriva.

Finally, smoking must be addressed. Strongly encouraging patients to kick the habit is still the most important intervention we can make in helping to improve the quality of life, and survival, of patients with COPD.

CHALLENGES TO IMPLEMENTATION: COPD guidelines need updating

There are no major challenges to incorporating this recommendation into clinical practice; the key challenge lies in diagnosing COPD and adequately monitoring and helping patients manage the disease.

Current guidelines do not distinguish between the efficacy of long-acting bronchodilators, but findings from this study are important enough to change future versions of national guidelines. The GOLD committee is due to release a new guideline report soon, and will likely update its recommendations at that time.

Acknowledgement

The PURLs Surveillance System is supported in part by Grant Number UL1RR024999 from the National Center for Research Resources, a Clinical Translational Science Award to the University of Chicago. The content is solely the responsibility of the authors and does not necessarily represent the official views of the National Center for Research Resources or the National Institutes of Health.

Click here to view PURL METHODOLOGY

References

1. Vogelmeier C, Hederer B, Glaab T, et al. POET-COPD investigators. Tiotropium versus salmeterol for the prevention of exacerbations of COPD. N Engl J Med. 2011;364:1093-1103.

2. National Heart, Lung, and Blood Institute. Morbidity and mortality: 2009 chart book on cardiovascular, lung, and blood diseases. Available at: http://www.nhlbi.nih.gov/resources/docs/04chtbk.pdf. Accessed October 1, 2011.

3. Miravitlles M, Murio C, Guerrero T, et al. DAFNE Study Group. Pharmacoeconomic evaluation of acute exacerbations of chronic bronchitis and COPD. Chest. 2002;121:1449-1455.

4. Global Initiative for Chronic Obstructive Lung Disease. Global strategy for the diagnosis, management, and prevention of chronic obstructive pulmonary disease. Updated 2010. Available at: http://www.goldcopd.org. Accessed October 1, 2011.

5. Salinas GD, Williamson JC, Kalhan R, et al. Barriers to adherence to chronic obstructive pulmonary disease guidelines by primary care physicians. Int J Chron Obstruct Pulmon Dis. 2011;6:171-179.

6. Yawn BP, Wollan PC. Knowledge and attitudes of family physicians coming to COPD continuing medical education. Int J Chron Obstruct Pulmon Dis. 2008;3:311-318.

7. Calverly PMA, Anderson JA, Celli B, et al. Salmeterol and fluticasone propionate and survival in chronic obstructive pulmonary disease. N Engl J Med. 2007;356:775-789.

8. Casaburi R, Mahler DA, Jones PW, et al. A long-term evaluation of once-daily inhaled tiotropium in chronic obstructive pulmonary disease. Eur Respir J. 2002;19:217-224.

9. Donahue JF, van Noord JA, Bateman ED, et al. A 6-month, placebo-controlled study comparing lung function and health status changes in COPD patients treated with tiotropium or salmeterol. Chest. 2002;122:47-55.

10. Tashkin DP, Celli B, Senn S, et al. A 4-year trial of tiotropium in chronic obstructive pulmonary disease. N Engl J Med. 2008;359:1543-1554.

Article PDF
Author and Disclosure Information

Mark Gwynne, DO
Department of Family Medicine, University of North Carolina at Chapel Hill

Anne Mounsey, MD
Department of Family Medicine, University of North Carolina at Chapel Hill

PURLs EDITOR
John Hickner, MD, MSc
Department of Family, Medicine, Cleveland Clinic

Issue
The Journal of Family Practice - 61(02)
Publications
Topics
Page Number
94-96
Sections
Author and Disclosure Information

Mark Gwynne, DO
Department of Family Medicine, University of North Carolina at Chapel Hill

Anne Mounsey, MD
Department of Family Medicine, University of North Carolina at Chapel Hill

PURLs EDITOR
John Hickner, MD, MSc
Department of Family, Medicine, Cleveland Clinic

Author and Disclosure Information

Mark Gwynne, DO
Department of Family Medicine, University of North Carolina at Chapel Hill

Anne Mounsey, MD
Department of Family Medicine, University of North Carolina at Chapel Hill

PURLs EDITOR
John Hickner, MD, MSc
Department of Family, Medicine, Cleveland Clinic

Article PDF
Article PDF
PRACTICE CHANGER

Consider adding tiotropium to the medication regimen of patients with moderate to very severe chronic obstructive pulmonary disease (COPD), as a multinational study found it to be more effective than salmeterol in preventing exacerbations.1

STRENGTH OF RECOMMENDATION

A: Based on one well-designed randomized controlled trial.

Vogelmeier C, Hederer B, Glaab T, et al; POET-COPD investigators. Tiotropium versus salmeterol for the prevention of exacerbations of COPD. N Engl J Med. 2011;364:1093-1103.

 

ILLUSTRATIVE CASE

A 60-year-old patient with moderate COPD and a history of frequent exacerbations comes in for a follow-up visit. She has been using albuterol and ipratropium intermittently. you want to add a longer-acting bronchodilator and wonder if tiotropium or salmeterol is more effective for reducing exacerbations.

COPD is the fourth leading cause of death in the United States.2 More than 12 million Americans have been diagnosed with COPD, and it is estimated that another 12 million would have a COPD diagnosis if all smokers older than 45 years underwent spirometry.2 The disorder accounts for some 16 million physician visits each year and costs the US health care system approximately $19 billion annually, with acute exacerbations and hospitalizations representing 58% of the total.2,3

Despite guidelines, COPD is often undertreated
One of the main goals of COPD treatment is to reduce the frequency and intensity of acute exacerbations, both to improve patients’ quality of life and reduce health care costs. The Global Initiative for Chronic Obstructive Lung Disease (GOLD) has developed guidelines for effective management of COPD, which recommend long-acting bronchodilators as first-line maintenance therapy for patients whose disease is moderate to very severe.4

Evidence suggests that physicians frequently undertreat moderate to severe COPD, however, following national guidelines only about a quarter of the time.5 This is, in part, because many clinicians doubt the efficacy of COPD treatment for improving symptoms or decreasing exacerbations.5,6 Yet studies have shown that the long-acting broncho dilators tiotropium (an anticholinergic agent) and salmeterol (a beta2-adrenergic agonist), used with or without inhaled corticosteroids, are effective in reducing the frequency of COPD exacerbations, improving quality of life and lung function, and reducing the number of hospitalizations.7-10

Long-acting bronchodilators are therefore clearly indicated but, until recently, there was little evidence as to which one is better.

STUDY SUMMARY: Tiotropium group had fewer exacerbations…

The Prevention Of Exacerbations with Tiotropium in COPD (POET-COPD) trial compared tiotropium with salmeterol for their ability to prevent exacerbations.1 This was a randomized double-blind trial of 7376 patients with moderate to very severe COPD diagnosed by spirometry. Participants were recruited from 725 medical centers in 25 countries. To be eligible, they had to be ≥40 years, with at least a 10 pack-year history of smoking, a forced expiratory volume in 1 second (FEV1) <70% predicted, an FEV1/forced vital capacity (FVC ) <70%, and at least one exacerbation in the previous year.

Patients were randomly assigned to either the tiotropium or the salmeterol group. Those on tiotropium received a daily dose of 18 mcg through a HandiHaler device, plus a placebo with a metered-dose inhaler twice a day. Patients in the other group received 50 mcg salmeterol through a metered-dose inhaler twice daily, plus a placebo with a HandiHaler once a day. These medications were in addition to patients’ current medication regimens, including inhaled corticosteroids, with this exception: Use of anticholinergics and long-acting beta-agonists was discontinued for the course of the trial.

All participants were followed for one year, with clinic visits at 2, 4, 8, and 12 months to assess for medication adherence and symptoms of exacerbation. The primary endpoint was the time to first exacerbation. This was defined as an increase in, or a new onset of, more than one symptom of COPD (ie, cough, sputum production, wheezing, dyspnea, and chest tightness), with at least one symptom lasting ≥3 days and leading to treatment with glucocorticoids and/or antibiotics, or hospitalization. Secondary outcomes were times to first moderate and severe exacerbations and use of steroids and antibiotics.

There were significant differences in several outcomes. The time to first exacerbation was 187 days for tiotropium vs 145 days for salmeterol, a difference of 42 days (hazard ratio [HR]=0.83; 95% confidence interval [CI], 0.77-0.90; P<.001). In addition, tiotropium reduced the annual number of exacerbations compared with salmeterol (rate ratio=0.89; 95% CI, 0.83-0.96; P=.002), with a number needed to treat (NNT) of 24 patients to prevent one moderate to severe exacerbation per year.

 

 

 

…and used fewer drugs

Compared with salmeterol, there was a 14% reduction in risk of a moderate exacerbation associated with tiotropium (HR=0.86; 95% CI, 0.79-0.93; P<.001; NNT=32) and a 28% reduction in risk of a severe exacerbation (HR=0.72; 95% CI, 0.61-0.85; P<.001; NNT=48). In addition, the tiotropium group had a 23% risk reduction in the use of systemic glucocorticoids (HR=0.77; 95% CI, 0.69-0.85; P<.001; NNT=26) compared with the salmeterol group, and a 15% risk reduction in the use of antibiotics (HR=0.85; 95% CI, 0.78-0.92; P<0.001; NNT=31). The difference in reduction in death rates between the 2 groups was not statistically significant.

The observed differences were consistent across all major subgroups (age, sex, smoking status, and severity of COPD) of patients studied. Interestingly, patients with low BMI or very severe COPD appeared to benefit the most from tiotropium.

WHAT’S NEW: The difference between 2 agents is clear

Although national guidelines recommend long-acting bronchodilators for COPD that is moderate or worse, there have been few data to guide clinicians in determining which one to use. The findings of this study suggest that tiotropium should be our first choice. Tiotropium’s once-a-day dosing is an additional benefit, as patients using it will likely have better compliance than those using twice-daily salmeterol. The data may also prompt development of a once-daily inhaled corticosteroid/ long-acting anticholinergic combination.

CAVEATS: Cost, funding source

Cost may be an issue. Spiriva and Serevent, the brand names for tiotropium and salmeterol, respectively, are second-tier medications on several formularies, and tiotropium is about 45% more expensive (tiotropium=$262, salmeterol=$181 for one month’s supply; www.drugstore.com, accessed January 19, 2012). There are also several long-acting beta-agonists in development that will be dosed once daily; once they’re approved, tiotropium’s once-a-day dosing may no longer be seen as an advantage.

It is also worth noting that this trial was supported by Boehringer Ingelheim and Pfizer, which jointly market Spiriva.

Finally, smoking must be addressed. Strongly encouraging patients to kick the habit is still the most important intervention we can make in helping to improve the quality of life, and survival, of patients with COPD.

CHALLENGES TO IMPLEMENTATION: COPD guidelines need updating

There are no major challenges to incorporating this recommendation into clinical practice; the key challenge lies in diagnosing COPD and adequately monitoring and helping patients manage the disease.

Current guidelines do not distinguish between the efficacy of long-acting bronchodilators, but findings from this study are important enough to change future versions of national guidelines. The GOLD committee is due to release a new guideline report soon, and will likely update its recommendations at that time.

Acknowledgement

The PURLs Surveillance System is supported in part by Grant Number UL1RR024999 from the National Center for Research Resources, a Clinical Translational Science Award to the University of Chicago. The content is solely the responsibility of the authors and does not necessarily represent the official views of the National Center for Research Resources or the National Institutes of Health.

Click here to view PURL METHODOLOGY

PRACTICE CHANGER

Consider adding tiotropium to the medication regimen of patients with moderate to very severe chronic obstructive pulmonary disease (COPD), as a multinational study found it to be more effective than salmeterol in preventing exacerbations.1

STRENGTH OF RECOMMENDATION

A: Based on one well-designed randomized controlled trial.

Vogelmeier C, Hederer B, Glaab T, et al; POET-COPD investigators. Tiotropium versus salmeterol for the prevention of exacerbations of COPD. N Engl J Med. 2011;364:1093-1103.

 

ILLUSTRATIVE CASE

A 60-year-old patient with moderate COPD and a history of frequent exacerbations comes in for a follow-up visit. She has been using albuterol and ipratropium intermittently. you want to add a longer-acting bronchodilator and wonder if tiotropium or salmeterol is more effective for reducing exacerbations.

COPD is the fourth leading cause of death in the United States.2 More than 12 million Americans have been diagnosed with COPD, and it is estimated that another 12 million would have a COPD diagnosis if all smokers older than 45 years underwent spirometry.2 The disorder accounts for some 16 million physician visits each year and costs the US health care system approximately $19 billion annually, with acute exacerbations and hospitalizations representing 58% of the total.2,3

Despite guidelines, COPD is often undertreated
One of the main goals of COPD treatment is to reduce the frequency and intensity of acute exacerbations, both to improve patients’ quality of life and reduce health care costs. The Global Initiative for Chronic Obstructive Lung Disease (GOLD) has developed guidelines for effective management of COPD, which recommend long-acting bronchodilators as first-line maintenance therapy for patients whose disease is moderate to very severe.4

Evidence suggests that physicians frequently undertreat moderate to severe COPD, however, following national guidelines only about a quarter of the time.5 This is, in part, because many clinicians doubt the efficacy of COPD treatment for improving symptoms or decreasing exacerbations.5,6 Yet studies have shown that the long-acting broncho dilators tiotropium (an anticholinergic agent) and salmeterol (a beta2-adrenergic agonist), used with or without inhaled corticosteroids, are effective in reducing the frequency of COPD exacerbations, improving quality of life and lung function, and reducing the number of hospitalizations.7-10

Long-acting bronchodilators are therefore clearly indicated but, until recently, there was little evidence as to which one is better.

STUDY SUMMARY: Tiotropium group had fewer exacerbations…

The Prevention Of Exacerbations with Tiotropium in COPD (POET-COPD) trial compared tiotropium with salmeterol for their ability to prevent exacerbations.1 This was a randomized double-blind trial of 7376 patients with moderate to very severe COPD diagnosed by spirometry. Participants were recruited from 725 medical centers in 25 countries. To be eligible, they had to be ≥40 years, with at least a 10 pack-year history of smoking, a forced expiratory volume in 1 second (FEV1) <70% predicted, an FEV1/forced vital capacity (FVC ) <70%, and at least one exacerbation in the previous year.

Patients were randomly assigned to either the tiotropium or the salmeterol group. Those on tiotropium received a daily dose of 18 mcg through a HandiHaler device, plus a placebo with a metered-dose inhaler twice a day. Patients in the other group received 50 mcg salmeterol through a metered-dose inhaler twice daily, plus a placebo with a HandiHaler once a day. These medications were in addition to patients’ current medication regimens, including inhaled corticosteroids, with this exception: Use of anticholinergics and long-acting beta-agonists was discontinued for the course of the trial.

All participants were followed for one year, with clinic visits at 2, 4, 8, and 12 months to assess for medication adherence and symptoms of exacerbation. The primary endpoint was the time to first exacerbation. This was defined as an increase in, or a new onset of, more than one symptom of COPD (ie, cough, sputum production, wheezing, dyspnea, and chest tightness), with at least one symptom lasting ≥3 days and leading to treatment with glucocorticoids and/or antibiotics, or hospitalization. Secondary outcomes were times to first moderate and severe exacerbations and use of steroids and antibiotics.

There were significant differences in several outcomes. The time to first exacerbation was 187 days for tiotropium vs 145 days for salmeterol, a difference of 42 days (hazard ratio [HR]=0.83; 95% confidence interval [CI], 0.77-0.90; P<.001). In addition, tiotropium reduced the annual number of exacerbations compared with salmeterol (rate ratio=0.89; 95% CI, 0.83-0.96; P=.002), with a number needed to treat (NNT) of 24 patients to prevent one moderate to severe exacerbation per year.

 

 

 

…and used fewer drugs

Compared with salmeterol, there was a 14% reduction in risk of a moderate exacerbation associated with tiotropium (HR=0.86; 95% CI, 0.79-0.93; P<.001; NNT=32) and a 28% reduction in risk of a severe exacerbation (HR=0.72; 95% CI, 0.61-0.85; P<.001; NNT=48). In addition, the tiotropium group had a 23% risk reduction in the use of systemic glucocorticoids (HR=0.77; 95% CI, 0.69-0.85; P<.001; NNT=26) compared with the salmeterol group, and a 15% risk reduction in the use of antibiotics (HR=0.85; 95% CI, 0.78-0.92; P<0.001; NNT=31). The difference in reduction in death rates between the 2 groups was not statistically significant.

The observed differences were consistent across all major subgroups (age, sex, smoking status, and severity of COPD) of patients studied. Interestingly, patients with low BMI or very severe COPD appeared to benefit the most from tiotropium.

WHAT’S NEW: The difference between 2 agents is clear

Although national guidelines recommend long-acting bronchodilators for COPD that is moderate or worse, there have been few data to guide clinicians in determining which one to use. The findings of this study suggest that tiotropium should be our first choice. Tiotropium’s once-a-day dosing is an additional benefit, as patients using it will likely have better compliance than those using twice-daily salmeterol. The data may also prompt development of a once-daily inhaled corticosteroid/ long-acting anticholinergic combination.

CAVEATS: Cost, funding source

Cost may be an issue. Spiriva and Serevent, the brand names for tiotropium and salmeterol, respectively, are second-tier medications on several formularies, and tiotropium is about 45% more expensive (tiotropium=$262, salmeterol=$181 for one month’s supply; www.drugstore.com, accessed January 19, 2012). There are also several long-acting beta-agonists in development that will be dosed once daily; once they’re approved, tiotropium’s once-a-day dosing may no longer be seen as an advantage.

It is also worth noting that this trial was supported by Boehringer Ingelheim and Pfizer, which jointly market Spiriva.

Finally, smoking must be addressed. Strongly encouraging patients to kick the habit is still the most important intervention we can make in helping to improve the quality of life, and survival, of patients with COPD.

CHALLENGES TO IMPLEMENTATION: COPD guidelines need updating

There are no major challenges to incorporating this recommendation into clinical practice; the key challenge lies in diagnosing COPD and adequately monitoring and helping patients manage the disease.

Current guidelines do not distinguish between the efficacy of long-acting bronchodilators, but findings from this study are important enough to change future versions of national guidelines. The GOLD committee is due to release a new guideline report soon, and will likely update its recommendations at that time.

Acknowledgement

The PURLs Surveillance System is supported in part by Grant Number UL1RR024999 from the National Center for Research Resources, a Clinical Translational Science Award to the University of Chicago. The content is solely the responsibility of the authors and does not necessarily represent the official views of the National Center for Research Resources or the National Institutes of Health.

Click here to view PURL METHODOLOGY

References

1. Vogelmeier C, Hederer B, Glaab T, et al. POET-COPD investigators. Tiotropium versus salmeterol for the prevention of exacerbations of COPD. N Engl J Med. 2011;364:1093-1103.

2. National Heart, Lung, and Blood Institute. Morbidity and mortality: 2009 chart book on cardiovascular, lung, and blood diseases. Available at: http://www.nhlbi.nih.gov/resources/docs/04chtbk.pdf. Accessed October 1, 2011.

3. Miravitlles M, Murio C, Guerrero T, et al. DAFNE Study Group. Pharmacoeconomic evaluation of acute exacerbations of chronic bronchitis and COPD. Chest. 2002;121:1449-1455.

4. Global Initiative for Chronic Obstructive Lung Disease. Global strategy for the diagnosis, management, and prevention of chronic obstructive pulmonary disease. Updated 2010. Available at: http://www.goldcopd.org. Accessed October 1, 2011.

5. Salinas GD, Williamson JC, Kalhan R, et al. Barriers to adherence to chronic obstructive pulmonary disease guidelines by primary care physicians. Int J Chron Obstruct Pulmon Dis. 2011;6:171-179.

6. Yawn BP, Wollan PC. Knowledge and attitudes of family physicians coming to COPD continuing medical education. Int J Chron Obstruct Pulmon Dis. 2008;3:311-318.

7. Calverly PMA, Anderson JA, Celli B, et al. Salmeterol and fluticasone propionate and survival in chronic obstructive pulmonary disease. N Engl J Med. 2007;356:775-789.

8. Casaburi R, Mahler DA, Jones PW, et al. A long-term evaluation of once-daily inhaled tiotropium in chronic obstructive pulmonary disease. Eur Respir J. 2002;19:217-224.

9. Donahue JF, van Noord JA, Bateman ED, et al. A 6-month, placebo-controlled study comparing lung function and health status changes in COPD patients treated with tiotropium or salmeterol. Chest. 2002;122:47-55.

10. Tashkin DP, Celli B, Senn S, et al. A 4-year trial of tiotropium in chronic obstructive pulmonary disease. N Engl J Med. 2008;359:1543-1554.

References

1. Vogelmeier C, Hederer B, Glaab T, et al. POET-COPD investigators. Tiotropium versus salmeterol for the prevention of exacerbations of COPD. N Engl J Med. 2011;364:1093-1103.

2. National Heart, Lung, and Blood Institute. Morbidity and mortality: 2009 chart book on cardiovascular, lung, and blood diseases. Available at: http://www.nhlbi.nih.gov/resources/docs/04chtbk.pdf. Accessed October 1, 2011.

3. Miravitlles M, Murio C, Guerrero T, et al. DAFNE Study Group. Pharmacoeconomic evaluation of acute exacerbations of chronic bronchitis and COPD. Chest. 2002;121:1449-1455.

4. Global Initiative for Chronic Obstructive Lung Disease. Global strategy for the diagnosis, management, and prevention of chronic obstructive pulmonary disease. Updated 2010. Available at: http://www.goldcopd.org. Accessed October 1, 2011.

5. Salinas GD, Williamson JC, Kalhan R, et al. Barriers to adherence to chronic obstructive pulmonary disease guidelines by primary care physicians. Int J Chron Obstruct Pulmon Dis. 2011;6:171-179.

6. Yawn BP, Wollan PC. Knowledge and attitudes of family physicians coming to COPD continuing medical education. Int J Chron Obstruct Pulmon Dis. 2008;3:311-318.

7. Calverly PMA, Anderson JA, Celli B, et al. Salmeterol and fluticasone propionate and survival in chronic obstructive pulmonary disease. N Engl J Med. 2007;356:775-789.

8. Casaburi R, Mahler DA, Jones PW, et al. A long-term evaluation of once-daily inhaled tiotropium in chronic obstructive pulmonary disease. Eur Respir J. 2002;19:217-224.

9. Donahue JF, van Noord JA, Bateman ED, et al. A 6-month, placebo-controlled study comparing lung function and health status changes in COPD patients treated with tiotropium or salmeterol. Chest. 2002;122:47-55.

10. Tashkin DP, Celli B, Senn S, et al. A 4-year trial of tiotropium in chronic obstructive pulmonary disease. N Engl J Med. 2008;359:1543-1554.

Issue
The Journal of Family Practice - 61(02)
Issue
The Journal of Family Practice - 61(02)
Page Number
94-96
Page Number
94-96
Publications
Publications
Topics
Article Type
Display Headline
Would this long-acting bronchodilator be better for your patient?
Display Headline
Would this long-acting bronchodilator be better for your patient?
Sections
PURLs Copyright

Copyright © 2012 The Family Physicians Inquiries Network.
All rights reserved.

Disallow All Ads
Alternative CME
Article PDF Media

Colicky baby? Here’s a surprising remedy

Article Type
Changed
Mon, 01/14/2019 - 11:38
Display Headline
Colicky baby? Here’s a surprising remedy
PRACTICE CHANGER

Suggest that parents of colicky breastfed infants try probiotics (Lactobacillus reuteri), which can significantly reduce daily crying time with no adverse effects.1

STRENGTH OF RECOMMENDATION:

A: A good-quality randomized controlled trial (RCT).

Savino F, Cordisco L, Tarasco V, et al. Lactobacillus reuteri DSM 17938 in infantile colic: a randomized, double-blind, placebo-controlled trial. Pediatrics. 2010;126:e526-e533.

 

ILLUSTRATIVE CASE

The parents of an otherwise healthy 10-week-old breastfed baby girl bring her in for the second time in 2 weeks because of her in-consolable crying. Physical examination and work-up remain normal, and you again diagnose colic. What can you suggest to help decrease the baby’s crying, other than the usual dietary advice?

Colic affects up to 28% of infants, causing considerable stress for parents and for their health care providers.2 Indeed, in the first 3 months of a baby’s life, crying is the No. 1 reason for pediatric visits.3 Parents often perceive—incorrectly—that the inconsolable crying is either a sign of serious illness or a result of poor parenting skills.4

A distressing problem, with few remedies
Despite the ubiquity and frustration that accompany colic, its exact etiology remains unclear and effective treatments remain elusive. With very little quality evidence to support interventions for colicky infants, we often have nothing more than grandmotherly advice to offer parents of babies with this vexing condition.

Current guidelines recommend only one strategy for breastfeeding mothers: a lowallergen diet.5 However, recent studies suggest that low counts of intestinal lactobacilli may play a role in colic and have documented improved symptoms after treatment with lactobacilli compared with treatment with simethicone.6-8 Infant formulas that contain probiotics are now available, as a result.

Although the results of the recent studies have been promising, they were not double-blinded or placebo-controlled. The study detailed here is the first to provide compelling evidence for a safe intervention for colicky breastfed infants.

STUDY SUMMARY: Lactobacilli cut crying time

In a randomized, double-blinded, placebo-controlled trial, Savino et al randomly assigned 50 exclusively breastfed colicky infants ages 2 to 16 weeks to receive either L reuteri DSM 17 938 (108 colony-forming units) or placebo daily for 21 days. Diagnosis of colic was made according to the modified Wessel’s criteria—fussy crying for ≥3 hours per day for ≥3 days per week in the week before enrollment. Their mothers were told to avoid ingesting cow’s milk during the course of the study.

Term infants adequate for gestational age were eligible for inclusion in the study. Exclusion criteria included evidence of chronic illness or gastrointestinal disorders, any intake of probiotics and/or antibiotics in the week preceding recruitment, and any formula feeding.

Parents and providers were blinded during the study, and they reported daily crying time, stool characteristics, adverse events, and growth patterns. An identical looking and tasting triglyceride oil without live bacteria was used in the placebo group. Each infant received 5 drops of L reuteri or placebo each morning 30 minutes before the morning feeding.

The primary outcome was a reduction of average crying time to <3 hours a day by Day 21. A secondary outcome was the number of infants in each group who experienced a 50% decrease in the daily average crying time from baseline on Days 7, 14, and 21 of the study.

Initially, the babies were divided equally between the control and intervention groups, but 4 participants in the control group were later excluded from analysis (1 had fever, 1 had reflux, and the parents of 2 infants did not complete the analysis).

At the time of enrollment, no significant differences were noted between the intervention and control groups regarding type of delivery, sex, age, family history of gastrointestinal disorders, growth parameters, and median daily crying time (370 minutes for the probiotics group vs 300 minutes for the placebo group).

 

 

 

By Day 21, the number of infants with crying times >3 hours was significantly lower in the treatment group compared with the placebo group (4 vs 12, respectively; P=.009). At all stages in the study, crying time for those in the treatment group was less than in the placebo group; median crying times for the intervention group were 95, 60, and 35 minutes per day, vs 185, 150, and 90 minutes for the controls, at 7, 14, and 21 days, respectively.

The number of infants with a 50% reduction in crying time was significantly greater in the treatment group than in the placebo group on Days 7, 14, and 21 (TABLE), although both groups saw an increase in the number of children whose average crying time had dropped by 50% as time went by. The number needed to treat to reduce crying time by 50% on Day 21 was 4.

There were no differences between the groups in growth, weight gain, frequency of stools, or incidence of regurgitation or constipation. No adverse events related to the treatment were reported.

TABLE
Babies respond* to probiotics

Day of studyL reuteri n=25 (%)Placebo n=21 (%)P value
720 (80)8 (36).006
1424 (96)13 (62).007
2124 (96)15 (71).036
*An infant with a decrease in daily average crying time of 50% from baseline was defined as a responder.

WHAT’S NEW: We have an evidence-based remedy that’s safe and effective

This study represents the first randomized, double-blinded, placebo-controlled investigation of probiotics to reduce infant colic. The researchers’ focus on patient-oriented outcomes and their solid study design move the notion of probiotics’ efficacy from conjecture to evidence. Furthermore, the study documents the safety of the intervention in the treatment group. This study increases our evidence-based armamentarium for treating colic, and family physicians should consider prescribing probiotics for healthy breastfed infants with colic.

CAVEATS: Will it work for bottle-fed infants?

This study was conducted in exclusively breast-fed, healthy infants whose mothers avoided dietary cow’s milk, which limits its applicability to a more general infant population. The study was funded by the makers of the probiotic, but the rigorous study design with random allocation, double-blind design, and intention-to-treat analysis makes bias unlikely. Although no adverse effects were reported during this study, there is little available evidence about the long-term effects of probiotics in infants. As L reuteri are naturally occurring gut bacteria, however, it seems unlikely that it would be harmful in the long term.

CHALLENGES TO IMPLEMENTATION: Parents will need to purchase the probiotics

As with any non–FDA-regulated product, it will be important to guide patients toward reputable manufacturers to ensure homogeneity of dosing. A 29-day supply of BioGaia probiotic drops (100 million units once a day), which costs $37 according to the manufacturer’s Web site, http://www.biogaia.com/consumer/biogaia-probiotic-products/probiotic-drops, should be affordable for most parents. Otherwise, little stands in the way of using this therapy to reduce the crying and subsequent stress associated with infant colic.

Acknowledgement

The PURLs Surveillance System is supported in part by Grant Number UL1RR024999 from the National Center for Research Resources, a Clinical Translational Science Award to the University of Chicago. The content is solely the responsibility of the authors and does not necessarily represent the official views of the National Center for Research Resources or the National Institutes of Health.

Click here to view PURL METHODOLOGY

References

1. Savino F, Cordisco L, Tarasco V, et al. Lactobacillus reuteri DSM 17938 in infantile colic: a randomized, double-blind, placebo-controlled trial. Pediatrics. 2010;126:e526-e533.

2. Keefe MR, Kajrlsen KA, Lobo ML, et al. Reducing parenting stress in families with irritable infants. Nurs Res. 2006;55:198-205

3. Forsyth BWC, McCarthy PL, Leventhal JM. Problems of early infancy, formula changes, and mothers’ beliefs about their infants. J Pediatr. 1985;106:1012-1017.

4. Lehtonen LA, Rautava PT. Infantile colic: natural history and treatment. Curr Probl Pediatr. 1996;26:79.-

5. Cincinnati Children’s Hospital Medical Center. Best evidence statement (BESt). Maternal dietary antigen avoidance in lactation. Available at: http://www.cincinnatichildrens.org/svc/alpha/h/health-policy/best.htm. Accessed December 14, 2010.

6. Savino F, Cresi F, Pautasso S, et al. Intestinal microflora in breast-fed colicky and non-colicky infants. Acta Paediatr. 2004;93:825-829.

7. Savino F, Bailo E, Oggero R, et al. Bacterial counts of intestinal Lactobacillus species in infants with colic. Pediatr Allergy Immunol. 2005;16:72-75.

8. Savino F, Pelle E, Palumeri E, Oggero R, et al. Lactobacillus reuteri (American type culture collection strain 55 730) versus simethicone in the treatment of infantile colic: a prospective randomized study. Pediatrics. 2007;119:e124-e130.

Article PDF
Author and Disclosure Information

Thomas Koonce, MD, MPH
Department of Family Medicine, The University of North Carolina at Chapel Hill

Anne Mounsey, MD
Department of Family Medicine, The University of North Carolina at Chapel Hill

Kate Rowland, MD
Department of Family Medicine, University of Chicago

PURLs EDITOR
John Hickner, MD, MSc
Cleveland Clinic

Issue
The Journal of Family Practice - 60(1)
Publications
Topics
Page Number
34-36
Legacy Keywords
Thomas Koonce; PURLs; colicky baby; probiotics; breastfed infant; lactobacilli
Sections
Author and Disclosure Information

Thomas Koonce, MD, MPH
Department of Family Medicine, The University of North Carolina at Chapel Hill

Anne Mounsey, MD
Department of Family Medicine, The University of North Carolina at Chapel Hill

Kate Rowland, MD
Department of Family Medicine, University of Chicago

PURLs EDITOR
John Hickner, MD, MSc
Cleveland Clinic

Author and Disclosure Information

Thomas Koonce, MD, MPH
Department of Family Medicine, The University of North Carolina at Chapel Hill

Anne Mounsey, MD
Department of Family Medicine, The University of North Carolina at Chapel Hill

Kate Rowland, MD
Department of Family Medicine, University of Chicago

PURLs EDITOR
John Hickner, MD, MSc
Cleveland Clinic

Article PDF
Article PDF
PRACTICE CHANGER

Suggest that parents of colicky breastfed infants try probiotics (Lactobacillus reuteri), which can significantly reduce daily crying time with no adverse effects.1

STRENGTH OF RECOMMENDATION:

A: A good-quality randomized controlled trial (RCT).

Savino F, Cordisco L, Tarasco V, et al. Lactobacillus reuteri DSM 17938 in infantile colic: a randomized, double-blind, placebo-controlled trial. Pediatrics. 2010;126:e526-e533.

 

ILLUSTRATIVE CASE

The parents of an otherwise healthy 10-week-old breastfed baby girl bring her in for the second time in 2 weeks because of her in-consolable crying. Physical examination and work-up remain normal, and you again diagnose colic. What can you suggest to help decrease the baby’s crying, other than the usual dietary advice?

Colic affects up to 28% of infants, causing considerable stress for parents and for their health care providers.2 Indeed, in the first 3 months of a baby’s life, crying is the No. 1 reason for pediatric visits.3 Parents often perceive—incorrectly—that the inconsolable crying is either a sign of serious illness or a result of poor parenting skills.4

A distressing problem, with few remedies
Despite the ubiquity and frustration that accompany colic, its exact etiology remains unclear and effective treatments remain elusive. With very little quality evidence to support interventions for colicky infants, we often have nothing more than grandmotherly advice to offer parents of babies with this vexing condition.

Current guidelines recommend only one strategy for breastfeeding mothers: a lowallergen diet.5 However, recent studies suggest that low counts of intestinal lactobacilli may play a role in colic and have documented improved symptoms after treatment with lactobacilli compared with treatment with simethicone.6-8 Infant formulas that contain probiotics are now available, as a result.

Although the results of the recent studies have been promising, they were not double-blinded or placebo-controlled. The study detailed here is the first to provide compelling evidence for a safe intervention for colicky breastfed infants.

STUDY SUMMARY: Lactobacilli cut crying time

In a randomized, double-blinded, placebo-controlled trial, Savino et al randomly assigned 50 exclusively breastfed colicky infants ages 2 to 16 weeks to receive either L reuteri DSM 17 938 (108 colony-forming units) or placebo daily for 21 days. Diagnosis of colic was made according to the modified Wessel’s criteria—fussy crying for ≥3 hours per day for ≥3 days per week in the week before enrollment. Their mothers were told to avoid ingesting cow’s milk during the course of the study.

Term infants adequate for gestational age were eligible for inclusion in the study. Exclusion criteria included evidence of chronic illness or gastrointestinal disorders, any intake of probiotics and/or antibiotics in the week preceding recruitment, and any formula feeding.

Parents and providers were blinded during the study, and they reported daily crying time, stool characteristics, adverse events, and growth patterns. An identical looking and tasting triglyceride oil without live bacteria was used in the placebo group. Each infant received 5 drops of L reuteri or placebo each morning 30 minutes before the morning feeding.

The primary outcome was a reduction of average crying time to <3 hours a day by Day 21. A secondary outcome was the number of infants in each group who experienced a 50% decrease in the daily average crying time from baseline on Days 7, 14, and 21 of the study.

Initially, the babies were divided equally between the control and intervention groups, but 4 participants in the control group were later excluded from analysis (1 had fever, 1 had reflux, and the parents of 2 infants did not complete the analysis).

At the time of enrollment, no significant differences were noted between the intervention and control groups regarding type of delivery, sex, age, family history of gastrointestinal disorders, growth parameters, and median daily crying time (370 minutes for the probiotics group vs 300 minutes for the placebo group).

 

 

 

By Day 21, the number of infants with crying times >3 hours was significantly lower in the treatment group compared with the placebo group (4 vs 12, respectively; P=.009). At all stages in the study, crying time for those in the treatment group was less than in the placebo group; median crying times for the intervention group were 95, 60, and 35 minutes per day, vs 185, 150, and 90 minutes for the controls, at 7, 14, and 21 days, respectively.

The number of infants with a 50% reduction in crying time was significantly greater in the treatment group than in the placebo group on Days 7, 14, and 21 (TABLE), although both groups saw an increase in the number of children whose average crying time had dropped by 50% as time went by. The number needed to treat to reduce crying time by 50% on Day 21 was 4.

There were no differences between the groups in growth, weight gain, frequency of stools, or incidence of regurgitation or constipation. No adverse events related to the treatment were reported.

TABLE
Babies respond* to probiotics

Day of studyL reuteri n=25 (%)Placebo n=21 (%)P value
720 (80)8 (36).006
1424 (96)13 (62).007
2124 (96)15 (71).036
*An infant with a decrease in daily average crying time of 50% from baseline was defined as a responder.

WHAT’S NEW: We have an evidence-based remedy that’s safe and effective

This study represents the first randomized, double-blinded, placebo-controlled investigation of probiotics to reduce infant colic. The researchers’ focus on patient-oriented outcomes and their solid study design move the notion of probiotics’ efficacy from conjecture to evidence. Furthermore, the study documents the safety of the intervention in the treatment group. This study increases our evidence-based armamentarium for treating colic, and family physicians should consider prescribing probiotics for healthy breastfed infants with colic.

CAVEATS: Will it work for bottle-fed infants?

This study was conducted in exclusively breast-fed, healthy infants whose mothers avoided dietary cow’s milk, which limits its applicability to a more general infant population. The study was funded by the makers of the probiotic, but the rigorous study design with random allocation, double-blind design, and intention-to-treat analysis makes bias unlikely. Although no adverse effects were reported during this study, there is little available evidence about the long-term effects of probiotics in infants. As L reuteri are naturally occurring gut bacteria, however, it seems unlikely that it would be harmful in the long term.

CHALLENGES TO IMPLEMENTATION: Parents will need to purchase the probiotics

As with any non–FDA-regulated product, it will be important to guide patients toward reputable manufacturers to ensure homogeneity of dosing. A 29-day supply of BioGaia probiotic drops (100 million units once a day), which costs $37 according to the manufacturer’s Web site, http://www.biogaia.com/consumer/biogaia-probiotic-products/probiotic-drops, should be affordable for most parents. Otherwise, little stands in the way of using this therapy to reduce the crying and subsequent stress associated with infant colic.

Acknowledgement

The PURLs Surveillance System is supported in part by Grant Number UL1RR024999 from the National Center for Research Resources, a Clinical Translational Science Award to the University of Chicago. The content is solely the responsibility of the authors and does not necessarily represent the official views of the National Center for Research Resources or the National Institutes of Health.

Click here to view PURL METHODOLOGY

PRACTICE CHANGER

Suggest that parents of colicky breastfed infants try probiotics (Lactobacillus reuteri), which can significantly reduce daily crying time with no adverse effects.1

STRENGTH OF RECOMMENDATION:

A: A good-quality randomized controlled trial (RCT).

Savino F, Cordisco L, Tarasco V, et al. Lactobacillus reuteri DSM 17938 in infantile colic: a randomized, double-blind, placebo-controlled trial. Pediatrics. 2010;126:e526-e533.

 

ILLUSTRATIVE CASE

The parents of an otherwise healthy 10-week-old breastfed baby girl bring her in for the second time in 2 weeks because of her in-consolable crying. Physical examination and work-up remain normal, and you again diagnose colic. What can you suggest to help decrease the baby’s crying, other than the usual dietary advice?

Colic affects up to 28% of infants, causing considerable stress for parents and for their health care providers.2 Indeed, in the first 3 months of a baby’s life, crying is the No. 1 reason for pediatric visits.3 Parents often perceive—incorrectly—that the inconsolable crying is either a sign of serious illness or a result of poor parenting skills.4

A distressing problem, with few remedies
Despite the ubiquity and frustration that accompany colic, its exact etiology remains unclear and effective treatments remain elusive. With very little quality evidence to support interventions for colicky infants, we often have nothing more than grandmotherly advice to offer parents of babies with this vexing condition.

Current guidelines recommend only one strategy for breastfeeding mothers: a lowallergen diet.5 However, recent studies suggest that low counts of intestinal lactobacilli may play a role in colic and have documented improved symptoms after treatment with lactobacilli compared with treatment with simethicone.6-8 Infant formulas that contain probiotics are now available, as a result.

Although the results of the recent studies have been promising, they were not double-blinded or placebo-controlled. The study detailed here is the first to provide compelling evidence for a safe intervention for colicky breastfed infants.

STUDY SUMMARY: Lactobacilli cut crying time

In a randomized, double-blinded, placebo-controlled trial, Savino et al randomly assigned 50 exclusively breastfed colicky infants ages 2 to 16 weeks to receive either L reuteri DSM 17 938 (108 colony-forming units) or placebo daily for 21 days. Diagnosis of colic was made according to the modified Wessel’s criteria—fussy crying for ≥3 hours per day for ≥3 days per week in the week before enrollment. Their mothers were told to avoid ingesting cow’s milk during the course of the study.

Term infants adequate for gestational age were eligible for inclusion in the study. Exclusion criteria included evidence of chronic illness or gastrointestinal disorders, any intake of probiotics and/or antibiotics in the week preceding recruitment, and any formula feeding.

Parents and providers were blinded during the study, and they reported daily crying time, stool characteristics, adverse events, and growth patterns. An identical looking and tasting triglyceride oil without live bacteria was used in the placebo group. Each infant received 5 drops of L reuteri or placebo each morning 30 minutes before the morning feeding.

The primary outcome was a reduction of average crying time to <3 hours a day by Day 21. A secondary outcome was the number of infants in each group who experienced a 50% decrease in the daily average crying time from baseline on Days 7, 14, and 21 of the study.

Initially, the babies were divided equally between the control and intervention groups, but 4 participants in the control group were later excluded from analysis (1 had fever, 1 had reflux, and the parents of 2 infants did not complete the analysis).

At the time of enrollment, no significant differences were noted between the intervention and control groups regarding type of delivery, sex, age, family history of gastrointestinal disorders, growth parameters, and median daily crying time (370 minutes for the probiotics group vs 300 minutes for the placebo group).

 

 

 

By Day 21, the number of infants with crying times >3 hours was significantly lower in the treatment group compared with the placebo group (4 vs 12, respectively; P=.009). At all stages in the study, crying time for those in the treatment group was less than in the placebo group; median crying times for the intervention group were 95, 60, and 35 minutes per day, vs 185, 150, and 90 minutes for the controls, at 7, 14, and 21 days, respectively.

The number of infants with a 50% reduction in crying time was significantly greater in the treatment group than in the placebo group on Days 7, 14, and 21 (TABLE), although both groups saw an increase in the number of children whose average crying time had dropped by 50% as time went by. The number needed to treat to reduce crying time by 50% on Day 21 was 4.

There were no differences between the groups in growth, weight gain, frequency of stools, or incidence of regurgitation or constipation. No adverse events related to the treatment were reported.

TABLE
Babies respond* to probiotics

Day of studyL reuteri n=25 (%)Placebo n=21 (%)P value
720 (80)8 (36).006
1424 (96)13 (62).007
2124 (96)15 (71).036
*An infant with a decrease in daily average crying time of 50% from baseline was defined as a responder.

WHAT’S NEW: We have an evidence-based remedy that’s safe and effective

This study represents the first randomized, double-blinded, placebo-controlled investigation of probiotics to reduce infant colic. The researchers’ focus on patient-oriented outcomes and their solid study design move the notion of probiotics’ efficacy from conjecture to evidence. Furthermore, the study documents the safety of the intervention in the treatment group. This study increases our evidence-based armamentarium for treating colic, and family physicians should consider prescribing probiotics for healthy breastfed infants with colic.

CAVEATS: Will it work for bottle-fed infants?

This study was conducted in exclusively breast-fed, healthy infants whose mothers avoided dietary cow’s milk, which limits its applicability to a more general infant population. The study was funded by the makers of the probiotic, but the rigorous study design with random allocation, double-blind design, and intention-to-treat analysis makes bias unlikely. Although no adverse effects were reported during this study, there is little available evidence about the long-term effects of probiotics in infants. As L reuteri are naturally occurring gut bacteria, however, it seems unlikely that it would be harmful in the long term.

CHALLENGES TO IMPLEMENTATION: Parents will need to purchase the probiotics

As with any non–FDA-regulated product, it will be important to guide patients toward reputable manufacturers to ensure homogeneity of dosing. A 29-day supply of BioGaia probiotic drops (100 million units once a day), which costs $37 according to the manufacturer’s Web site, http://www.biogaia.com/consumer/biogaia-probiotic-products/probiotic-drops, should be affordable for most parents. Otherwise, little stands in the way of using this therapy to reduce the crying and subsequent stress associated with infant colic.

Acknowledgement

The PURLs Surveillance System is supported in part by Grant Number UL1RR024999 from the National Center for Research Resources, a Clinical Translational Science Award to the University of Chicago. The content is solely the responsibility of the authors and does not necessarily represent the official views of the National Center for Research Resources or the National Institutes of Health.

Click here to view PURL METHODOLOGY

References

1. Savino F, Cordisco L, Tarasco V, et al. Lactobacillus reuteri DSM 17938 in infantile colic: a randomized, double-blind, placebo-controlled trial. Pediatrics. 2010;126:e526-e533.

2. Keefe MR, Kajrlsen KA, Lobo ML, et al. Reducing parenting stress in families with irritable infants. Nurs Res. 2006;55:198-205

3. Forsyth BWC, McCarthy PL, Leventhal JM. Problems of early infancy, formula changes, and mothers’ beliefs about their infants. J Pediatr. 1985;106:1012-1017.

4. Lehtonen LA, Rautava PT. Infantile colic: natural history and treatment. Curr Probl Pediatr. 1996;26:79.-

5. Cincinnati Children’s Hospital Medical Center. Best evidence statement (BESt). Maternal dietary antigen avoidance in lactation. Available at: http://www.cincinnatichildrens.org/svc/alpha/h/health-policy/best.htm. Accessed December 14, 2010.

6. Savino F, Cresi F, Pautasso S, et al. Intestinal microflora in breast-fed colicky and non-colicky infants. Acta Paediatr. 2004;93:825-829.

7. Savino F, Bailo E, Oggero R, et al. Bacterial counts of intestinal Lactobacillus species in infants with colic. Pediatr Allergy Immunol. 2005;16:72-75.

8. Savino F, Pelle E, Palumeri E, Oggero R, et al. Lactobacillus reuteri (American type culture collection strain 55 730) versus simethicone in the treatment of infantile colic: a prospective randomized study. Pediatrics. 2007;119:e124-e130.

References

1. Savino F, Cordisco L, Tarasco V, et al. Lactobacillus reuteri DSM 17938 in infantile colic: a randomized, double-blind, placebo-controlled trial. Pediatrics. 2010;126:e526-e533.

2. Keefe MR, Kajrlsen KA, Lobo ML, et al. Reducing parenting stress in families with irritable infants. Nurs Res. 2006;55:198-205

3. Forsyth BWC, McCarthy PL, Leventhal JM. Problems of early infancy, formula changes, and mothers’ beliefs about their infants. J Pediatr. 1985;106:1012-1017.

4. Lehtonen LA, Rautava PT. Infantile colic: natural history and treatment. Curr Probl Pediatr. 1996;26:79.-

5. Cincinnati Children’s Hospital Medical Center. Best evidence statement (BESt). Maternal dietary antigen avoidance in lactation. Available at: http://www.cincinnatichildrens.org/svc/alpha/h/health-policy/best.htm. Accessed December 14, 2010.

6. Savino F, Cresi F, Pautasso S, et al. Intestinal microflora in breast-fed colicky and non-colicky infants. Acta Paediatr. 2004;93:825-829.

7. Savino F, Bailo E, Oggero R, et al. Bacterial counts of intestinal Lactobacillus species in infants with colic. Pediatr Allergy Immunol. 2005;16:72-75.

8. Savino F, Pelle E, Palumeri E, Oggero R, et al. Lactobacillus reuteri (American type culture collection strain 55 730) versus simethicone in the treatment of infantile colic: a prospective randomized study. Pediatrics. 2007;119:e124-e130.

Issue
The Journal of Family Practice - 60(1)
Issue
The Journal of Family Practice - 60(1)
Page Number
34-36
Page Number
34-36
Publications
Publications
Topics
Article Type
Display Headline
Colicky baby? Here’s a surprising remedy
Display Headline
Colicky baby? Here’s a surprising remedy
Legacy Keywords
Thomas Koonce; PURLs; colicky baby; probiotics; breastfed infant; lactobacilli
Legacy Keywords
Thomas Koonce; PURLs; colicky baby; probiotics; breastfed infant; lactobacilli
Sections
PURLs Copyright

Copyright © 2011 The Family Physicians Inquiries Network.
All rights reserved.

Disallow All Ads
Alternative CME
Article PDF Media

What is the role of prokinetic agents for constipation?

Article Type
Changed
Mon, 01/14/2019 - 11:25
Display Headline
What is the role of prokinetic agents for constipation?
EVIDENCE-BASED ANSWER

Erythromycin has a limited role in treating pediatric patients (strength of recommendation [SOR]: B, limited-quality, patient-oriented evidence). Tegaserod and cisapride are the only prokinetic agents available for constipated adults (SOR: A, consistent, good-quality, patient-oriented evidence for tegaserod; SOR: B, for cisapride), but cardiovascular risk restricts prescribing of both medications.

Evidence summary

Prokinetic agents promote transit of intestinal contents by increasing the frequency or strength of small intestine contractions. Available prokinetics include erythromycin and metoclopramide. Metoclopramide has been tested only for upper gastrointestinal mobility. The only randomized controlled trials (RCTs) of erythromycin for constipation have been conducted in children. Cisapride and tegaserod have been withdrawn from general use because of adverse side effects. The TABLE summarizes the available data.

TABLE
Prokinetics for constipation: What the research tells us

DRUGDESIGN (N)DOSEOUTCOMENNT
Erythromycin estolate1Crossover children
(14)
20 mg/kg/day divided qid†Constipation and laxative use10
Cisapride5RCT adults
(69)
5-10 mg tidü Spontaneous BM
† Abdominal pain
4
Cisapride6RCT adults
(82)
5-10 mg tidAbdominal pain and constipation, drug=placeboN/A
Tegaserod7RCT adults
(1348)
2 mg or 6 mg bid† Constipation
ü Spontaneous BM
6 (2 mg)
5 (6 mg)
Tegaserod8RCT adults
(1264)
2 mg or 6 mg bid† Constipation and abdominal pain11 (2 mg)
7 (6 mg)
Renzapride11Pilot study adults
(17)
Escalating dose: 2 mg daily to 2 mg bid† Abdominal pain and bloatingNot enough information to calculate
Renzapride12Parallel group adults
(48)
1, 2, or 4 mg dailyü Colonic transit; stool form and ease of passage, drug=placeboN/A
BM, bowel movement; N/A, not available; NNT, number needed to treat; RCT, randomized controlled trial.

Pediatric constipation: Erythromycin helps; watch dosage

A small RCT of 14 children between 4 and 13 years of age showed that erythromycin improved symptoms of constipation and decreased laxative use (number needed to treat [NNT]=10).1 Two RCTs in neonates demonstrated that erythromycin shortened intestinal transit time and improved feeding tolerance.2,3

The erythromycin dose used in these studies was lower than the dosage for antibiotic purposes; no adverse effects were reported. However, cardiac arrhythmias and death have occurred when erythromycin is given to adults and children at the usual antibiotic doses.4

 

 

Adult constipation: The options are limited

One RCT of cisapride for constipation showed that it improved symptoms,5 whereas another demonstrated no significant difference between cisapride and placebo in constipation-predominant irritable bowel syndrome.6 Reports of fatal arrhythmias have prompted restrictions on the use of the drug.

In 2 RCTs of tegaserod for constipation, patients exhibited improved abdominal symptoms and increased spontaneous bowel movements (NNT=6 for 2 mg and 5 for 6 mg in the first study; NNT=11 for 2 mg and 7 for 6 mg in the second study).7,8 A pooled analysis of RCTs of tegaserod revealed an increase in cardiovascular events, prompting withdrawal of the drug from the market (number needed to harm=1000).9 Tegaserod is available only for emergency and investigational use.

Renzapride, a newer prokinetic similar to cisapride, is under investigation. It is one tenth the strength of cisapride and carries a lower potential risk of cardiac complications.10 Two small placebo-controlled trials demonstrated improved abdominal pain and stool consistency, but only 1 showed statistically significant results compared with placebo.11,12

Recommendations

The North American Society for Pediatric Gastroenterology, Hepatology, and Nutrition states that the benefits of cisapride do not outweigh the risks.13 The American College of Gastroenterology’s Chronic Constipation Task Force states that tegaserod effectively treats chronic constipation.14 Neither guideline includes recommendations regarding other prokinetic agents.

References

1. Bellomo-Brandao MA, Collares EF, da-Costa-Pinto EA. Use of erythromycin for the treatment of severe chronic constipation in children. Braz J Med Biol Res. 2003;36:1391-1396.

2. Costalos C, Gounaris A, Varhalama E, et al. Erythromycin as a prokinetic agent in preterm infants. J Pediatr Gastroenterol Nutr. 2002;34:23-25.

3. Costalos C, Gavrili V, Skouteri V, et al. The effect of low-dose erythromycin on whole gastrointestinal transit time of preterm infants. Early Hum Dev. 2001;65:91-96.

4. Ray WA, Murray KT, Meredith S, et al. Oral erythromycin and the risk of sudden death from cardiac causes. N Engl J Med. 2004;351:1089-1096.

5. Van Outryve M, Milo R, Toussaint J, et al. “Prokinetic” treatment of constipation-predominant irritable bowel syndrome: a placebo-controlled study of cisapride. J Clin Gastroenterol. 1991;13:49-57.

6. Ziegenhagen DJ, Kruis W. Cisapride treatment of constipation-predominant irritable bowel syndrome is not superior to placebo. J Gastroenterol Hepatol. 2004;19:744-749.

7. Johanson JF, Wald A, Tougas G, et al. Effect of tegaserod in chronic constipation: a randomized, double-blind, controlled trial. Clin Gastroenterol Hepatol. 2004;2:796-805.

8. Kamm MA, Müller-Lissner S, Talley NJ, et al. Tegaserod for the treatment of chronic constipation: a randomized, double-blind, placebo-controlled multinational study. Am J Gastroenterol. 2005;100:362-372.

9. US Food and Drug Administration, Center for Drug Evaluation and Research. FDA Public Health Advisory: tegaserod maleate. March 30, 2007. Available at: www.fda.gov/cder/drug/advisory/tegaserod.htm. Accessed November 17, 2007.

10. Galligan JJ, Vanner S. Basic and clinical pharmacology of new motility promoting agents. Neurogastroenterol Motil. 2005;17:643-653.

11. Tack J, Middleton SJ, Horne MC, et al. Pilot study of the efficacy of renzapride on gastrointestinal motility and symptoms in patients with constipation-predominant irritable bowel syndrome. Aliment Pharmacol Ther. 2006;23:1655-1665.

12. Camilleri M, McKinzie S, Fox J, et al. Effect of renzapride on transit in constipation-predominant irritable bowel syndrome. Clin Gastroenterol Hepatol. 2004;2:895-904.

13. North American Society for Pediatric Gastroenterology, Hepatology and Nutrition. Evaluation and treatment of constipation in children: summary of updated recommendations of the North American Society for Pediatric Gastroenterology, Hepatology and Nutrition. J Pediatr Gastroenterol Nutr. 2006;43:405-407.

14. American College of Gastroenterology Chronic Constipation Task Force. An evidence-based approach to the management of chronic constipation in North America. Am J Gastroenterol. 2005;100(suppl 1):S1-S4.

Article PDF
Author and Disclosure Information

Ginger Boyle, MD
Spartanburg Family Medicine Residency Program, Spartanburg, SC

Anne Mounsey, MD
University of North Carolina, Chapel Hill

Karen Crowell, MLIS
University of North Carolina, Chapel Hill

Issue
The Journal of Family Practice - 58(4)
Publications
Page Number
220d-220f
Sections
Author and Disclosure Information

Ginger Boyle, MD
Spartanburg Family Medicine Residency Program, Spartanburg, SC

Anne Mounsey, MD
University of North Carolina, Chapel Hill

Karen Crowell, MLIS
University of North Carolina, Chapel Hill

Author and Disclosure Information

Ginger Boyle, MD
Spartanburg Family Medicine Residency Program, Spartanburg, SC

Anne Mounsey, MD
University of North Carolina, Chapel Hill

Karen Crowell, MLIS
University of North Carolina, Chapel Hill

Article PDF
Article PDF
EVIDENCE-BASED ANSWER

Erythromycin has a limited role in treating pediatric patients (strength of recommendation [SOR]: B, limited-quality, patient-oriented evidence). Tegaserod and cisapride are the only prokinetic agents available for constipated adults (SOR: A, consistent, good-quality, patient-oriented evidence for tegaserod; SOR: B, for cisapride), but cardiovascular risk restricts prescribing of both medications.

Evidence summary

Prokinetic agents promote transit of intestinal contents by increasing the frequency or strength of small intestine contractions. Available prokinetics include erythromycin and metoclopramide. Metoclopramide has been tested only for upper gastrointestinal mobility. The only randomized controlled trials (RCTs) of erythromycin for constipation have been conducted in children. Cisapride and tegaserod have been withdrawn from general use because of adverse side effects. The TABLE summarizes the available data.

TABLE
Prokinetics for constipation: What the research tells us

DRUGDESIGN (N)DOSEOUTCOMENNT
Erythromycin estolate1Crossover children
(14)
20 mg/kg/day divided qid†Constipation and laxative use10
Cisapride5RCT adults
(69)
5-10 mg tidü Spontaneous BM
† Abdominal pain
4
Cisapride6RCT adults
(82)
5-10 mg tidAbdominal pain and constipation, drug=placeboN/A
Tegaserod7RCT adults
(1348)
2 mg or 6 mg bid† Constipation
ü Spontaneous BM
6 (2 mg)
5 (6 mg)
Tegaserod8RCT adults
(1264)
2 mg or 6 mg bid† Constipation and abdominal pain11 (2 mg)
7 (6 mg)
Renzapride11Pilot study adults
(17)
Escalating dose: 2 mg daily to 2 mg bid† Abdominal pain and bloatingNot enough information to calculate
Renzapride12Parallel group adults
(48)
1, 2, or 4 mg dailyü Colonic transit; stool form and ease of passage, drug=placeboN/A
BM, bowel movement; N/A, not available; NNT, number needed to treat; RCT, randomized controlled trial.

Pediatric constipation: Erythromycin helps; watch dosage

A small RCT of 14 children between 4 and 13 years of age showed that erythromycin improved symptoms of constipation and decreased laxative use (number needed to treat [NNT]=10).1 Two RCTs in neonates demonstrated that erythromycin shortened intestinal transit time and improved feeding tolerance.2,3

The erythromycin dose used in these studies was lower than the dosage for antibiotic purposes; no adverse effects were reported. However, cardiac arrhythmias and death have occurred when erythromycin is given to adults and children at the usual antibiotic doses.4

 

 

Adult constipation: The options are limited

One RCT of cisapride for constipation showed that it improved symptoms,5 whereas another demonstrated no significant difference between cisapride and placebo in constipation-predominant irritable bowel syndrome.6 Reports of fatal arrhythmias have prompted restrictions on the use of the drug.

In 2 RCTs of tegaserod for constipation, patients exhibited improved abdominal symptoms and increased spontaneous bowel movements (NNT=6 for 2 mg and 5 for 6 mg in the first study; NNT=11 for 2 mg and 7 for 6 mg in the second study).7,8 A pooled analysis of RCTs of tegaserod revealed an increase in cardiovascular events, prompting withdrawal of the drug from the market (number needed to harm=1000).9 Tegaserod is available only for emergency and investigational use.

Renzapride, a newer prokinetic similar to cisapride, is under investigation. It is one tenth the strength of cisapride and carries a lower potential risk of cardiac complications.10 Two small placebo-controlled trials demonstrated improved abdominal pain and stool consistency, but only 1 showed statistically significant results compared with placebo.11,12

Recommendations

The North American Society for Pediatric Gastroenterology, Hepatology, and Nutrition states that the benefits of cisapride do not outweigh the risks.13 The American College of Gastroenterology’s Chronic Constipation Task Force states that tegaserod effectively treats chronic constipation.14 Neither guideline includes recommendations regarding other prokinetic agents.

EVIDENCE-BASED ANSWER

Erythromycin has a limited role in treating pediatric patients (strength of recommendation [SOR]: B, limited-quality, patient-oriented evidence). Tegaserod and cisapride are the only prokinetic agents available for constipated adults (SOR: A, consistent, good-quality, patient-oriented evidence for tegaserod; SOR: B, for cisapride), but cardiovascular risk restricts prescribing of both medications.

Evidence summary

Prokinetic agents promote transit of intestinal contents by increasing the frequency or strength of small intestine contractions. Available prokinetics include erythromycin and metoclopramide. Metoclopramide has been tested only for upper gastrointestinal mobility. The only randomized controlled trials (RCTs) of erythromycin for constipation have been conducted in children. Cisapride and tegaserod have been withdrawn from general use because of adverse side effects. The TABLE summarizes the available data.

TABLE
Prokinetics for constipation: What the research tells us

DRUGDESIGN (N)DOSEOUTCOMENNT
Erythromycin estolate1Crossover children
(14)
20 mg/kg/day divided qid†Constipation and laxative use10
Cisapride5RCT adults
(69)
5-10 mg tidü Spontaneous BM
† Abdominal pain
4
Cisapride6RCT adults
(82)
5-10 mg tidAbdominal pain and constipation, drug=placeboN/A
Tegaserod7RCT adults
(1348)
2 mg or 6 mg bid† Constipation
ü Spontaneous BM
6 (2 mg)
5 (6 mg)
Tegaserod8RCT adults
(1264)
2 mg or 6 mg bid† Constipation and abdominal pain11 (2 mg)
7 (6 mg)
Renzapride11Pilot study adults
(17)
Escalating dose: 2 mg daily to 2 mg bid† Abdominal pain and bloatingNot enough information to calculate
Renzapride12Parallel group adults
(48)
1, 2, or 4 mg dailyü Colonic transit; stool form and ease of passage, drug=placeboN/A
BM, bowel movement; N/A, not available; NNT, number needed to treat; RCT, randomized controlled trial.

Pediatric constipation: Erythromycin helps; watch dosage

A small RCT of 14 children between 4 and 13 years of age showed that erythromycin improved symptoms of constipation and decreased laxative use (number needed to treat [NNT]=10).1 Two RCTs in neonates demonstrated that erythromycin shortened intestinal transit time and improved feeding tolerance.2,3

The erythromycin dose used in these studies was lower than the dosage for antibiotic purposes; no adverse effects were reported. However, cardiac arrhythmias and death have occurred when erythromycin is given to adults and children at the usual antibiotic doses.4

 

 

Adult constipation: The options are limited

One RCT of cisapride for constipation showed that it improved symptoms,5 whereas another demonstrated no significant difference between cisapride and placebo in constipation-predominant irritable bowel syndrome.6 Reports of fatal arrhythmias have prompted restrictions on the use of the drug.

In 2 RCTs of tegaserod for constipation, patients exhibited improved abdominal symptoms and increased spontaneous bowel movements (NNT=6 for 2 mg and 5 for 6 mg in the first study; NNT=11 for 2 mg and 7 for 6 mg in the second study).7,8 A pooled analysis of RCTs of tegaserod revealed an increase in cardiovascular events, prompting withdrawal of the drug from the market (number needed to harm=1000).9 Tegaserod is available only for emergency and investigational use.

Renzapride, a newer prokinetic similar to cisapride, is under investigation. It is one tenth the strength of cisapride and carries a lower potential risk of cardiac complications.10 Two small placebo-controlled trials demonstrated improved abdominal pain and stool consistency, but only 1 showed statistically significant results compared with placebo.11,12

Recommendations

The North American Society for Pediatric Gastroenterology, Hepatology, and Nutrition states that the benefits of cisapride do not outweigh the risks.13 The American College of Gastroenterology’s Chronic Constipation Task Force states that tegaserod effectively treats chronic constipation.14 Neither guideline includes recommendations regarding other prokinetic agents.

References

1. Bellomo-Brandao MA, Collares EF, da-Costa-Pinto EA. Use of erythromycin for the treatment of severe chronic constipation in children. Braz J Med Biol Res. 2003;36:1391-1396.

2. Costalos C, Gounaris A, Varhalama E, et al. Erythromycin as a prokinetic agent in preterm infants. J Pediatr Gastroenterol Nutr. 2002;34:23-25.

3. Costalos C, Gavrili V, Skouteri V, et al. The effect of low-dose erythromycin on whole gastrointestinal transit time of preterm infants. Early Hum Dev. 2001;65:91-96.

4. Ray WA, Murray KT, Meredith S, et al. Oral erythromycin and the risk of sudden death from cardiac causes. N Engl J Med. 2004;351:1089-1096.

5. Van Outryve M, Milo R, Toussaint J, et al. “Prokinetic” treatment of constipation-predominant irritable bowel syndrome: a placebo-controlled study of cisapride. J Clin Gastroenterol. 1991;13:49-57.

6. Ziegenhagen DJ, Kruis W. Cisapride treatment of constipation-predominant irritable bowel syndrome is not superior to placebo. J Gastroenterol Hepatol. 2004;19:744-749.

7. Johanson JF, Wald A, Tougas G, et al. Effect of tegaserod in chronic constipation: a randomized, double-blind, controlled trial. Clin Gastroenterol Hepatol. 2004;2:796-805.

8. Kamm MA, Müller-Lissner S, Talley NJ, et al. Tegaserod for the treatment of chronic constipation: a randomized, double-blind, placebo-controlled multinational study. Am J Gastroenterol. 2005;100:362-372.

9. US Food and Drug Administration, Center for Drug Evaluation and Research. FDA Public Health Advisory: tegaserod maleate. March 30, 2007. Available at: www.fda.gov/cder/drug/advisory/tegaserod.htm. Accessed November 17, 2007.

10. Galligan JJ, Vanner S. Basic and clinical pharmacology of new motility promoting agents. Neurogastroenterol Motil. 2005;17:643-653.

11. Tack J, Middleton SJ, Horne MC, et al. Pilot study of the efficacy of renzapride on gastrointestinal motility and symptoms in patients with constipation-predominant irritable bowel syndrome. Aliment Pharmacol Ther. 2006;23:1655-1665.

12. Camilleri M, McKinzie S, Fox J, et al. Effect of renzapride on transit in constipation-predominant irritable bowel syndrome. Clin Gastroenterol Hepatol. 2004;2:895-904.

13. North American Society for Pediatric Gastroenterology, Hepatology and Nutrition. Evaluation and treatment of constipation in children: summary of updated recommendations of the North American Society for Pediatric Gastroenterology, Hepatology and Nutrition. J Pediatr Gastroenterol Nutr. 2006;43:405-407.

14. American College of Gastroenterology Chronic Constipation Task Force. An evidence-based approach to the management of chronic constipation in North America. Am J Gastroenterol. 2005;100(suppl 1):S1-S4.

References

1. Bellomo-Brandao MA, Collares EF, da-Costa-Pinto EA. Use of erythromycin for the treatment of severe chronic constipation in children. Braz J Med Biol Res. 2003;36:1391-1396.

2. Costalos C, Gounaris A, Varhalama E, et al. Erythromycin as a prokinetic agent in preterm infants. J Pediatr Gastroenterol Nutr. 2002;34:23-25.

3. Costalos C, Gavrili V, Skouteri V, et al. The effect of low-dose erythromycin on whole gastrointestinal transit time of preterm infants. Early Hum Dev. 2001;65:91-96.

4. Ray WA, Murray KT, Meredith S, et al. Oral erythromycin and the risk of sudden death from cardiac causes. N Engl J Med. 2004;351:1089-1096.

5. Van Outryve M, Milo R, Toussaint J, et al. “Prokinetic” treatment of constipation-predominant irritable bowel syndrome: a placebo-controlled study of cisapride. J Clin Gastroenterol. 1991;13:49-57.

6. Ziegenhagen DJ, Kruis W. Cisapride treatment of constipation-predominant irritable bowel syndrome is not superior to placebo. J Gastroenterol Hepatol. 2004;19:744-749.

7. Johanson JF, Wald A, Tougas G, et al. Effect of tegaserod in chronic constipation: a randomized, double-blind, controlled trial. Clin Gastroenterol Hepatol. 2004;2:796-805.

8. Kamm MA, Müller-Lissner S, Talley NJ, et al. Tegaserod for the treatment of chronic constipation: a randomized, double-blind, placebo-controlled multinational study. Am J Gastroenterol. 2005;100:362-372.

9. US Food and Drug Administration, Center for Drug Evaluation and Research. FDA Public Health Advisory: tegaserod maleate. March 30, 2007. Available at: www.fda.gov/cder/drug/advisory/tegaserod.htm. Accessed November 17, 2007.

10. Galligan JJ, Vanner S. Basic and clinical pharmacology of new motility promoting agents. Neurogastroenterol Motil. 2005;17:643-653.

11. Tack J, Middleton SJ, Horne MC, et al. Pilot study of the efficacy of renzapride on gastrointestinal motility and symptoms in patients with constipation-predominant irritable bowel syndrome. Aliment Pharmacol Ther. 2006;23:1655-1665.

12. Camilleri M, McKinzie S, Fox J, et al. Effect of renzapride on transit in constipation-predominant irritable bowel syndrome. Clin Gastroenterol Hepatol. 2004;2:895-904.

13. North American Society for Pediatric Gastroenterology, Hepatology and Nutrition. Evaluation and treatment of constipation in children: summary of updated recommendations of the North American Society for Pediatric Gastroenterology, Hepatology and Nutrition. J Pediatr Gastroenterol Nutr. 2006;43:405-407.

14. American College of Gastroenterology Chronic Constipation Task Force. An evidence-based approach to the management of chronic constipation in North America. Am J Gastroenterol. 2005;100(suppl 1):S1-S4.

Issue
The Journal of Family Practice - 58(4)
Issue
The Journal of Family Practice - 58(4)
Page Number
220d-220f
Page Number
220d-220f
Publications
Publications
Article Type
Display Headline
What is the role of prokinetic agents for constipation?
Display Headline
What is the role of prokinetic agents for constipation?
Sections
PURLs Copyright

Evidence-based answers from the Family Physicians Inquiries Network

Disallow All Ads
Article PDF Media

Arthroscopic surgery for knee osteoarthritis? Just say no

Article Type
Changed
Fri, 06/19/2020 - 14:15
Display Headline
Arthroscopic surgery for knee osteoarthritis? Just say no
 

ILLUSTRATIVE CASE

A 52-year-old man comes to your office complaining of ongoing knee pain—pain that he knows is related to his osteoarthritis (OA). The patient does not want a total knee replacement, and it’s unlikely that his arthritis is extensive enough to warrant it. You wonder whether he’s a potential candidate for arthroscopic knee surgery and if the lavage and articular cartilage debridement the procedure entails would alleviate his symptoms.

Knee pain related to OA is a common complaint in the office setting, and primary care physicians use many medical and physical interventions to manage the symptoms. If these fall short in patients with more advanced disease, however, physicians often recommend an orthopedic surgery consult to consider surgical management.

Lavage and debridement: The (questionable) effects

Arthroscopic knee surgery involves lavage (to remove particulate material, such as cartilage fragments) and debridement (to smooth the articular surfaces). Theoretically, this widely used surgery reduces synovitis and improves joint motion, resulting in a decrease in pain and an improvement in function. But what does the latest research tell us?

A randomized controlled trial (RCT) by Moseley et al in 2002 found arthroscopic knee surgery to be of no benefit for moderate to severe OA.2 Because this finding was so contrary to current practice, the authors’ conclusion was not widely accepted. Arthroscopic surgery continued to be used for moderately severe knee arthritis.3 Indeed, the 2008 guidelines from the American Academy of Orthopaedic Surgeons (AAOS) state that “arthroscopic partial meniscectomy or loose body removal is an option in patients with symptomatic OA of the knee who also have primary signs and symptoms of a torn meniscus and/or a loose body.”4

However, these guidelines do not include the evidence from the study by Kirkley et al1 detailed below.

STUDY SUMMARY: New RCT echoes earlier conclusion

Kirkley et al conducted a nonblinded RCT of 188 patients with moderate to severe OA of the knee; those with large meniscal tears, malalignment, previous arthroscopic surgery, or severe bicompartmental arthritis were excluded.

The control group received optimal medical and physical therapy, consisting of 1 hour of physical therapy a week, twice-daily exercises, and stepwise use of acetaminophen, nonsteroidal anti-inflammatory drugs (NSAIDs), and intraarticular hyaluronic acid injections. The intervention group had arthroscopic surgery (debridement of articular cartilage and menisci, excision of osteophytes, and removal of loose bodies), and received medical and physical therapy.

The primary outcome measure was the validated Western Ontario and McMaster Universities Arthritis Index (WOMAC) score. (The range is 0 to 2400, with higher scores indicating more severe symptoms.)

After 2 years, the researchers found minimal difference in the WOMAC scores of the control group (897±583) and the surgery group (874±624); the absolute difference was –23±605 (95% confidence interval, –208 to 161; P=.22). There was no difference in the secondary outcomes of quality of life, pain, and function. Nor did surgery provide any benefit to the subgroup of patients with mechanical symptoms.1

These findings echoed those of Moseley et al’s 2002 single-blinded RCT, in which researchers assigned 180 patients to arthroscopic surgery or sham surgery, and found surgery to be of no benefit.2 That study was criticized because of its methodology; the researchers used an outcome measure that was not validated and failed to exclude patients with more advanced disease and malalignment, who might be expected to have a poor response to surgery. The 2008 study by Kirkley et al had no such methodological flaws and, in retrospect, it appears that these perceived flaws did not account for the negative findings of the 2002 study.

WHAT’S NEW?: No room for doubt

Evidence from the new RCT confirms the findings of the 2002 trial. It clearly shows that arthroscopic surgery for knee OA is not beneficial, even in patients with mechanical symptoms. Kirkley’s study avoided the criticism of the earlier study by using a validated outcome measure, excluding patients with malalignment, and performing a subgroup analysis of patients with mechanical symptoms. We now have 2 studies that show no benefit from arthroscopic knee surgery in patients with OA, whether or not they have mechanical problems.

So what can you do for patients with moderate to severe knee pain from osteoarthritis? Offer them medical and physical therapy (TABLE) and the assurance that there is nothing to be gained from arthroscopic surgery.

TABLE

How to treat knee OA without surgery

  • Start by offering adequate analgesia. Acetaminophen (4 g/d) is effective at relieving pain in about 75% of patients with knee arthritis. NSAIDs are slightly more effective, but their gastric toxicity is a disadvantage. Combining NSAIDs with a proton pump inhibitor decreases gastric ulceration.
  • Consider the combination of glucosamine and chondroitin. Chondroitin alone is not beneficial; when paired with glucosamine, however, it may provide some pain relief associated with moderate to severe OA.5-7 Patients may also benefit from intra-articular steroid injections, which have been found to be slightly more effective than placebo.8
  • Suggest nonpharmacological therapy. Physical therapy and regular exercise typically reduce pain and improve the function of an arthritic knee.9 Acupuncture may have a small benefit, as well.10
 

 

 

CAVEATS: Large meniscal tears: An exception to the rule

These findings do not necessarily apply to patients with evidence of large meniscal tears. This subset of knee OA patients may benefit from surgical management.

CHALLENGES TO IMPLEMENTATION: What to say to patients seeking a referral

Patients may have read about arthroscopic knee surgery or know someone who underwent the procedure and come to you asking for a referral to an orthopedic surgeon. In such a case, we suggest a straightforward approach.

Discuss arthroscopic surgery’s proven lack of benefit and offer equally effective conservative therapies. For patients who may be eligible for a total knee replacement, a referral to an orthopedic surgeon for evaluation is appropriate.

Acknowledgements

The PURLs Surveillance System is supported in part by Grant Number UL1RR024999 from the National Center for Research Resources, a Clinical Translational Science Award to the University of Chicago. The content is solely the responsibility of the authors and does not necessarily represent the official views of the National Center for Research Resources or the National Institutes of Health.

PURLs methodology

This study was selected and evaluated using FPIN’s Priority Updates from the Research Literature (PURL) Surveillance System methodology. The criteria and findings leading to the selection of this study as a PURL can be accessed at www.jfponline.com/purls.

Files
References

1. Kirkley A, Birmingham TB, Litchfield RB, et al. A randomized trial of arthroscopic surgery for osteoarthritis of the knee. N Engl J Med. 2008;359:1097-1107.

2. Moseley JB, O’Malley K, Petersen NJ, et al. A controlled trial of arthroscopic surgery for osteoarthritis of the knee. N Engl J Med. 2002;347:81-88.

3. Hawker G, Guan J, Judge A, et al. Knee arthroscopy in England and Ontario: patterns of use, changes over time, and relationship to total knee replacement. J Bone Joint Surg Am. 2008;90:2337-2345.

4. American Academy of Orthopaedic Surgeons. Treatment of osteoarthritis of the knee (non-arthroplasty): full guideline. December 6, 2008. Available at: http://www.aaos.org/Research/guidelines/GuidelineOAKnee.asp. Accessed February 16, 2009.

5. Towheed TE, Maxwell L, Anastassiades TP, et al. Glucosamine therapy for treating osteoarthritis. Cochrane Database Syst Rev. 2005;(1):CD002946.-

6. Michel BA, Stucki G, Frey D, et al. Chondroitins 4 and 6 sulfate in osteoarthritis of the knee: a randomized, controlled trial. Arthritis Rheum. 2005;52:779-786.

7. Clegg DO, Reda DJ, Harris CL, et al. Glucosamine, chondroitin sulfate, and the two in combination for painful knee osteoarthritis. N Engl J Med. 2006;354:795-808.

8. Arroll B, Goodyear-Smith F. Corticosteroid injections for osteoarthritis of the knee: meta-analysis. BMJ. 2004;328:869-870.

9. Fransen M, Crosbie J, Edmonds J. Physical therapy is effective for patients with osteoarthritis of the knee: a randomized controlled clinical trial. J Rheumatol. 2001;28:156-164.

10. Berman BM, Lao L, Langenberg P, et al. Effectiveness of acupuncture as adjunctive therapy in osteoarthritis of the knee: a randomized, controlled trial. Ann Intern Med. 2004;141:901-910.

Article PDF
Author and Disclosure Information

Anne Mounsey, MD
Department of Family Medicine, University of North Carolina, Chapel Hill

Bernard Ewigman, MD, MSPH
Department of Family Medicine, The University of Chicago
PURLs EDITOR
John Hickner, MD, MSc
Department of Family Medicine, Cleveland Clinic

Issue
The Journal of Family Practice - 58(3)
Publications
Topics
Page Number
143-144
Legacy Keywords
Anne Mounsey; physical therapy; lavage and debridement; meniscal tears; knee osteoarthritis
Sections
Files
Files
Author and Disclosure Information

Anne Mounsey, MD
Department of Family Medicine, University of North Carolina, Chapel Hill

Bernard Ewigman, MD, MSPH
Department of Family Medicine, The University of Chicago
PURLs EDITOR
John Hickner, MD, MSc
Department of Family Medicine, Cleveland Clinic

Author and Disclosure Information

Anne Mounsey, MD
Department of Family Medicine, University of North Carolina, Chapel Hill

Bernard Ewigman, MD, MSPH
Department of Family Medicine, The University of Chicago
PURLs EDITOR
John Hickner, MD, MSc
Department of Family Medicine, Cleveland Clinic

Article PDF
Article PDF
 

ILLUSTRATIVE CASE

A 52-year-old man comes to your office complaining of ongoing knee pain—pain that he knows is related to his osteoarthritis (OA). The patient does not want a total knee replacement, and it’s unlikely that his arthritis is extensive enough to warrant it. You wonder whether he’s a potential candidate for arthroscopic knee surgery and if the lavage and articular cartilage debridement the procedure entails would alleviate his symptoms.

Knee pain related to OA is a common complaint in the office setting, and primary care physicians use many medical and physical interventions to manage the symptoms. If these fall short in patients with more advanced disease, however, physicians often recommend an orthopedic surgery consult to consider surgical management.

Lavage and debridement: The (questionable) effects

Arthroscopic knee surgery involves lavage (to remove particulate material, such as cartilage fragments) and debridement (to smooth the articular surfaces). Theoretically, this widely used surgery reduces synovitis and improves joint motion, resulting in a decrease in pain and an improvement in function. But what does the latest research tell us?

A randomized controlled trial (RCT) by Moseley et al in 2002 found arthroscopic knee surgery to be of no benefit for moderate to severe OA.2 Because this finding was so contrary to current practice, the authors’ conclusion was not widely accepted. Arthroscopic surgery continued to be used for moderately severe knee arthritis.3 Indeed, the 2008 guidelines from the American Academy of Orthopaedic Surgeons (AAOS) state that “arthroscopic partial meniscectomy or loose body removal is an option in patients with symptomatic OA of the knee who also have primary signs and symptoms of a torn meniscus and/or a loose body.”4

However, these guidelines do not include the evidence from the study by Kirkley et al1 detailed below.

STUDY SUMMARY: New RCT echoes earlier conclusion

Kirkley et al conducted a nonblinded RCT of 188 patients with moderate to severe OA of the knee; those with large meniscal tears, malalignment, previous arthroscopic surgery, or severe bicompartmental arthritis were excluded.

The control group received optimal medical and physical therapy, consisting of 1 hour of physical therapy a week, twice-daily exercises, and stepwise use of acetaminophen, nonsteroidal anti-inflammatory drugs (NSAIDs), and intraarticular hyaluronic acid injections. The intervention group had arthroscopic surgery (debridement of articular cartilage and menisci, excision of osteophytes, and removal of loose bodies), and received medical and physical therapy.

The primary outcome measure was the validated Western Ontario and McMaster Universities Arthritis Index (WOMAC) score. (The range is 0 to 2400, with higher scores indicating more severe symptoms.)

After 2 years, the researchers found minimal difference in the WOMAC scores of the control group (897±583) and the surgery group (874±624); the absolute difference was –23±605 (95% confidence interval, –208 to 161; P=.22). There was no difference in the secondary outcomes of quality of life, pain, and function. Nor did surgery provide any benefit to the subgroup of patients with mechanical symptoms.1

These findings echoed those of Moseley et al’s 2002 single-blinded RCT, in which researchers assigned 180 patients to arthroscopic surgery or sham surgery, and found surgery to be of no benefit.2 That study was criticized because of its methodology; the researchers used an outcome measure that was not validated and failed to exclude patients with more advanced disease and malalignment, who might be expected to have a poor response to surgery. The 2008 study by Kirkley et al had no such methodological flaws and, in retrospect, it appears that these perceived flaws did not account for the negative findings of the 2002 study.

WHAT’S NEW?: No room for doubt

Evidence from the new RCT confirms the findings of the 2002 trial. It clearly shows that arthroscopic surgery for knee OA is not beneficial, even in patients with mechanical symptoms. Kirkley’s study avoided the criticism of the earlier study by using a validated outcome measure, excluding patients with malalignment, and performing a subgroup analysis of patients with mechanical symptoms. We now have 2 studies that show no benefit from arthroscopic knee surgery in patients with OA, whether or not they have mechanical problems.

So what can you do for patients with moderate to severe knee pain from osteoarthritis? Offer them medical and physical therapy (TABLE) and the assurance that there is nothing to be gained from arthroscopic surgery.

TABLE

How to treat knee OA without surgery

  • Start by offering adequate analgesia. Acetaminophen (4 g/d) is effective at relieving pain in about 75% of patients with knee arthritis. NSAIDs are slightly more effective, but their gastric toxicity is a disadvantage. Combining NSAIDs with a proton pump inhibitor decreases gastric ulceration.
  • Consider the combination of glucosamine and chondroitin. Chondroitin alone is not beneficial; when paired with glucosamine, however, it may provide some pain relief associated with moderate to severe OA.5-7 Patients may also benefit from intra-articular steroid injections, which have been found to be slightly more effective than placebo.8
  • Suggest nonpharmacological therapy. Physical therapy and regular exercise typically reduce pain and improve the function of an arthritic knee.9 Acupuncture may have a small benefit, as well.10
 

 

 

CAVEATS: Large meniscal tears: An exception to the rule

These findings do not necessarily apply to patients with evidence of large meniscal tears. This subset of knee OA patients may benefit from surgical management.

CHALLENGES TO IMPLEMENTATION: What to say to patients seeking a referral

Patients may have read about arthroscopic knee surgery or know someone who underwent the procedure and come to you asking for a referral to an orthopedic surgeon. In such a case, we suggest a straightforward approach.

Discuss arthroscopic surgery’s proven lack of benefit and offer equally effective conservative therapies. For patients who may be eligible for a total knee replacement, a referral to an orthopedic surgeon for evaluation is appropriate.

Acknowledgements

The PURLs Surveillance System is supported in part by Grant Number UL1RR024999 from the National Center for Research Resources, a Clinical Translational Science Award to the University of Chicago. The content is solely the responsibility of the authors and does not necessarily represent the official views of the National Center for Research Resources or the National Institutes of Health.

PURLs methodology

This study was selected and evaluated using FPIN’s Priority Updates from the Research Literature (PURL) Surveillance System methodology. The criteria and findings leading to the selection of this study as a PURL can be accessed at www.jfponline.com/purls.

 

ILLUSTRATIVE CASE

A 52-year-old man comes to your office complaining of ongoing knee pain—pain that he knows is related to his osteoarthritis (OA). The patient does not want a total knee replacement, and it’s unlikely that his arthritis is extensive enough to warrant it. You wonder whether he’s a potential candidate for arthroscopic knee surgery and if the lavage and articular cartilage debridement the procedure entails would alleviate his symptoms.

Knee pain related to OA is a common complaint in the office setting, and primary care physicians use many medical and physical interventions to manage the symptoms. If these fall short in patients with more advanced disease, however, physicians often recommend an orthopedic surgery consult to consider surgical management.

Lavage and debridement: The (questionable) effects

Arthroscopic knee surgery involves lavage (to remove particulate material, such as cartilage fragments) and debridement (to smooth the articular surfaces). Theoretically, this widely used surgery reduces synovitis and improves joint motion, resulting in a decrease in pain and an improvement in function. But what does the latest research tell us?

A randomized controlled trial (RCT) by Moseley et al in 2002 found arthroscopic knee surgery to be of no benefit for moderate to severe OA.2 Because this finding was so contrary to current practice, the authors’ conclusion was not widely accepted. Arthroscopic surgery continued to be used for moderately severe knee arthritis.3 Indeed, the 2008 guidelines from the American Academy of Orthopaedic Surgeons (AAOS) state that “arthroscopic partial meniscectomy or loose body removal is an option in patients with symptomatic OA of the knee who also have primary signs and symptoms of a torn meniscus and/or a loose body.”4

However, these guidelines do not include the evidence from the study by Kirkley et al1 detailed below.

STUDY SUMMARY: New RCT echoes earlier conclusion

Kirkley et al conducted a nonblinded RCT of 188 patients with moderate to severe OA of the knee; those with large meniscal tears, malalignment, previous arthroscopic surgery, or severe bicompartmental arthritis were excluded.

The control group received optimal medical and physical therapy, consisting of 1 hour of physical therapy a week, twice-daily exercises, and stepwise use of acetaminophen, nonsteroidal anti-inflammatory drugs (NSAIDs), and intraarticular hyaluronic acid injections. The intervention group had arthroscopic surgery (debridement of articular cartilage and menisci, excision of osteophytes, and removal of loose bodies), and received medical and physical therapy.

The primary outcome measure was the validated Western Ontario and McMaster Universities Arthritis Index (WOMAC) score. (The range is 0 to 2400, with higher scores indicating more severe symptoms.)

After 2 years, the researchers found minimal difference in the WOMAC scores of the control group (897±583) and the surgery group (874±624); the absolute difference was –23±605 (95% confidence interval, –208 to 161; P=.22). There was no difference in the secondary outcomes of quality of life, pain, and function. Nor did surgery provide any benefit to the subgroup of patients with mechanical symptoms.1

These findings echoed those of Moseley et al’s 2002 single-blinded RCT, in which researchers assigned 180 patients to arthroscopic surgery or sham surgery, and found surgery to be of no benefit.2 That study was criticized because of its methodology; the researchers used an outcome measure that was not validated and failed to exclude patients with more advanced disease and malalignment, who might be expected to have a poor response to surgery. The 2008 study by Kirkley et al had no such methodological flaws and, in retrospect, it appears that these perceived flaws did not account for the negative findings of the 2002 study.

WHAT’S NEW?: No room for doubt

Evidence from the new RCT confirms the findings of the 2002 trial. It clearly shows that arthroscopic surgery for knee OA is not beneficial, even in patients with mechanical symptoms. Kirkley’s study avoided the criticism of the earlier study by using a validated outcome measure, excluding patients with malalignment, and performing a subgroup analysis of patients with mechanical symptoms. We now have 2 studies that show no benefit from arthroscopic knee surgery in patients with OA, whether or not they have mechanical problems.

So what can you do for patients with moderate to severe knee pain from osteoarthritis? Offer them medical and physical therapy (TABLE) and the assurance that there is nothing to be gained from arthroscopic surgery.

TABLE

How to treat knee OA without surgery

  • Start by offering adequate analgesia. Acetaminophen (4 g/d) is effective at relieving pain in about 75% of patients with knee arthritis. NSAIDs are slightly more effective, but their gastric toxicity is a disadvantage. Combining NSAIDs with a proton pump inhibitor decreases gastric ulceration.
  • Consider the combination of glucosamine and chondroitin. Chondroitin alone is not beneficial; when paired with glucosamine, however, it may provide some pain relief associated with moderate to severe OA.5-7 Patients may also benefit from intra-articular steroid injections, which have been found to be slightly more effective than placebo.8
  • Suggest nonpharmacological therapy. Physical therapy and regular exercise typically reduce pain and improve the function of an arthritic knee.9 Acupuncture may have a small benefit, as well.10
 

 

 

CAVEATS: Large meniscal tears: An exception to the rule

These findings do not necessarily apply to patients with evidence of large meniscal tears. This subset of knee OA patients may benefit from surgical management.

CHALLENGES TO IMPLEMENTATION: What to say to patients seeking a referral

Patients may have read about arthroscopic knee surgery or know someone who underwent the procedure and come to you asking for a referral to an orthopedic surgeon. In such a case, we suggest a straightforward approach.

Discuss arthroscopic surgery’s proven lack of benefit and offer equally effective conservative therapies. For patients who may be eligible for a total knee replacement, a referral to an orthopedic surgeon for evaluation is appropriate.

Acknowledgements

The PURLs Surveillance System is supported in part by Grant Number UL1RR024999 from the National Center for Research Resources, a Clinical Translational Science Award to the University of Chicago. The content is solely the responsibility of the authors and does not necessarily represent the official views of the National Center for Research Resources or the National Institutes of Health.

PURLs methodology

This study was selected and evaluated using FPIN’s Priority Updates from the Research Literature (PURL) Surveillance System methodology. The criteria and findings leading to the selection of this study as a PURL can be accessed at www.jfponline.com/purls.

References

1. Kirkley A, Birmingham TB, Litchfield RB, et al. A randomized trial of arthroscopic surgery for osteoarthritis of the knee. N Engl J Med. 2008;359:1097-1107.

2. Moseley JB, O’Malley K, Petersen NJ, et al. A controlled trial of arthroscopic surgery for osteoarthritis of the knee. N Engl J Med. 2002;347:81-88.

3. Hawker G, Guan J, Judge A, et al. Knee arthroscopy in England and Ontario: patterns of use, changes over time, and relationship to total knee replacement. J Bone Joint Surg Am. 2008;90:2337-2345.

4. American Academy of Orthopaedic Surgeons. Treatment of osteoarthritis of the knee (non-arthroplasty): full guideline. December 6, 2008. Available at: http://www.aaos.org/Research/guidelines/GuidelineOAKnee.asp. Accessed February 16, 2009.

5. Towheed TE, Maxwell L, Anastassiades TP, et al. Glucosamine therapy for treating osteoarthritis. Cochrane Database Syst Rev. 2005;(1):CD002946.-

6. Michel BA, Stucki G, Frey D, et al. Chondroitins 4 and 6 sulfate in osteoarthritis of the knee: a randomized, controlled trial. Arthritis Rheum. 2005;52:779-786.

7. Clegg DO, Reda DJ, Harris CL, et al. Glucosamine, chondroitin sulfate, and the two in combination for painful knee osteoarthritis. N Engl J Med. 2006;354:795-808.

8. Arroll B, Goodyear-Smith F. Corticosteroid injections for osteoarthritis of the knee: meta-analysis. BMJ. 2004;328:869-870.

9. Fransen M, Crosbie J, Edmonds J. Physical therapy is effective for patients with osteoarthritis of the knee: a randomized controlled clinical trial. J Rheumatol. 2001;28:156-164.

10. Berman BM, Lao L, Langenberg P, et al. Effectiveness of acupuncture as adjunctive therapy in osteoarthritis of the knee: a randomized, controlled trial. Ann Intern Med. 2004;141:901-910.

References

1. Kirkley A, Birmingham TB, Litchfield RB, et al. A randomized trial of arthroscopic surgery for osteoarthritis of the knee. N Engl J Med. 2008;359:1097-1107.

2. Moseley JB, O’Malley K, Petersen NJ, et al. A controlled trial of arthroscopic surgery for osteoarthritis of the knee. N Engl J Med. 2002;347:81-88.

3. Hawker G, Guan J, Judge A, et al. Knee arthroscopy in England and Ontario: patterns of use, changes over time, and relationship to total knee replacement. J Bone Joint Surg Am. 2008;90:2337-2345.

4. American Academy of Orthopaedic Surgeons. Treatment of osteoarthritis of the knee (non-arthroplasty): full guideline. December 6, 2008. Available at: http://www.aaos.org/Research/guidelines/GuidelineOAKnee.asp. Accessed February 16, 2009.

5. Towheed TE, Maxwell L, Anastassiades TP, et al. Glucosamine therapy for treating osteoarthritis. Cochrane Database Syst Rev. 2005;(1):CD002946.-

6. Michel BA, Stucki G, Frey D, et al. Chondroitins 4 and 6 sulfate in osteoarthritis of the knee: a randomized, controlled trial. Arthritis Rheum. 2005;52:779-786.

7. Clegg DO, Reda DJ, Harris CL, et al. Glucosamine, chondroitin sulfate, and the two in combination for painful knee osteoarthritis. N Engl J Med. 2006;354:795-808.

8. Arroll B, Goodyear-Smith F. Corticosteroid injections for osteoarthritis of the knee: meta-analysis. BMJ. 2004;328:869-870.

9. Fransen M, Crosbie J, Edmonds J. Physical therapy is effective for patients with osteoarthritis of the knee: a randomized controlled clinical trial. J Rheumatol. 2001;28:156-164.

10. Berman BM, Lao L, Langenberg P, et al. Effectiveness of acupuncture as adjunctive therapy in osteoarthritis of the knee: a randomized, controlled trial. Ann Intern Med. 2004;141:901-910.

Issue
The Journal of Family Practice - 58(3)
Issue
The Journal of Family Practice - 58(3)
Page Number
143-144
Page Number
143-144
Publications
Publications
Topics
Article Type
Display Headline
Arthroscopic surgery for knee osteoarthritis? Just say no
Display Headline
Arthroscopic surgery for knee osteoarthritis? Just say no
Legacy Keywords
Anne Mounsey; physical therapy; lavage and debridement; meniscal tears; knee osteoarthritis
Legacy Keywords
Anne Mounsey; physical therapy; lavage and debridement; meniscal tears; knee osteoarthritis
Sections
PURLs Copyright

Copyright © 2009 The Family Physicians Inquiries Network.
All rights reserved.

Disallow All Ads
Alternative CME
Use ProPublica
Hide sidebar & use full width
render the right sidebar.
Conference Recap Checkbox
Not Conference Recap
Clinical Edge
Article PDF Media
Media Files